Download as pdf or txt
Download as pdf or txt
You are on page 1of 147

Cardiology 4

Infective Endocarditis (2020) 4


Atrial Fibrillation (AF) 6
Difficult discharge post syncopal episode (2020) 9
Heart failure (2020) 10
Pre-conception and hypertension counselling (2020) 14
Respiratory 16
Recurrent DVTs (2020) 16
VTE Prophylaxis (2020) 17
Pulmonary Embolism (2020) 18
Post-MI complication 20
Obstructive Sleep Apnoea (2020) 22
COPD Exacerbation (2020) 25
Pleural Effusion (2020) 27
Fever and myalgia post-coronary stenting (2020) 29
Haematology 31
Unstable Angina and Anaemia (2020) 31
Macrocytic Anaemia (2020) 33
Thrombocytopaenia (2020) 35
Drug Induced Neutropenia (2020) 38
Acute Transfusion Reaction (2020) 40
Pre-op warfarin management (2020) 43
Gastroenterology 45
Ascites (2020) 45
Hepatocellular Carcinoma (2020) 48
Massive Haematemesis (2020) 50
Ulcerative colitis (2020) 52
Renal 54
Acute Kidney Injury – AKI (2020) 54
Urinary retention (2020) 57
Hyperkalaemia post-op (2020) 58
Hyperkalaemia in CKD 60
Hypokalaemia 2o to Vomiting (2020) 62
Hyponatraemia (2020) 64
Hypercalcaemia (2020) 69
Hypocalcaemia post-denosumab injection (2020) 72
Peritoneal Dialysis Peritonitis (2020) 75
Rhabdomyolysis (2020) 78
UTI (Recurrent) 80
Neurology 81
Status Epilepticus/Seizure 2o to Viral Encephalitis (2020) 81
TIA (2020) 84
Chronic Daily Headache (2020) 86
Back Pain (2020) 89
Parkinson’s Disease 92
Recurring dizziness with autonomic failure (2020) 94
Rheumatology 96
Statin Induced Myopathy (2020) 96
Rheumatoid arthritis 98
Methotrexate Monitoring (2020) 102
SLE and Pleuritic Chest Pain (2020) 104
Palliative Care 107
End of Life Care (2020) 107
Guardianship Act 109
Geriatrics 111
Recurrent Falls 111
Fall/Fracture (Autonomic Failure) 113
Delirium [Altered mental status post cellulitis] (2020) 114
Altered mental status post fall (2020) 117
2
Dementia (2020) 118
Endocrinology 121
Dyslipidaemia (2020) 121
Diabetes (2020) 124
Obesity (2020 x 2) 129
Hyperthyroidism (2020) 132
Dermatology 137
Drug reaction (2020) 137
Infectious Diseases 139
HIV 139
Travel Preparation (2020) 142
Miscellaneous 144
Needle Stick Injury (2020) 144
Appendix 146
MET Call 146
Managing SOB in lung cancer patients 147

3
Cardiology
Infective Endocarditis (2020)
A 74 year old man presents to the hospital because a blood culture taken by his GP 24 hours prior has grown a gram positive coccus, likely
Staphylococcus, in 2/2 bottles. This patient has a mechanical aortic valve and is on warfarin. He has been unwell for a few days and saw his GP with
fever, malaise and rigors. The GP diagnosed an URTI and prescribed Amoxycillin/clavulanic acid (Augmentin). How would you assess and manage
him?
Introductory statement
Provisional Diagnosis: Infective endocarditis, given the two positive blood cultures on a background of a prosthetic aortic valve. I
am concerned about potential septic emboli which can have devastating effects such as a stroke or valvular dysfunction leading to
cardiac failure)
DDx:
• Include other causes of a bacteraemia e.g. sepsis from pneumonia.
Goals
• Ensure patient is clinically stable
• Perform a targeted history/examination/investigations to use Duke’s criteria to confirm the diagnosis of infective
endocarditis
• provide appropriate management with IV antibiotics and possibly surgical intervention to treat the infection and prevent
complications
ABCDE Assessment
• Ensure the patient is hemodynamically stable according to ABCDE and exclude life threatening complications e.g. sepsis,
overt heart failure
History
Symptoms
• Constitutional symptoms fever, chills, malaise, weakness, night sweats, weight loss, arthralgias, myalgias
• Cardio-resp symptoms Dyspnoea, cough, pleuritic chest pain
Complications
• Cardiac symptoms of progressive heart failure (dyspnoea, oedema), arrythmias
• Septic emboli
o Lungs (PE) dyspnoea, haematemesis
o Neurologic (paresis, seizures)
o Kidney (renal failure) haematuria and anuria
o Spleen (splenomegaly) LUQ pain
o Joints (arthritis) joint pain
• AKI (due to glomerulonephritis) haematuria or oliguria
• Sepsis signs and symptoms of shock
Risk factors
• Demographics: Male, ↑ age
• Medical:
o Previous IE, Pre-damaged (rheumatic fever or IE) or prosthetic heart valves, Congenital heart defects, need for
chronic haemodialysis, impaired immune function (e.g., HIV infection),
o Bacteraemia Infected peripheral venous catheters, surgery, dental procedures; Bacterial infections of various
organs (e.g., UTIs, spondylodiscitis)
• Lifestyle: Non-sterile venous injections (e.g., IV drug abuse)
Examination
• Vitals
• Cardiovascular examination new onset heart murmur, signs of HF
o Peripheral manifestations of IE Roth spots, Osler’s nodes, Janeway lesions, splinter haemorrhages
• Other examinations based on clinical history to assess complications of septic emboli e.g. neurological exam
Investigations
Diagnostic
• Use Modified Duke’s criteria (BE FEVER) see appendix
• TOE/TTE visualise a vegetation, abscess or new valvular regurgitation
o NB: TOE needed to exclude the infective endocarditis, not the TTE
• Blood cultures (3 from 3 different sites) identify causative organism
o NB: ideally blood cultures before antibiotics so the bug can be determined
Bedside
• Urinalysis and urine MCS glomerulonephritis (leucocytes, proteinuria, red cell casts)
• ECG conductive system damage (AV block, BB block, or complete block) also helps to differentiate MI and PE
4
Laboratory
• FBC Leucocytosis
• Inflammatory markers (CRP) Raised
• EUCs glomerulonephritis or septic emboli
Imaging
• CXR assess for APO in HF/rule out respiratory differentials
Management
• Infective endocarditis, especially in a patient with a prosthetic valve requires input from an MDT including cardiology,
infectious diseases and cardiothoracic’s
o At POW, any staph infection needs to involve ID regarding antibiotic stewardship
Supportive
• Cease Augmentin
• ± Fluids if needed
• ± Anti-pyrectics
Definitive
• Empirical antibiotics
o Depends on cause i.e. IVDU vs native vs prosthetic valve – all IV
Modify after susceptibilities are known
Prosthetic valve IV flucloxacillin + vancomycin + gentamicin
• Staph most common cause for this group!
Native valve IV flucloxacillin + benzyl penicillin + gentamicin
Depending on risk of MRSA
• IV flucloxacillin for MSSA
• IV vancomycin for MRSA
• ± Anticoagulation if embolic event
• ± Surgical Management
o Indications
The infection has spread beyond the valve to contiguous cardiac tissue
Major valvular dysfunction
Persistent bacteraemia despite antibiotic therapy

Appendix: Duke’s criteria


• Clinically Modified Dukes Criteria: Need 2 major criteria OR 1 major criteria + 3 minor criteria OR 5 minor criteria (BE
FEVER)
• Major Criteria:
o Blood culture with either:
positive with typical microorganisms (listed in causative organism section) on 2 cultures
More commonly skin contaminants – three or a majority of >4 separate blood cultures with first and
last at least 1 hour apart
Single positive culture for Coxiella burnetti (infected animal Q fever ± compromised patients
endocarditis)
o Endocardial involvement on echo or auscultation e.g. mass, abscess, prosthetic valve dehiscence, new valvular
regurgitation
• Minor criteria:
o Fever > 38˚C
o Evidence from immunologic phenomena e.g. peripheral stigmata, Rheumatoid factor or glomerulonephritis
o Vascular findings e.g. emboli, Janeway lesions, major embolic infarct, intracranial haemorrhage
o Evidence from microbiology but not fulfilling major criteria i.e. atypicals
o Risk factors e.g. IVDU, RHD, congenital heart disease

5
Atrial Fibrillation (AF)
A 54 year old man complains of intermittent palpitations. He has no history of cardiac disease or hypertension. The ECG shows atrial fibrillation.
How would you manage him?
OR
A 57 year old woman is diagnosed with intermittent atrial fibrillation on Holter monitoring after she presented complaining of palpitations. How
would you assess and manage her risk of stroke complicating AF?
Note: 2nd case has an emphasis on anticoagulation considerations
Introductory Statement
Provisional Diagnosis: AF
Goals
• Assess if they are haemodynamically stable or unstable and resuscitate with electrical cardioversion
• Targeted H/E/I to determine aetiology of AF
• Appropriately manage reversible causes, consider rate/rhythm control and consider anticoagulation to minimise the
patient’s stroke risk in light of the CHA 2DS2-VaSc and HAS-BLED scores
ABCDE Assessment
• If unstable (heart failure, ischaemia, syncope, shock) perform a primary survey and simultaneously perform electrical
cardioversion to convert them to sinus rhythm (accepting risk of thromboembolic complications)
History
Symptoms ± Complications
• Most affected individuals are asymptomatic.
• Less commonly symptoms of arrhythmias such as palpitations, dizziness, syncope, fatigue, and or dyspnoea
• Onset and duration new onset (<48 hours) paroxysmal (self-terminating <48h, up to 7d), persistent (>7d), long-
standing (>1 years)
Complications
• LH failure pulmonary oedema
• Thromboembolic events (stroke/TIA, renal infarct, splenic infarct, intestinal ischaemia)
• Life-threatening VT
Risk Factors/Causes
• Demographics: age (20% of 80y/o will have AF at some point), fam hx
• Medical and lifestyle
o Chronic medical conditions obesity/OSA, any CVD (CAD, pericarditis, cardiomyopathies, CHF), any lung
disease, HT, DM, CKD, anaemia
o Acute medical conditions/other thyrotoxicosis, sepsis, electrolyte abnormalities, EtOH, caffeine,
drugs/medications
Other
• Assess risk of stroke as per CHA 2DS2-VaSc (≥1 is low-moderate risk)
o CHF, HTN, Age (≥65, ≥75), DM, Stroke/TIA/thomboembolus, Vascular disease (MI, PAD etc.), Female
• Assess for bleeding risk HA 2S-BLED2
o HTN, abnormal renal and liver function (1 point each), stroke, bleeding, labile INRs, elderly (>65), drugs/alcohol
(1 point each)
o High risk is score of ≥3 → caution and regular review of patient
potentially correctable bleeding RFs
• Medications
o Anti-thrombotics
o Compliance with medications (important if considering warfarin as AC)
• Falls history ↑ risk of complication from bleeding
Examination
• Vitals: irregularly irregular HR, temp (sepsis)
• Cardiovascular: AF
o Signs of valvular AF (important to determine whether valvular AF)
Mitral stenosis = opening snap, followed by a diastolic decrescendo murmur;
Mechanical valve = mechanical click
o Clinical features of conditions contributing to CHADSVASC – CCF, peripheral vascular disease, focal neurological
deficits suggesting stroke etc.
o Cause of AF (acute and chronic conditions) – anaemia, acute coronary syndrome, acute infection, pulmonary
disease, hyperthyroidism (lid lag, lid retraction, goitre, tremor, proximal myopathy, brisk reflexes), alcohol abuse
o Effects of AF on the cardiovascular system – acute heart failure (e.g. raised JVP)
Investigations
6
Bedside
• ECG (absence of p wave indicating AF, rate, other cardiac pathology e.g. q waves), BSL
Laboratory
• FBC (infection, anaemia prior to starting anticoagulation)
• EUCs/eGFR (renal function for management, electrolytes can cause AF), CMP (hypomag can cause AF)
• Coags (prior to starting anticoagulation)
• ± Trop if clinical evidence of ACS,
• TFTs (if expecting that as cause)
Imaging
• Echo (TTE)
o Valvular vs non-valvular will affect your choice of anticoagulant
Valvular AF defined as mitral stenosis, artificial heart valve, repaired mitral valve (causes low flow in the
LA; NOTE: MR, AS, AR etc are not considered valvular AF)
o Left atrial dilatation mitral regurgitation and mitral stenosis can cause LA → dilation can cause AF. Suggests
that the AF will be permanent rather than paroxysmal
o Left ventricular size/function e.g. ejection fraction may be impacted by CHF
Management
1) Correcting reversible causes and/or treatable conditions
• heart disease (CAD, MI, HTN, CHF, VHD), pericarditis and pericardial trauma, pulmonary disease (including PE), systemic
illness (e.g. sepsis, malignancy), stress, alcohol, hyperthyroidism, sick sinus syndrome, electrolytes (magnesium and
potassium)
• address contributing cardiovascular risk factors

2) Controlling heart rate and/or rhythm


• Unstable AF: emergent electrical cardioversion
• Stable AF: rate or rhythm control as below
Rate control Rhythm control
Goal and Normalizing the ventricular heart rate Terminating atrial fibrillation and
rationale restoring it to sinus rhythm in order to
prevent atrial remodelling
Indications Elderly patients >80 Symptomatic + <80
Comorbid conditions, irreversible trigger, failed rhythm Younger patients (prevents remodelling)
control Failure of rate-control strategy to
control symptoms
Contraindications AF due to pre-excitation syndromes Long-standing persistent AF
(e.g. WPW which may ↑ conduction down aberrant pathway) (Atrial modelling occurs within 6 months
of the onset of AF and any advantage
that would have been gained by the use
of the rhythm control strategy is lost)
1st line 1st choice: cardioselective beta 1st choice: Electrical cardioversion
blockers (metoprolol/atenolol) OR nondihydropyridine CCB (DCCV) check with TOE first for
(diltiazem, verapamil) thrombus (especially if >48h)
2nd choice: digoxin 2nd choice: pharmacologic
3rd choice: dronedarone, amiodarone cardioversion with antiarrhythmic
drugs such as flecainide, sotalol,
amiodarone
2nd line (ablative irreversible AV nodal ablation and implantation of a Catheter-based radiofrequency
procedures) permanent ventricular pacemaker (inserted 4-6 weeks prior to ablation of atrial tissue
ablation) around pulmonary vein openings
(pulmonary vein isolation)
usually in paroxysmal AF
With the exception of nebivolol, all cardioselective beta blockers begin with the letters A to M (B1 = first half of the alphabet). Except for beta blockers with alpha-blocking
action, all noncardioselective beta blockers begin with the letters N to Z (B2 = second half of the alphabet).

3) Anticoagulation
• Acute anticoagulation Before/After Cardioversion
o The sudden restoration of effective atrial contraction following cardioversion may cause a pre-existing
thrombus to dislodge, resulting in a thromboembolic event
o Anticoagulation therapy should be considered in all patients who are about to undergo cardioversion.
o If AF present for >48h (or unknown time), the risk of embolization during cardioversion is significant (2-5%), so
anticoagulate patients for 3 weeks before and 4 weeks after cardioversion (3 weeks before to dissolve clot; 4
weeks after to prevent clot when the atrium is stunned)
7
• Long term Anticoagulation
o discuss benefits/risks with pt in light of CHA 2DS2-VaSc and HAS-BLED, rationale of AC, what their stroke risk is
o NOTE: antiplatelets (aspirin or prostaglandin inhibitors) do NOT prevent the risk of stroke in patients with AF
o Nonvalvular AF
Consider HAS-BLED for risk of bleeding high risk if HASBLED ≥ 3
Consider CHA2DS2-VaSc
• If = 0 no AC
• If =1 patient choice
• If >1 AC
Choice of AC
• NOACs (apixaban, rivaroxaban, dabigatran) if no contraindications (renal failure (CrCl <25),
mechanical valves)
• If above CI Warfarin (target INR 2-3)
o Valvular AF
Anticoagulation with warfarin ± bridging therapy (not always required if no clot present) regardless of
CHADSVASC
• The AMH recommends 5 mg once daily for 2 days, then adjusting according to the INR.
• NOACs have been shown to ↑ mortality in these patients
o NB: no anti-platelets

8
Difficult discharge post syncopal episode (2020)
An 85-year-old non-English speaking female has been in hospital for 3 days under your team following a syncopal episode, for which no cause was
found despite routine blood tests, serial ECGs, and a CT brain. You are told on the ward round to discharge her, but her children insist she stay in
hospital until a cause is found. How would you manage this situation?
Introductory Statement
Goals
1. Discuss with the patient and her children to understand their concerns utilising a translator to ensure the patient is
informed in the discussions.
2. Reassure that adequate testing has been conducted and explain the result if appropriate.
3. Ensure to safety net & also arrange for GP follow-up
History
• Ensure the patient and children understand and utilise an interpreter if necessary
• Screen for any current symptoms to confirm there are no new symptoms
Examination
• Ensure vitals stable, otherwise no further investigations necessary
Investigations
• Review investigations and confirm exclusion of any sinister diagnoses that could have caused the syncope including
geriatric syndromes
Management
• Counselling station: consider environment + translator + capacity; avoid medical jargon + use visual aids
Counselling
• Elicit specific concerns of family re safety, financial issues, etc
o FRAP = fear & anxiety, religious/cultural beliefs, anecdotal evidence, past experiences
• Explain to the patient and family the process that was undertaken at the hospital and how the life-threatening causes
were excluded. Reassure that all necessary investigations have been done and sometimes a cause may not be found.
• Inform that syncope (of no sinister cause) is common in the elderly population
• Emphasise the risks of prolonged hospitalisation (infection, falls, delirium)
• Use an MDT approach social worker for family dynamics
• Important to explain that care is not being stopped and that the following measures will be in place to assist:
o Safety net to ensure representation to ED if required
o GP/specialist follow-up
o Evaluation and management of co-morbidities and medications prior to discharge or on GP follow-up
o ± medical alert system e.g. apple watch falls detection, medical alert bracelet
o ± ComPacks package (maximum 6 weeks) for safe return from hospital to home
Depends on PSHx with patient independence
o ± Allied health (OT, physio)
o ± ACAT assessment for home care packages, short-term care option or aged care homes
• Escalate to senior staff if persistent refusal

9
Heart failure (2020)
An independent 72yo male presents with dyspnoea on exertion at 200m on flat surface, orthopnoea and lower limb swelling. He has no angina. He
had coronary artery bypass surgery 17yrs prior. His only medications are aspirin and rosuvastatin. How would you manage him?
Major causes
• General causes
o 3 major causes are CAD, HTN, DM
o Valvular heart disease, renal disease, infiltrative diseases (e.g. haemochromatosis, amyloidosis)
• Specific causes
o Systolic dysfunction (HFrEF)
Cardiac arrhythmias, dilated cardiomyopathy, myocarditis
o Diastolic dysfunction (HFpEF)
Constrictive pericarditis, restrictive or hypertrophic cardiomyopathy, pericardial tamponade
Introductory Statement
Provisional Diagnosis: My impression is that this man has a decompensation of congestive cardiac failure which is likely due to an
ischaemic cardiomyopathy secondary to IHD.
DDx: My differentials would include
• other cardiac causes (new onset ACS, valvular disease – MR, AS; arrhythmias – 2/3rd degree heart block, AF; cardiac
tamponade)
• respiratory causes (PE, pulmonary hypertension, interstitial lung disease, COPD, asthma).
• renal failure
Goals
• Ensure he is haemodynamically stable
• Focussed history, examination and investigation to confirm the diagnosis and grade the severity
• Manage with pharmacological and non-pharmacological strategies
History
Symptoms ± Complications
• Time frame for symptoms
• Heart failure dyspnoea/fatigue/exercise intolerance, orthopnoea/PND, unintentional weight loss (cachexia), volume
overload, hypotension/inadequate perfusion (e.g. mental status changes)
o Quantify exercise tolerance!
o Mood screen (not uncommon to get depression in HF)
• Causes of exacerbation cardiac causes (see differentials), diet changes/medication non-compliance, medications,
infection, anaemia/Fe deficiency, hyperthyroidism
• Determine severity based on NYHA stages
o 1 cardiac symptoms but asymptomatic and no limitation in physical activity
o 2 mild symptoms and slight limitation during ordinary activity
o 3 significant limitation in activity due to symptoms. Comfortable only at rest
o 4 severe limitations. Symptoms present at rest.
Risk Factors
• RF for CVD/HF
o History of CVD/HF Previous vs initial diagnosis? Cardiac history?
o Cardiac risk factors modifiable and non-modifiable
Differentials
• ACS chest pain, quick onset
• Valvular disease AS (syncope, chest pain, exertional dyspnoea), MR (cough, dyspnoea, non-specific)
• Heart block palpitations, dyspnoea, pre/syncope
• Pulmonary hypertension very similar presentation to CCF
• Other resp
Other
• Psychosocial Functional impact of disease! This is important in terms of management and holistic healthcare
Examination
• General/vitals narrowed pulse pressure, cachexia, fluid overload
• Cardiorespiratory
o Common features S3/S4 gallop, pulsus alternans
An S3 gallop indicates rapid ventricular filling (dilated cardiomyopathy), while an S4 gallop indicates
ventricular hypertrophy (reduced compliance).
o Left heart failure pulmonary oedema (bibasal creps), orthopnoea, displaced apex beat, cool extremities,
sweating

10
o Right heart failure peripheral pitting oedema, ↑ JVP, hepatojugular reflex, hepatic venous congestion
(hepatosplenomegaly, abdominal pain, jaundice, ascites)
o In clinical practice, biventricular heart failure with features of left and right heart failure is more likely than
isolated failure of one ventricle!
o Other valvulopathies
Investigations
Bedside
• ECG (new onset ACS, old ACS, heart block)
Laboratory
• FBC ± Fe studies (anaemia can exacerbate HF, HF patients are at increased risk of developing Fe deficiency), EUC/eGFR
(hypoNa, worsening renal function in HF, baseline before start medications), LFT (hypoalbuminemia, congestive
hepatopathy (deranged lfts)). BNP / NT-pro BNO (high levels in patients with classic symptoms can confirm the diagnosis)
• Others to consider troponin (if suspicious of ACS), TFT (hyperthyroidism can exacerbate HF), lipids/BSL/HbA1c
(optimisation of risk factors)
Imaging
• Echo
o At rest and can consider stress
o Provides non-invasive measure of haemodynamics
o Looking for segmental impairment (due to old ACS) and LVEF
Normal EF >55%
• CXR
o ABCDE in heart failure
Alveolar oedema (bat wing opacities), Kerley ‘B’ lines, cardiomegaly, diversion and dilation of upper
lobe vessels, pleural effusion
• Other specialised imaging
o Radionuclide ventriculography, cardiac MRI
Special
• Right heart catheterisation evaluate pulmonary pressures
o Certainly wouldn’t do this in the acute setting and would be the decision of the cardiologist. Used to work up
patients for more advanced therapies and evaluation of pulmonary pressures
Management
• MDT Approach!!! GP, cardiologist, dietician, exercise physiologist/physio
Acute
• Diuretics (frusemide) to get rid of fluid overload
• Rest i.e. minimal exertion
• Positioning sit upright
• Fluid restriction
• ± CPAP if severe APO

Long term
Non-pharmacological
• Patient education
• Na restriction (≤3g/day) and fluid restriction with daily weights mane after voiding and before breakfast
o only fluid restrict if late stage - individual dependent 1.5L/day is good start
o This patient may not need fluid restriction in the long term
• Optimise SNAP risk factors and minimise obesity
o Stop smoking, no/minimal alcohol, weight loss and exercise when possible
• Cardiac rehabilitation
o Graded approach to exercise
o Needs to be individualised so as not to exacerbate symptoms
o Should stop exercise while in acute exacerbation
• Immunisation (pneumococcal and season flu vaccine) since infections can exacerbate CHF
• Monitor for depression and manage accordingly
• Continue his current medications + encourage compliance
• Avoid exacerbating medications
o E.g. ibuprofen and other anti-inflammatories

Pharmacological (combination pharmacotherapy is key in improving outcomes)


• Initial
o Frusemide to reduce volume overload
No mortality benefit, only symptomatic treatment
11
Goal weight loss is 1kg/day
This is very important as shouldn’t be starting any other medications until euvolaemic as they can
actually worsen mortality
Can be used PRN at any stage
o ± Nitrates if significant APO e.g. sublingual glyceryl nitrate
o ± IV morphine if significant APO
• After stabilised (BAM)
o B-blocker (mortality benefit)
Examples metoprolol XR, nividelol, bisoprolol, atenolol, carvedilol
• Avoid b-blockers with sympathomimetic effects pindolol, acebutalol
• With the exception of nebivolol, all cardioselective beta blockers begin with the letters A to M
(B1 = first half of the alphabet). Except for beta blockers with alpha-blocking action, all
noncardioselective beta blockers begin with the letters N to Z (B2 = second half of the
alphabet).
Reduces strain on heart
Can only be started when patient has no or minimal evidence of fluid retention
Start at v low dose and then titrate up until max dose (effect is dose dependent)
Monitor weight weight gain of 1/1.5kg should be noted could indicate decompensation b-
blocker should be stopped or decreased transiently to improve symptoms
o ACEI/ARB (mortality benefit)
Examples captopril, enalapril, lisinopril
Reduce afterload decreases stress on heart
ACEI first line and only use ARB if ACEI not tolerated
Avoid combo therapy has no survival benefit
Start at low dose and then up-titrate in 1-2 week intervals to maximal dose
o Mineralocorticoid receptor antagonists (spironolactone) (mortality benefit)
Use in addition to ACEI/ARB/ARNI in patients with low LVEF but can also consider starting all patients
on this
Need to monitor K and creatinine levels within one week of starting this medication
o Additional therapies (likely referring to cardiologist for use of these meds)
Angiotensin-neprilysin inhibitors (ARNI e.g. sacubitril-valsartan)
• Neprilysin inhibitor reduces degradation of natriuretic peptides.
• Use in patients who tolerate ACEI/ARB
• Debate over whether this should be initial therapy or added later
Ivabradine
• To reduce HR (use if HR≥70) in patients on max b-blocker dose or b-blockers are
contraindicated
Hydralazine + nitrate
• Hydralazine is predominantly an arterial vasodilator
• If LVEF low despite optimal treatment or if ACEI/ARB contraindicated
Digoxin
• Persistent symptoms despite optimal medical therapy
o CI medications NSAIDs (worsen renal perfusion), CCB, thiazolidinediones, moxonidine

Invasive procedures
• Implantable cardiac defibrillator (ICD): prevents sudden cardiac death
o Used in those with poor EF and ↑ risk of life-threatening cardiac arrhythmias
• Cardiac resynchronisation therapy
• Ventricular assist devices
• Cardiac transplantation
• Manage other pathology
o CAD coronary revascularisation
o Valvulopathies valvular surgery

References
• Liz (Francis notes), med group combined notes
• https://www.uptodate.com.acs.hcn.com.au/contents/overview-of-the-management-of-heart-failure-with-reduced-
ejection-fraction-in-
adults?search=heart%20failure&source=search_result&selectedTitle=2~150&usage_type=default&display_rank=2#H393
0361416

12
• https://tgldcdp.tg.org.au.acs.hcn.com.au/viewTopic?topicfile=heart-
failure&guidelineName=Cardiovascular&topicNavigation=navigateTopic#toc_d1e402
• Jared notes from ILP from American college cardiology guidelines

13
Pre-conception and hypertension counselling (2020)
A 36yo female is planning for pregnancy. Her BP in your office is repeatedly high and is 152/94mmHg today. Her BMI is 27.5. The patient does not
want to take medications as she does not feel unwell. How would you manage her?
Refer to Primary Care: hypertension for similar case
Introductory Statement
Goals My approach to this woman would be;
1. Thorough history/examination/investigation
a. To determine whether her hypertension
i. primary (essential HTN)
ii. secondary
1. (renal artery stenosis, phaeochromocytoma, Cushing’s/hypercortisolism,
hyperreninism/hyperaldosteronism, pituitary adenoma, hyperthyroidism, medications e.g.
NSAIDs, corticosteroids, OCP, sympathomimetics)
iii. whitecoat hypertension
b. Including obstetric and preconception counselling
2. Manage her hypertension, weight and preconception considerations with both pharm and non-pharm strategies
History
Symptoms ± Complications
• Hypertension
o Mostly asymptomatic
o Non-specific headaches esp. early morning, dizziness/tinnitus/blurred vision, flushed appearance, epistaxis,
chest discomfort/palpitations, nervousness, fatigue/sleep disturbances
o Complications
End organ damage: cardiovascular disease (IHD, aortic dissection), cerebrovascular disease, peripheral
arterial disease, hypertensive retinopathy, hypertensive nephrosclerosis
Differentials
• Exclude secondary HTN
o Endocrine, renal (urinary symptoms), medication review, pregnancy status, Cushing’s features
Risk Factors
• Cardiac risk factors non-modifiable and modifiable
Other
• Obstetric/Pre-conception history
o See ‘Pre-conception Counselling’ O&G case
o Summary O&G history, medical history, medications, psychosocial (smoking, alcohol, drugs, vaccinations,
occupation/exposure risks, support systems, CST/STI last check)
Examination
• General health: vitals + anthropometrics (height, weight, BMI and waist circumference) + general appearance
(cushingoid features)
o BP
Take on 2 separate occasions with a calibrated mercury sphygmomanometer or automated device
Measure on both arms, use the arm with the higher reading
Repeat upon standing for at least 2 minutes
Ambulatory BP monitoring if
• Unusual variation between BP readings in clinic
• Suspected white coat hypertension
• HTN resistant to drug treatment
• Suspected hypotensive episodes, e.g. elderly or diabetics
Ambulatory monitoring is considered a superior predictor of outcomes than clinic BP measurements
• Systematic exam for preconception heart, lungs, abdo, breasts, thyroid, mouth, genital tract
Investigations
Bedside
• Ambulatory BP if indicated, ECG (signs of IHD in context of HTN), urinalysis (end-organ damage e.g. proteinuria), urine
albumin:creatinine (same as UA)
Laboratory
• FBC, EUC/eGFR (kidney function), Lipids (TC, HDL, LDL)
• Fasting glucose (diabetes screen)
Others
• Secondary screen
o Renal artery dopplers (renal artery stenosis)
14
o Urine catecholamines + plasma metanephrines (phaeochromocytoma)
o Renin/aldosterone levels + ratio (hyper-reninism, hyperaldosteronism)
o Morning cortisol + Dexamethasone suppression testing (Cushing’s)
1) Confirming hypercortisolism: early morning serum cortisol levels following a low-dose
dexamethasone suppression test OR 24 hour urine cortisol
• ↑ as it not suppressed by dex
2) Primary vs. Secondary: serum ACTH levels
• ↓ in primary and normal
• ↑ in secondary
o 3) Confirmed by a high dose dexamethasone suppression test
Here there will be adequate suppression to <50% baseline in Cushing’s
disease
No suppression in ectopic ACTH production
o TFT (hyperthyroidism)
Pre-conception
• Can consider doing antenatal work up here but would probably wait
• Most important things to do would be varicella and rubella serology as they have a live vaccine that needs to be given
before pregnancy (ideally ≥1month before)
Management
Non-pharmacological
• Hypertension
o SNAP
DASH diet is Na <1.5g/day
o Weight loss
• Weight
o As above
• Pre-conception
o Counselling about risks of HTN in pregnancy caesarean, pre-term birth, pre/eclampsia, low birth weight
o Importance of BP reduction before pregnancy
o Consider prescribing folic acid and iodine supplement (e.g. Elevit)

Pharmacological
• Treat any secondary causes!!
• In this patient you can probably get away with ambulatory blood pressure monitoring and non-pharmacological
interventions
• You definitely want to treat if SBP >160, goal is SBP<140
• If choose to treat pharmacologically avoid ACEI/ARB as they are teratogenic
• Options “NHL and M”
o Labetalol, methyldopa, nifedipine, hydralazine (IV)

Arrange follow up for preconception work up


• At this visit probably not enough time to discuss preconception management e.g. family planning, optimisation of
conception etc + all the other work up

References
• Hypertension GP notes, preconception counselling O&G notes
• Liz (Francis notes), med group collaborative notes
• https://www.uptodate.com.acs.hcn.com.au/contents/the-preconception-office-
visit?search=preconception%20hypertension&source=search_result&selectedTitle=2~150&usage_type=default&display
_rank=2#H21
• https://www.uptodate.com.acs.hcn.com.au/contents/use-of-antihypertensive-drugs-during-pregnancy-and-
postpartum?search=preconception%20hypertension&topicRef=448&source=see_link#H3907771801
• https://ranzcog.edu.au/RANZCOG_SITE/media/RANZCOG-MEDIA/Women%27s%20Health/SOMANZ-Hypertension-
Pregnancy-Guideline-April-2014.pdf?ext=.pdf

15
Respiratory
Recurrent DVTs (2020)
60yo M developed a DVT in his right leg 2 months after stopping warfarin for a similar episode on the left. How would you manage him?
See: Surgery DVT post-op [master case]
Introductory Statement
Provisional Diagnosis: My impression is this man has a recurrent DVT in the context of warfarin cessation
Goals
• Initial assessment to rule out life threatening PE
• Focussed hx/ex/Ix to determine aetiology behind his recurrent DVTs
• Anticoagulate him to prevent life-threatening VTEs
• Counsel him on his likely lifelong use of warfarin
Rest of case
• See: Surgery DVT post-op (therapeutic [master case]
• Particular focus in Hx:
o cause of recurrent DVT, e.g. provoked or unprovoked
o Warfarin hx
Reason for cessation scheduled or for another reason e.g. side effects, repeated testing
Compliance, screen for interactions, side effects, why warfarin and not NOAC, which brand of warfarin
Management
• ± Haematology consult
• Long term anticoagulation
o Choice = Usually [1] NOAC but warfarin should be strongly considered since he was previously on it
o Duration = most likely long-term since recurrence
• Counselling: WARFARIN
o Watch your warfarin!
Document doses and INR, use same brand, carry warfarin treatment card
Best to take warfarin at the same time each day, around 4 pm (allows for testing in morning, chasing
results and adjustment of dose)
Explain what to do if a ‘missed dose’ occurs (what to do depends on how much time has lapsed
between the scheduled dose and the forgotten dose).
o Alcohol and other drugs
CYP450 inducers (decrease effects of warfarin) alcohol (chronic), anti-epileptics (carbamazepine,
phenytoin), rifampicin
CYP450 inhibitors (increase effects of warfarin) valproate, erythromycin, isoniazid
Check with pharmacist when changing meds/OTC included
NSAIDs can interfere with warfarin clearance
o Reduce risk of bleeding
Avoid falls, electric shaver etc
o Foods
Normal balanced diet, avoid excessive vit K rich foods (e.g. green leafy vegetables). Grapefruit can
enhance metabolism making less effective. Variation INR instability
o Adverse reactions
Inform about A/E e.g. bleeding (dark bowel motions, unexplained bruising, bleeding mucosal
surfaces etc.), skin necrosis, GIT upset
o Regular INR Testing (RIN)
Once initially stabilised at least 4 weekly (often graduated program i.e. weekly, fortnightly etc)
More testing if sick, start new drugs, short course

16
VTE Prophylaxis (2020)
A 68 year old man with known lung cancer is admitted for staging investigations. How would you assess and manage his risk of venous-
thromboembolism during the admission?
Introductory Statement
Provisional Diagnosis: It is important to establish this man’s VTE risk, especially given his added risk of malignancy.
Rest of case:
See Surgery: DVT Prophylaxis pre-op

17
Pulmonary Embolism (2020)
A 36 year old woman is receiving chemotherapy for breast cancer. On the fourth day of her hospitalisation she becomes acutely short of breath. Her T
is 37.4°C, PR is 120/min, BP 100/85mmHg, RR 32/min and O2 saturation 88%. How would you assess and manage her?
Introductory statement
Provisional Diagnosis: PE given the acute onset of symptoms with a history of malignancy, a hypercoagulable state.
DDx: many of these not completely relevant to this stem as the patient has become ‘acutely’ short of breath
• Resp
o Chest wall and pleura Pneumothorax, pleural effusion or massive ascites, pleuritis, pleural tumour
o Airway disease asthma exacerbation, pneumonia, Influenza, acute bronchitis,
o Parenchymal disease ARDS, infiltrative or metastatic tumour
o Pulmonary circulation chronic thromboembolic
• Cardio
o ACS, Acute decompensated heart failure, Constrictive Pericarditis, Pericardial effusion/Tamponade
• MSK
o Costochondritis, Rib fracture or flail chest
• Psychogenic
o Anxiety or panic attack
Goals:
• Ensure she is haemodynamically stable
• Targeted history, examination and investigations to confirm diagnosis
• Provide definitive management with anticoagulation
Resuscitation
• Senior clinician --> activate rapid response/MET call (likely will need ICU/HDU given vitals are borderline)
• A: Ensure airway is patent
• B: Start 15L of oxygen via a non-rebreather mask to correct desaturation
• C: IV access and resuscitative fluids if systolic BP <90
o ECG – rule out cardiac causes, identify patterns consistent with PE
• D: GCS, PEARL
• E: assess temperature
History
Symptoms
• Symptoms of PE dyspnoea at rest or on exertion, pleuritic chest pain, non-productive cough, haemoptysis (more
common with pulmonary infarction), presyncope/syncope, feeling of apprehension, orthopnoea calf pain/swelling (DVT)
Risk factors
• Virchow’s triad as above
Others
• CI to mechanical prophylaxis and chemical anticoagulation
Examination
• General observation + vitals (massive PE presents with SBP <90mmHg for >15MIN)
• Respiratory exam tachypnoea, reduced air entry, wheeze, adventitious breath sounds
• Cardiac exam tachycardia, raised JVP
• Lower limb exam calf/thigh swelling, erythema, oedema, tenderness, palpable cords
• Differentials
o Skin wounds, infection
Investigations
Diagnostic
• PE rule-out criteria (PERC) D-dimer CTPA (if high pre-test probability) V/Q scan (if CTPA inconclusive)
• Based on Well’s criteria for PE ≤4 or >4
o ≤4 apply PERC and if any points then administer D-Dimer
o >4 CTPA
• Alternatives: TTE useful in emergency setting with suspected PE but patient haemodynamically unstable as can be
done at bedside looking for right heart strain (dilated RV and right sided hypokinesis/dysfunction), ?McConnel’s sign
(highly specific for PE)
o can see clot if proximal enough + see right heart strain)
Bedside
• ECG (Most commonly sinus tachycardia (44%) and nonspecific ST segment and T wave changes (up to 50%) which may
correspond to RV strain, but classically S1Q3T3)

18
• ABG hypoxia, resp alkalosis and hypocapnia as the hypoperfusion stimulates tachypnoea but CO2 is exchanged over
the alveolar membrane faster than O2 so CO2 is blown off, causing a respiratory alkalosis
• B-HCG (woman of child bearing age) anticoagulation considerations
Laboratory
• FBC infection, need their initial platelet count to see if HITS happens after you give heparin
• CRP, ESR infectious causes
• Coagulation for treatment
• EUC renal function for contrast, possible LMWH dose adjustment, secondary AKI
• Troponins can be raised in both PE and MI. Used prognostically in PE not diagnostically
Imaging
• CXR rule out other differentials
• Venous doppler US looking for DVT as a source (not necessary for diagnosis)
Management
Supportive measures
• Respiratory support
• Analgesia for chest pain
• Haemodynamic support (fluids)
• See Surgery: DVT post-op (therapeutic). Summary:
o Mechanical prophylaxis
o Ambulation
Definitive
• See Surgery: DVT post-op (therapeutic). Summary:
• Anticoagulation (identical for DVT and PE)
• ± Adjuncts
o ± Fibrinolysis/embolectomy
Haemodynamically unstable fibrinolysis or embolectomy considered
o ± IVC filter (greenfield filter) if anticoagulation CI, recurrent PE, etc.
• Long term
o See Surgery: DVT post-op (therapeutic)

Appendix
Well’s Criteria PE
Mnemonic Criteria Points
Don’t Clinical suspected DVT (signs and symptoms) 3 points
Die PE most likely diagnosis 3 points
Tell Tachycardia (>100bpm) 1.5 points
The team Three days of immobilization or surgery in previous four weeks 1.5 points
To Past history of thromboembolism (DVT or PE) 1.5 points
Calculate Coughing up blood (i.e. haemoptysis) 1 point
Criteria Cancer: Active malignancy (treatment within 6 months) or palliative 1 point

S1Q3T3 change
• deep S wave in lead I, Q wave in III, inverted T wave in III. This “classic” finding is neither sensitive nor specific for
pulmonary embolism; found in only 20% of patients with PE.

19
Post-MI complication
A 59 year old man is admitted following a myocardial infarction requiring coronary stenting. On the 4th day of hospitalisation he becomes acutely
dyspnoeic. How would you assess and manage him?
Introductory Statement
Provisional Diagnosis: My impression is the patient’s dyspnoea is likely a result of a complication following his myocardial
infarction
DDx:
• Cardiovascular
o ischaemic – reinfarction (possibly due to stent thrombosis), infarct extension
o arrhythmic – atrial or ventricular arrhythmia
o mechanical – rupture ± cardiogenic shock
• Respiratory
o PE, pneumothorax, pneumonia, atelectasis
Goals: My priorities are:
• A to E assessment to confirm the patient is HD stable, providing emergency management as needed
• H/E/I to confirm the cause of his acute dyspnoea
• Provide definitive management to treat the underlying cause
Rapid Assessment
• A: check airway is patent and protected
• B: check breathing and oxygen saturation, provide supplemental O2 if hypoxic (<93%)
• C: check blood pressure + serial 12-lead ECG, looking for new ischaemic changes and arrhythmias, fluid resuscitation if
hypotensive
o Check bloods
ABG – if resp cause
Serial troponins (+CK-MB) trops may still be elevated 7-10 days, CK-MB less specific but falls within
48-72 hours which is more useful for re-infraction
FBC + CRP
± D-dimer
• D: GCS
• E: check temperature
History
• Hypothesis driven with emphasis on ruling out red flags
Symptoms ± Complications
• Identifying cause
o ACS – central crushing chest pain, radiating down L arm and to jaw, SOB, N+V
o CCF – dyspnoea, PND, orthopnoea
o PE – pleuritic chest pain, cough, haemoptysis, calf pain/swelling
o Pneumonia – productive cough, fever
Risk Factors/Causes
• Review notes for initial presentation, treatment received and progress in hospital
• PE - VTE prophylaxis and whether this has been adhered too
Examination
• General obs + vitals
• Cardiovascular exam Beck’s triad for tamponade (hypotension, raised JVP, muffled heart sounds), new onset systolic
murmur (MR, VSD)
• Respiratory exam listen to lungs (crepitations suggesting infection or APO), look for pleural effusion suggesting
infection (decreased chest expansion, stony dull), look for a pneumothorax (hyper-resonant, decreased air entry)
• Lower limbs – look for DVT
Investigations
• See resus continue ECG monitoring
Imaging
• Echo – identify cardiac rupture, signs of heart failure, pericardial effusion (pericarditis) infarcted area
• CXR – respiratory differentials, increased cardiac shadow (CCF, pericarditis)
• CTPA if high risk for PE
Management
Supportive
• oxygen, analgesia, position

20
Definitive
• Cardiovascular
o Re-infarction ACS pathway - GTN, DAPT, anticoagulation, beta blocker thrombolysis/PCI
o Tamponade drain the pericardial fluid (percutaneous or surgical), surgical repair of ruptured wall
o VSD + MR surgical repair
o CHF POND – nitrates, commence diuretics ± ACE inhibitor + beta blocker
o Acute pericarditis aspirin (usually already on)
• Respiratory
o PE anticoagulation
o Pneumothorax exclude tension pneumothorax, thoracostomy
o Atelectasis chest physio, ventilation
o Pneumonia empirical antibiotics

Long-term
• Cardiac rehabilitation
• Modification of risk factors e.g. smoking cessation, optimising blood pressure control etc.

Appendix: Complications post-MI


Timing Complication
0-24h • Arrhythmias (VT,VF, AF, heart block)
• Heart failure ± cardiogenic shock (if >40% of myocardium affected)
1-3d • Acute fibrinous pericarditis
3-14d • Cardiac rupture usually at 3-7 days when maximal remodelling
o Free wall rupture (usual LV) cardiac tamponade due to strangulation of RV/RA (low pressure
systems)
o IV septum L-R shunt
o papillary muscle (almost always mitral) ± valve dysfunction MR LVF cardiogenic shock
• DVT/PE
>14d • Arrhythmia’s
• CHF
• Reinfarction
• Dressler’s syndrome
• Atrial and ventricular aneurysms (rupture, arrhythmia, mural thrombus)
• Mural thrombus

21
Obstructive Sleep Apnoea (2020)
A 45 year old man presents with daytime sleepiness and heavy snoring. How would you assess and manage him?
Introductory statement
Provisional Diagnosis: My impression is that this gentleman has obstructive sleep apnoea which results in apnoeas and
hypoapnoeas and excessive daytime sleepiness, often associated with a background of metabolic syndrome
Goals
• History and exam to assess severity of complications of OSA including HTN, arrhythmias, CVD and CVAs and motor
vehicle accident.
• Confirm diagnosis of OSA with polysomnography
• Treat appropriately by advising good sleep habits + CPAP
DDx:
• Other sleep disordered breathing – OSA co-existing with chronic lung disorders, central sleep apnoea, obesity
hypoventilation
o OSA – Most common form of sleep disordered breathing. Has apnoeas and hypopnoeas during sleep due to
upper airway obstruction
o Central sleep apnoea occurs when there is either brief periodic or prolonged loss of respiratory drive during
sleep due to impaired function of respiratory centre in brain (no airway obstruction). Common in HF, regular
opioid users, premature infants
o Obesity hypoventilation syndrome – obesity reduces inspiratory muscle strength and restricts respiratory
excursions. ↓ PaO2 and increased PaCO2 during sleep (PaCO2 retention extends to the waking hours) obesity
(BMI > 35) with no other identifiable cause of hypoventilation
• Poor sleep hygiene
• Endocrine – hypothyroidism (causes tiredness)
• Substance abuse – alcohol, BZD
• Psychiatric – Stress, depression
• Narcolepsy – patient is old, usually not snoring. Disorder of sleep wake regulation rather than a problem with sleep
History
Symptoms
• including collateral history from partner
• Night-time symptoms
o snoring (frequency, loudness, whether the partner needs to leave the bedroom), waking through the night with
a sensation of choking, nocturnal interruption to breathing recognised by the partner (witnessed apnoea),
nocturnal sweating, restless sleep
• Consequences of disordered sleeping
o daytime sleepiness and irritability, impaired cognitive function (poor concentration/attention, memory loss),
morning headache, ↓ libido
o Epworth Sleepiness Scale – indicates severity of daytime sleepiness (but subjective). May also need to be filled
out by the partner to be accurate. Asks about the situations in which the patient feels tired
• STOP-BANG questionnaire screening tool and used for referral for a sleep study
o snoring, tiredness, observed apnoeas, pressure (BP), BMI, age, neck circumference >40cm, gender male. High
risk OSA 5+
Complications
• Metabolic syndrome
o Hypertension (due to ↑ sympathetic activation)
o CVD arrhythmias e.g. AF (↑ Hypoxic pulmonary vasoconstriction → ↑ pulmonary hypertension → cor
pulmonale)
o Cerebrovascular events
• Motor vehicle accidents - especially single-vehicle accidents at night
• (if female) pregnancy-related hypertension and pre-eclampsia.
Differentials
• Substance use - alcohol, benzodiazepines
• Mental health screen
• Hypothyroidism symptoms
Risk Factors
• For OSA
o Lifestyle Obesity, alcohol consumption before sleep, intake of sedatives and/or B-blockers before sleep,
smoking
o Demographics FMHx
o Medical related acromegaly, hypothyroidism, macroglossia, retrognathia, general crowding
Others
22
• Risk assessment
o Job / career – potential for impaired driving. Need to stop driving and operating machinery until condition is
adequately stabilised on treatment.
o Consider other RF to predict driving risk – acute sleep deprivation, circadian misalignment (e.g. shift work, jet
lag), narcolepsy, alcohol, comorbidities
Examination
• Vitals (BP)
• Metabolic screen weight, BMI and waist circumference
• ENT examination
o Structural abnormalities that impair respiratory flow oropharyngeal crowding adenotonsillar hyperplasia
(especially in children), nasal septum deviation, previous upper airway surgery, enlarged uvula, tongue, or soft
palate (especially in adults), overbite with a small chin, short thick neck, hypertrophied pharyngeal muscles,
nasal polyps
Nasal examination polyps, allergic rhinitis, mouth breather (ask)
o Signs of Down’s syndrome, acromegaly (shoe size changed)
• Thyroid exam - hypothyroidism
• Cardiorespiratory exam – looking for AF, signs of pulmonary HTN RHF (clubbing, right ventricular heave, raised JVP,
lower limb oedema)
Examples
• In OSA typically overweight, middle aged male with a short thick neck and history of alcohol and smoking.
• Obesity hypoventilation syndrome – typically marked obesity, cyanosis or plethora, RHF or biventricular HF
Investigations
Diagnostic
• Polysomnography (diagnostic component + titrating component, can be done on same night if need be)
o Indications can use STOP-BANG questionnaire
Urgent assessment if unstable cardiovascular status (e.g. angina), hypercapnic respiratory failure, high
pre-test probability and upcoming major surgery, significant drowsiness when driving
o Overnight in laboratory polysomnogram.
o Monitors body position, sleep stage, apnoea/hypopnea index (AHI) and level of desaturation. Want to grade
severity.
Look at mild /moderate/severe Severe –≥20-30 AHI (events per hour)
o Categorise
Apnoea vs. hypopnoea apnoea = 90% ↓ in oronasal thermistor amplitude for 10s while hypopnea is
30%
Obstructive vs. central vs. mixed obstructive = ↑ respiratory effort while central is complete
absence
• Alt. - home based tests (high specificity and low sensitivity, therefore only use if high pre-test probability)
o ambulatory screening method based on the use of a device for monitoring cardiorespiratory parameters during
the night.
Laboratory
• Metabolic profile (BGL, lipids), TFT
Others
• Rule out complications – echo (heart), ambulatory 24h BP
Management
• I would review this patient with a sleep physician. The mainstay of treatment for OSA is CPAP, but both lifestyle
measures should also be implemented
Supportive
• Ensure safety
o consider restriction of driving, especially if industrial driving
• SNAP risk factors smoking cessation can reduce nasal resistance
• Sleep hygiene avoid alcohol/drugs that affect sleep, increase time in bed
• Positional therapy (consider sleeping on side if supine OSA e.g. foam blocks or tennis balls in back of shirt)
• Weight loss lifestyle, medical or surgical (see weight loss case)
• ± modafinil/armodafinil For persistent sleepiness

Definitive
• CPAP (splints the upper airway open; need to titrate during second polysomnogram, the CPAP determination study).
o Blows air in and pushes tongue forward as air moves behind which relieves obstruction
o Consider for severe OSA or OSA with daytime dysfunction or complications e.g. HTN/CV

23
o Common problems with long term CPAP – nasal symptoms due to dry air of machine (treat with intranasal
corticosteroid spray, pressure reduction or add on humidifiers), mouth leak, dry mouth (usually indicates
significant mouth leak, try chin straps), skin ulceration over nasal bridge (refit mask), ear discomfort (use
intranasal corticosteroid spray)
o Ensure mask is comfortable, humidification is operational, no anatomic obstruction requiring surgical
intervention
o Can rent early and see if clinical improvement in the case that sleep study can’t be done immediately
• Oral appliances (e.g. Mandibular advancement splints, tongue retaining devices) – only effective for mild to moderate
OSA. Requires long term dental review
o Either design holds the soft tissues of the oropharynx away from the posterior pharyngeal wall, thereby
maintaining upper airway patency.
• Surgical usually last line
o Nasal, upper pharyngeal, lower pharyngeal, global airway procedures
o Uvulopalatopharyngoplasty is the most common surgical procedure for OSA.

Long-term
• Identify and treat underlying respiratory disorders (e.g. COPD, asthma) – needs to be treated appropriately in order to
achieve best outcome for OSA
• Monitoring for adherence and must notify RMS if driving for work

24
COPD Exacerbation (2020)
A 60 year old man is admitted to hospital with an acute exacerbation with COPD. Within hours of his admission, the nursing staff report that he has
become drowsy and difficult to rouse. How would you assess and manage him?
Introductory Statement
Provisional Diagnosis: This patient has an altered mental state, most likely in the context of O2 therapy removing his hypoxic drive
and causing CO2 retention. I would want to rule out any life-threatening issues like an MI or PE
DDx: Other differentials for altered mental state
• Delirium sepsis/infection, electrolyte abnormalities, neurological drugs and toxins (withdrawal), hypoxia (PE, MI),
uraemia (kidney failure), glucose, sleep deprivation, constipation, urinary retention, pain
Goals
• ABCDE assessment and resuscitation of the patient with the mainstay being reducing the oxygen therapy
• Focussed H/E/I to work out cause and ensure it won’t happen moving forward
Resuscitation/Rapid Assessment
• Airway (as usual)
o If ↓ GCS may not be able to maintain airway
• Breathing
o Assess breathing vitals, look, feel, listen
o Oxygen therapy if oxygen therapy has already been administered, most likely is causing worsening symptoms
Consider cessation / stepping down of oxygen therapy entirely or at least a reduction based on
saturations
• Step down delivery method from venturi Hudson nasal prongs etc.
Aim for 88-92% useful to venturi mask
o Provide first line therapies for COPD exacerbations
SABA and SAMA (most likely nebulised)
± systemic corticosteroids e.g. PO short course prednisolone 30-50mg OR IV hydrocortisone
routinely for severe exacerbations
± non-invasive ventilation (BiPaP)
• Circulation (as usual) +
o Assess for signs of septic shock
o ECG rule out MI, arrhythmias
o ± fluid resuscitation if required
o Bloods
BSL, ABG (type II respiratory failure), FBC, EUC (renal function), ± LFT, ± septic screen
• Disability
o Serial GCS to assess resuscitation
• Exposure (as usual)

History
• Want to characterise the lead up to the current situation, will need a collateral
Symptoms ± Complications
• Sx of acute exacerbation worsening dyspnoea, worsening cough, ↑ sputum volume/purulence
• Timeline of symptoms during the patient’s hospital admission
Differentials
• Cardiac symptoms palpitations, chest pain, ECG recordings
Examination
• Vitals
• Respiratory exam
o respiratory distress, cyanosis
o Other COPD signs (see Primary Care: COPD)
• Systems review looking for any source of infection or cause for patient’s altered mental status
Investigations
• As per ABCDE assessment
Imaging
• CXR Useful during acute exacerbation to rule out complications (pneumonia, pneumothorax)
Management
Definitive
• Resuscitation as above
• If above measures don’t work ICU referral for potential intubation
25
• Depends on cause
o COPD as above ± antibiotics (doxycycline or amoxicillin) + long-term management see Primary Care: COPD
o Sepsis eTG antibiotics
o Others
Supportive
• Communicate to the nursing staff regarding O2 sat targets + close obs

Long-term
• COPD long-term management as per Primary Care: COPD

26
Pleural Effusion (2020)
An 82 year old woman presents with shortness of breath and is found to have a moderate sized left pleural effusion. How would you assess and
manage her?
Introductory Statement
Provisional Diagnosis: This patient has a pleural effusion that can be due to either a transudative or exudative cause of which
there are many. Given the size, unilateral and age I would be particularly worried about an exudative cause – particularly
malignancy.
DDx:
• Transudative
o CHF
o Cirrhosis
o Nephrotic syndrome
o Protein losing enteropathy
o CKD (Na+ retention)
• Exudative (more one-sided)
o Infection (pneumonia, bronchitis, TB, influenza A, parasitic illness)
o Trauma (esp. thoracic surgery)
o Malignancy (lung, breast, lymphoma, mesothelioma, ovarian)
o PE
o Autoimmune (SLE, RA, vasculitis, sarcoidosis)
o Pancreatitis
o Other
Chylothorax
haemothorax
x
• exclude AMI or PE
Goals: My aims in this case would be to
• Initially determine whether she is HD unstable or in resp distress and perform emergency management with oxygen
supplementation and a chest drain as required
• Assuming HD stability, perform a focussed hx/examination/Ix to determine the underlying cause including a
thoracentesis using LIGHTs criteria to determine whether it is transudative or exudative
• Manage appropriately depending on the underlying cause
Resuscitation
• Ensure haemodynamically stable and saturating okay
History
Symptoms ± Complications
• Symptoms
o Pleural effusion SOB, pleuritic chest pain, dry non-productive cough
time frame, previous eps
o Symptoms of underlying disease
Transudative CCF, renal failure, cirrhosis etc
Exudative malignancy!, infective sx etc
• Complications
o symptoms of empyema (fever, chills, looks septic)
Differentials
• ACS, PE
Risk Factors/Causes
• PMHx of organ disease (e.g. CCF, kidneys, liver)
• RF for infection
• RF for malignancy (smoking hx, other malignancies thinking mets)
Other
• contraindication to chest drain/thoracentesis Anticoagulant medications/bleeding diathesis
Examination
• Vital signs (O2 sats and temp)
• Resp
o Signs of pleural effusion
Inspection and palpation reduced chest expansion (on left side), ↓ tactile fremitus
Auscultation reduced air entry, reduced vocal resonance
Percussion stony dull percussion
27
• Systems review and fluid status examination to look for cause
o Cardio (CCF), abdo (cirrhosis, renal failure), general (malignancy)
Investigations
Diagnostic
• CXR PA + lateral blunting/loss of costophrenic angles are key feature
o Need >250mL to show on PA, >50 for lateral
o Also might see a mass
• Pleural U/S
o More sensitive/specific than CXR, need >5mL, need it to locate fluid for thoracentesis
• Thoracentesis (can be diagnostic and therapeutic)
o Make sure to counsel patient on procedure and justification
Before (check meds + consent)
During (approach just above rib, initially syringe an amount but later can drain for therapeutic)
After (don’t breathe in e.g. exhale and hold or Valsalva, examine later for pneumothorax, follow-up
CXR, analgesia)
o Order pleural + serum LDH and protein lights criteria
An exudative pleural effusion is diagnosed if any of the following are found
• Pleural protein:serum protein >0.5
• Pleural LDH:serum LDH >0.6
• Pleural LDH >2/3 of upper limit of normal for serum LDH
o Further testing if exudative RBC (if v high indicates localised disease), WBC and differential (lymphocytes
>50% indicate malignancy, TB, chylothorax; lymphocytes >90% indicate lymphoma, TB; neutrophils indicate
pneumonia, empyema), cytology (malignancy), culture and AFB (empyema, parapneumonic effusion, TB),
glucose (decreased in malignancy, autoimmune, TB, empyema)
Bedside
• ABG (resp. distress), sputum MCS/AFB, ECG (rule out ACS)
Laboratory
• FBC/CRP (infection), EUC/CMP/LFT (underlying cause), CMP (paraneoplastic syndrome)
Imaging
• ± CT chest, ± staging investigations if malignancy ± CTPA if concerned PE, ± Echo if concerned about CCF
• ± thoracoscopy if patient not getting better
Management
• Aims are supportive and treatment of the underlying cause
Supportive
• O2 support, analgesia, physio

Definitive
• Thoracentesis (therapeutic if necessary) ± chest drain
• Transudative
o POND + salt restriction + med review if CCF
Positioning, oxygen, nitrates, diuretics
o ± albumin for liver failure/nephrotic syndrome
• Exudative
o Infective empirical IV abx
o Malignancy MDT management, consider chest drain, consider VATS (video-assisted thoracoscopy surgical)
pleurodesis
o Parapneumonic effusion and Empyema IV abx + intrapleural fibrinolytic agents (to aid drainage) + chest drain
± thorascopic debridement

28
Fever and myalgia post-coronary stenting (2020)
A 66yo male presented with an acute coronary syndrome 2 days ago and had coronary stenting performed yesterday without complication. Nursing
staff alert you that he developed a cough and myalgia overnight and how has a fever of 38.5℃. You are aware that other patients in the hospital
have had similar febrile presentations. How would you manage this situation?
Introductory Statement
Provisional Diagnosis: My impression is that this man likely has a post-procedural infection likely a viral infection including
influenza or COVID-19 given the myalgia and the presence of other patients with a similar presentation in the hospital.
DDx: My differentials would be broad and include
• other infective causes (pneumonia, UTI, line infection e.g. IVC/IDC, wound site infection e.g. infected haematoma,
pressure sore)
• non-infective causes (post MI acute fibrinous pericarditis, DVT/PE, atelectasis, medication induced).
Goals
• Don appropriate PPE and practice droplet precautions before assessing the patient
• Initial ABCDE assessment to rule out sepsis/septic shock
• History/examination/investigation to determine the cause
• Manage appropriately with definitive and supportive measures including infection control if necessary
ABCDE Assessment
• Don PPE and droplet precautions including N95 mask
• Negative pressure room etc.
• Ask NUM to help with infection control
• Consider sepsis sepsis 6 if appropriate (IV fluids, IV abx, O2 therapy, take lactate, take blood cultures, IDC)
History
Symptoms ± Complications
• Cough productive vs non-productive, colour, amount etc
• Other symptoms of viral illness coryzal symptoms, myalgias, headache, fatigue/malaise, dyspnoea
• COVID specific loss of taste/smell, abdo pain
Risk Factors
• Contact
o Patients in hospital, family/visitors
o What was diagnosis/treatment in other patients with same presentation
Differentials
• Infective try to localise infection by considering other causes of fever (in differentials)
• Non-infective DVT/PE (calf tenderness, risk factors, dyspnoea/pleuritic pain/haemoptysis), medication (medication
review)
Examination
• General observation/vitals
• Respiratory examination + inspect oropharynx
o URTI signs e.g. erythematous, swelling
o Non-specific findings in viral infection, focal consolidation (pneumonia), decreased air entry (atelectasis)
• Full systematic examination looking for infection
o Examine the wound site for an infected haematoma either radial or femoral
o Lines
o IDC bag if appropriate
o Skin + pressure sores
• Calf examination
o Looking for signs of a DVT
Investigations
Bedside
• Viral swabs (nasopharyngeal) rapid flu test (influenza A, B), respiratory viral PCR screen (influenza A/B, RSV,
parainfluenza, adenovirus, rhinovirus, human metapneumovirus) + SARSCOV2 test (COVID swab)
• Swabs for any other suspected infection
Laboratory
• FBC/CRP (infection, inflammation), consider if septic (VBG, blood cultures, EUC, LFT)
Imaging
• CXR (any signs of consolidation, atelectasis)
Management
• Cardiologist involvement since high risk patient
Supportive
29
• Infection control
o Adequate isolation
o Contact/droplet precautions
o Public health notification + isolation of close contacts if COVID-19
o If need oxygen therapy try to avoid NIV due to droplet spreading ICU for intubation
• Supportive care to consider if septic
o Consider IVF, IDC, serial monitoring to assess response to management

Definitive (can also ask how the other patients were treated)
• Influenza
o Mainly supportive + consider Tamiflu (oseltamivir) if <48hrs since symptom onset
o Consider prophylactic treatment of contacts
• COVID-19
o Supportive ± ICU admission
• Pneumonia
o Likely hospital acquired so empirical therapy
Mild-moderate = oral augmentin (preferred if tolerated) or IV ceftriaxone
Severe = IV piptaz
• DVT/PE
o Anticoagulation

Appendix
• Can focus more on post-op cause of fever or more on viral/other causes of the fever.
• The standard battery of post-op causes of fever may not be super relevant in this case (Francis thought most likely
diagnosis was atelectasis) because only minor procedure and therefore answer focussed more on viral causes

30
Haematology
Unstable Angina and Anaemia (2020)
A 66 year old man is in hospital being investigated for anaemia. He has a past history of stable angina and experiences an episode of angina only
partly relieved by glyceryl trinitrate. How would you assess and manage him?
NB: This case can be taken down the ACS pathway easily depending on examiner rather than the anaemia focus below
Introductory Statement
Provisional Diagnosis: This patient has an episode of angina (most likely unstable since not relieved by NTG) together with
anaemia on a background of ischaemic heart disease. Importantly, the anaemia can cause type II MI.
DDx:
• Causes of anaemia
o Microcytic, normocytic and macrocytic
In this age group, most concerned about GI bleeding secondary to cancer
• Other causes of chest pain
o Cardiac causes ACS, pericarditis
o Respiratory causes PE, pneumonia, pneumothorax
o GIT causes etc.
Goals: My primary aims in this case include
• Ensure HD stability
• Assess and manage the chest pain particularly worried about ACS, but consider other causes e.g. resp, gastro
• Determine aetiology and severity of anaemia and manage accordingly incl. RBC transfusion
ABCDE Assessment
• Particular focus on circulation
History
Symptoms ± Complications
• IHD Sx Chest pain, on exertion, radiation, exercise tolerance, dyspnoea, palpitations, dosage of GTN used
• Anaemia
o Decreased oxygen delivery fatigue, exertional dyspnoea, dyspnoea at rest, lethargy, confusion
o Pica (craving ice or dirt)
o Features suggesting hyperdynamic state palpitations
o Red flags for anaemia bleeding, symptoms of malignancy (fever, malaise, night sweats, weight loss, loss of
appetite)
Differentials
• Causes: look at previous hospital records
o Microcytic anaemia (Iron LAST) insufficient Hb (haem + globin) production
Haem deficiency iron deficiency anaemia (features of colorectal cancer or GI bleeding, diet with red
meat and greens, menstrual history if lady, malabsorption e.g. coeliac disease and IBD), Lead poisoning,
Anaemia of chronic disease (late phase, autoimmune conditions, CKD, CCF etc.), sideroblastic anaemia
(ineffective heme metabolism leading to iron trapping in mitochondria)
Globin deficiency thalassemia (FMHx and present since you)
o Normocytic anaemia ↓ blood volume or ↓ erythropoiesis
acute bleed (features of hypovolemia), Aplastic/bone marrow failure (e.g. haematological malignancies
including symptoms of pancytopenia), anaemia of chronic disease – early phase (e.g. CKD, connective
tissue disorders), haemolytic anaemia (e.g. G6PD deficiency)
o Macrocytic anaemia insufficient nucleus maturation
megaloblastic (B12/folate-B9 deficiency, medications e.g. cytotoxic), non-megaloblastic (alcoholism,
liver disease, hypothyroidism, myelodysplasia, multiple myeloma)
• Consider other causes of chest pain
o Resp, gastro, MSK, psych
Risk Factors
• CVD Risk factors
• Bleeding RF
o GIT recent FOBT, previous colonoscopies, RF for bowel cancer
o Liver pathology (leading to variceal and acute bleeding)
Examination
• Vitals: dyspnoea, tachy, postural hypotension
• Cardiorespiratory: heart failure
• Haematological exam:
o Chronic iron deficiency signs koilonychia, angular stomatitis
o Signs of anaemia palmar crease, conjunctival pallor, jaundice (suggests haemolysis)
31
o Hyperdynamic state bounding pulse, tachycardia, orthostatic hypotension, systolic flow murmur, wide pulse
pressure, signs of CHF
o Malignancy lymphadenopathy, hepatosplenomegaly, bony tenderness, features of thrombopenia (petechiae,
ecchymoses)
• Abdominal exam
o masses, palpable lesion, blood on DRE, chronic liver disease
Investigations
Bedside
• ECG, BSL
Laboratory
• IHD troponins, EUC, fasting lipids
• Anaemia
o FBC + differential + peripheral smear - look for MCV (determines later investigations)
Microcytic - workup for Fe deficiency (Fe studies), Hb electrophoresis
Normocytic - EUC (renal insufficiency), haemolytic screen (↑ LDH, ↑ indirect Br, ↓ haptoglobin,
reticulocytosis, spherocytes/schistocytes, direct and indirect Coombs test)
Macrocytic - serum B12, serum folate, TFT (hypothyroidism)
o Group and crossmatch if transfusion required
Imaging
• IHD consider echo, stress test, coronary angiogram, CT coronary angio dependent on troponins and ECG changes
• Anaemia consider endoscopy/colonoscopy if iron deficiency anaemia and evidence of GI bleeding/malignancy ± FOBT
Management
Anaemia
Supportive
• Transfusion of pRBC because he is symptomatic and has a b/g hx of IHD. In patient with IHD consider transfusion when
Hb <100g/L. Normally consider <70
o Consider maintaining higher Hb for patients with CAD/unstable coronary syndromes; uncontrolled,
unpredictable bleeding; impaired pulmonary function; increased O2 consumption
Definitive
• Treat underlying cause e.g. source of bleeding (avoid medications that can worsen bleeding e.g. anticoagulants etc.)
• Microcytic anaemia ± Iron Supplementation (if due to iron deficiency)
o Dietary - encourage intake of red meat, legumes and green leafy vegetables
o Supplementation (oral or IV) for 3-6 months
Oral - ferrous sulfate need 100 to 200mg elemental iron PO to treat
• Take with citrus juice (vitamin C) to enhance absorption; do not take with calcium or antacids
• S/e = bloating, N/V, constipation, diarrhoea, black stools
o Can split dose through the day to minimise or try alternative iron salt
IV (ferric carboxymaltose quick infusion, iron polymaltose slow, dextran) can be given if the
patient cannot tolerate or absorb oral iron.
o Monitoring response
Reticulocyte count will begin to increase after 1 week
Hb normalises by 10g/L per week (assuming no further blood loss)
Iron supplementation required for 4-6 months to replenish stores
• Macrocytic anaemia see Macrocytic Anaemia case below

Angina
• Ongoing monitoring of clinical status, ECG and biochemistry. If concerns of ACS ACS pathway
• Long term:
o Non-pharm: optimisation of RF
o Pharm: GTN ± long term therapy (DAPT, statin, ACEi, B-blocker, anticoagulation)

32
Macrocytic Anaemia (2020)
An 87 year old man is fatigued and pale. His FBC shows Hb 78 g/L (>130), MCV 108 (80-100), platelets 168 (>140). How would you assess and
manage him?
Introductory Statement
Provisional Diagnosis: This patient has a macrocytic anaemia.
DDx: In adults, it is most likely secondary to:
• Megaloblastic (Impaired DNA synthesis and/or repair with hypersegmented neutrophils)
o Vitamin B12 deficiency e.g. nutritional, pernicious, etc.
o Folate deficiency
o Medications inhibiting DNA synthesis e.g. phenytoin, sulfa drugs, trimethoprim, MTX, 6-mercaptopurine
some of these result in above
• Non-megaloblastic (Normal DNA synthesis without hypersegmented neutrophils)
o Liver disease (plays an important role in lipid synthesis, which is required for cell membranes, including those of
RBCs.)
o Alcohol (independent of folate/b12 dietary deficiency)
o Myelodysplastic syndrome
o Multiple myeloma
o Hypothyroidism
• Also broadly consider normocytic and microcytic anaemias
Goals: My approach to this patient would be to:
• Ensure he is HD stable, providing appropriate resuscitative measures as required
• Conduct a targeted H/E/I to identify the underlying cause of his anaemia
• Provide definitive management which may involve a blood transfusion as the patient is symptomatic
ABCDE Assessment
I would perform a primary survey if this patient was haemodynamically unstable (ABCDE) and consider transfusion with pRBC if
they were shocked
History
Symptoms ± Complications
• Anaemia Fatigue, SOB at rest/on exertion, palpitations, headache, dizziness/light-headedness, pica (craving for ice or
dirt mainly in iron deficiency)
o More severe cases -> severe lethargy, confusion, angina, MI
o Acute vs gradual onset, length of time of symptoms
Risk factors
• Previous diagnosis of anaemia
Complications
• Assess need for transfusion
o Some Patients require a higher Hb history of CAD/unstable coronary syndromes; uncontrolled, unpredictable
bleeding; impaired pulmonary function; increased O2 consumption)
Differentials
• First exclude life threatening emergent causes such as BM failure (e.g. haematological malignancy such as MM) - screen
for pancytopenia
o Symptoms of leukopenia (recurrent infections) and thrombocytopaenia (easy bruising)
o Constitutional symptoms of malignancy
o Signs of systemic spread including bone pain
o RF - FHx of haematological malignancies, previous chemo or radiotherapy
• Screen for causes of macrocytic anaemia -
o Liver disease/alcoholism - alcohol history including number of drinks, symptoms of chronic liver disease
o B12/folate deficiency - causes include diet (in particular vegans or vegetarians without supplemental B12; poor
nutritional intake), malabsorption (e.g. terminal ileum disease for B12)
Consequences of B12 deficiency - paraesthesia, numbness, decreased pain sensation, dementia
Associations with pernicious anaemia - other autoimmune disorder: thyrogastric cluster (Grave's,
Hashimoto, pernicious anaemia, DM)
o Medications - methotrexate, sulfa, chemotherapy
o Hypothyroidism - cold intolerance, weight gain, peri-orbital oedema
o Pregnancy (if young female) - pregnancy can lead to folate deficiency due to increased demand, and folate
deficiency can lead to neural tube defects in the fetus
Examination
• General observation + vital signs
• Haematological exam
33
o Anaemia – conjunctival pallor, palmar crease pallor, tachycardia, orthostatic hypotension, atrophic glossitis,
ulcers (vit B12/folate deficiency)
o Haematological malignancy – signs of anaemia, crepitations (infections), bruises, bony tenderness,
hepatosplenomegaly, signs of metastases (e.g. RUQ mass)
o Haemolytic causes – jaundice, cholelithiasis
• Neurological exam
o Fe deficiency – koilonychia
o B12 deficiency – cerebrum (confusion, delirium, dementia), cranial nerves (optic atrophy), posterior column
deficits (decreased vibration and proprioception; due to degeneration of posterior and lateral columns of spinal
cord), peripheral neuropathy (affects LL > UL)
• Abdominal exam
o signs of chronic liver disease
Investigations
Laboratory
• FBC + peripheral smear
o Hypersegmented neutrophils (folate/B12, medications)
↑ homocysteine levels both, ↑ methylmalonic acid B12 deficiency
o Absolute neutrophil count <1x109/L, plt <50x109/L, immature WBC (indicating urgent haematological
condition e.g. myelodysplasia, bone marrow failure or haematological malignancy)
o Increased reticulocytes in haemolytic anaemia
• Iron studies (co-existing Fe deficiency)
• LFT – chronic liver disease, Br for haemolysis, GGT for alcoholism
• Serum Vitamin B12/folate
• TFT - hypothyroidism
• Group + crossmatch
Invasive
• ± Bone marrow testing (rare for anaemia) and if no other causes identified
Management
Acute
• Transfusion of packed RBC
o Usually Hb under <70 but he is symptomatic
If acutely symptomatic with Hb <100 (MI, orthostatic hypotension, tachycardia, dyspnoea), consider
transfusion to relieve symptoms.
Hb <70g/L – Transfuse. Do not transfuse more than the minimum number of RBCs necessary to relieve
symptoms of anaemia or to return the patient to a safe Hb range (70-80g/L) in stable non-cardiac
patients
Active bleeding – Aim for Hb 70-100g/L
o Re-check Hb after transfusion, 1 pack should increase Hb by 10
• Haematology referral if no cause identified, pancytopenia, haemolytic anaemia, malignancy
Long-term
• Treat underlying cause
o Vitamin B12 hydroxylcobalamin 1g IM on alternate days for 2 weeks -> maintenance therapy every 2-3
months if required
o Folate 1-5mg PO daily for 1-4 months
Never give folate alone to an individual with megaloblastic anaemia, because it will mask B12
deficiency and neurological degeneration will continue (do tests first, then treat; OR if treatment is
urgently needed, treat with both folic acid + B12)
o Alcohol counselling, pharmacotherapy for abstinence
o Treat hypothyroidism, optimise liver disease
Appendix
• Pathophysiology of Macrocytosis (More Biomed stuff)
o Increased reticulocytes -> normal physiological reaction to anaemia of any cause, unless the ability for the bone
marrow to generate RBCs is blunted by other factors e.g. iron deficiency, cytokines in anaemia of chronic
disease
Seen in haemolytic anaemia
o Abnormal RBC development (megaloblastic) – due to impaired cell division due to a lack of nutrients available
for DNA synthesis
Vitamin B12/folate deficiency, medications that interfere with DNA synthesis

34
Thrombocytopaenia (2020)
A 29 year old woman presents with easy bruising and menorrhagia. The only haematological abnormality is a platelet count of 52 x 109/L (150-400).
How would you assess and manage her?
ITP
Pathophysiology
• IgG antibodies against GpIIb/IIIa on platelets → platelet-antibody complex removed from circulation by spleen → ↓
platelet count → bone marrow megakaryocytes and platelet production increase in response
Classification
• Primary ITP: idiopathic; may also be triggered by a previous viral/bacterial infection
• Secondary ITP: associated with lymphoma, leukemia (particularly CLL), SLE, HIV, HCV, drug reactions
Introductory statement
Provisional: My impression is this woman has ITP (Immune thrombocytopenia) given the isolated thrombocytopenia and the
demographics of her being a woman of child-bearing age (common for chronic ITP). Note: severe thrombocytopaenia is <50.
DDx:
• Disorders of primary haemostasis include
o ↓ production
Bone marrow failure e.g. aplastic anaemia (acquired or congenital e.g. Fanconi anaemia)
Bone marrow suppression: drugs, chemotherapy, radiation
Bone marrow infiltration: leukaemia, lymphoma, fibrosis
Nutritional deficiency: vitamin B12 and/or folate
Liver disease (↓ thrombopoietin)
Infection e.g. CMV, EBV, HCV, HIV, Parvovirus B18 etc these can cause things like ITP and aplastic
anaemia
o Platelet disorders NOT relevant as platelets should be normal
von Willebrand disease (can be genetic or acquired)
Congenital thrombocytopaenia: Bernard-Soulier syndrome, Wiskott-Aldrich syndrome, Alport
syndrome
o ↑ turnover
antiphospholipid syndrome ± SLE
Microangiopathic haemolytic anaemia – anaemia due to physical damage of RBC as they pass through
damaged small vessels (which have fibrin deposition and platelet aggregation)
• Haemolytic uraemic syndrome (HUS) more in children
• Thrombotic thrombocytopaenic purpura (TTP) more in adults
Infection, DIC and sepsis
Drug-induced: anti-platelets e.g. aspirin
Immune thrombocytopaenia (ITP)
Pre-eclampsia and HELLP syndrome
o Redistribution
Splenomegaly (sequestration of platelets in an enlarged spleen)
For other bleeding disorders see Haemostasis notes
Goals:
• My history and examination is aimed at working out the aetiology, ruling out my differentials and excluding any
complications of the bleeding.
• Treat appropriately if below threshold for treatment (<30) in conjunction with haematology (1 st line if ITP give
steroids)
History
Symptoms
• Bleeding history
o Determine primary vs secondary
Primary haemostasis (platelet) Secondary haemostasis (coagulation)
Effect of surface cuts Excessive, prolonged bleeding Normal or slightly prolonged bleeding
Onset after injury Immediate (can do a bleeding challenge) Delayed
Site of bleeding Superficial i.e. mucosal (nasal, gingival, GIT, Deep (i.e. joints, muscles, GIT, GU tract)
uterine) and skin Excessive post-traumatic bleeding
Lesions seen Petechiae, ecchymoses Hemarthroses, haematomas
o History of bleeding – new onset vs. congenital
o Normal menstrual history
o Exclude other causes of menorrhagia: fibroids, adenomyosis
Complications

35
• Bleeding symptoms of anaemia
Risk Factors
• Triggers of primary ITP signs of infection
• Secondary ITP associations lymphoma, leukemia (particularly CLL), SLE, HIV, HCV, drug reactions
Differentials
• ITP is a diagnosis of exclusion, defined as an isolated low platelet count with normal bone marrow and the absence of
other causes of thrombocytopenia
• Haematological malignancy fever, night sweats, weight loss, bony pain, symptoms of anaemia or neutropenia
• Infection/sepsis signs of infection
• DIC Thrombosis in any organ + bleeding (usually from the GIT, nose, puncture sites)
o “STOP Making Trouble!”- helps recall the aetiology of DIC. M - Malignancy, T - Transfusion, S -
Sepsis/Snakebites, T - Trauma (acute traumatic coagulopathy), O - Obstetric complications, P – Pancreatitis
• TTP (life threatening) HUS (microangiopathic haemolytic anaemia + thrombocytopenia + renal failure) + fever +
fluctuating neurological signs (ranges from mental status to hemiplegia)
o Primarily acquired and in adult individuals due to ↓ ADAMTS13 which normally cleaves vWF
• Liver disease
• SLE symptoms
• Drugs (Heparin, chemotherapy, alcohol)
Examination
• Haematology exam
o Look for bruising
o Organomegaly: should have normal spleen in ITP, if enlarged look for other causes
o Malignancy LN, bony tenderness
• Speculum and bimanual examination
Investigations
Bloods
• Repeat FBC + film (rule out malignancy)
o isolated ↓ platelet count, normal to large in size
• Coagulation studies (PT, aPTT) Normal (think ITP is platelets!), may be prolonged
• G+H (because bleeding)
• HIV/HCV serology (can trigger 2o ITP)
Imaging
• TV U/S

Important to rule out differentials


• SLE ANA (+ Anti-dsDNA Antibody, Anti-Sm.)
• TTP ↓ platelets, haemolytic screen results, smear (↑ schistocytes), urinalysis (haematuria, proteinuria)
• DIC ↑ aPTT, PT, INR ; ↓ serum fibrinogen, ↑ D-dimer
• Malignancy Bone marrow biopsy
o ITP ↑ megakaryocytes
Management
• Consult Haematology.
o Goal = safe platelet count to prevent clinically important bleeding > normalise platelet count
o Threshold for treatment <30
Definitive
• Thrombocytopaenia
o [1] glucocorticoids
high-dose e.g. PO for 4 days dexamethasone 40mg (preferred according to UpToDate) OR 1mg/kg daily
for 1-2 weeks PO prednisone
hypothesized that glucocorticoids suppress antibody production and/or stop platelet breakdown by
inhibiting phagocytosis. Treatment may be discontinued when the platelet count plateaus at >
50,000/mm3.
o Alt. [1] IVIg or IV Rho immunoglobulin (IV RhIG)
if intolerance to corticosteroids
Immunoglobulins can temporarily increase platelet counts, which is especially helpful before surgery.
o [3] Contraindications or refractory:
Thrombopoietin receptor agonists: romiplostim, eltrombopag
Rituximab (biological immunosuppressant - binds to CD20 antigen)
Platelet transfusion
Surgical

36
• [2] splenectomy
o Prevents platelet destruction by spleen
o Indication: treatment-resistant thrombocytopenia and severe bleeding
o Procedure: minimally invasive, laparoscopic operation is preferred
Follow-up
• Treatment of menorrhagia (see O&G: Menorrhagia) tranexamic acid + NSAIDs ± hormonal treatment (IUD)
o Follow-up and see if above treatment resolves menorrhagia
o If not, consider other pathology and treat appropriately
Appendix: Treatment of other disorders
TTP
• Supportive (monitoring and correction of fluids, electrolytes, acid-base, ± RBC transfusions)
• plasmapheresis ASAP (life-saving)
• glucocorticoids
• ± rituximab for severe cases
• ± platelet transfusion should only be reserved for patients who are bleeding or require an invasive procedure.

HUS
• Avoid antibiotics and antimotility agents (more likely HUS)
• Supportive (monitoring and correction of fluid status, electrolytes, acid/base, BP, RBC transfusion
• Dialysis as indicated
• Only refractory cases: plasma exchange therapy

DIC
• Most important treatment is correction of the underlying cause of DIC
• Bleeding and consumptive types
o replacement of haemostatic elements
a) platelet transfusion (to correct thrombocytopenia and platelet dysfunction)
b) fresh frozen plasma (corrects coagulation factor deficiencies)
c) cryoprecipitate (corrects fibrinogen deficiency)
d) packed RBC if haemorrhage without response to fluid therapy
o synthetic protease inhibitor
o antifibrinolytic therapy e.g. tranexamic acid
• Organ failure type: natural protease inhibitor
• Non-symptomatic type: unfractionated heparin or LMWH (non-bleeding patients only)

Malignancy
• MDT approach, as for lymphoma, leukemia case

37
Drug Induced Neutropenia (2020)
A 42 year old woman presents with fever and painful gums and on examination, she has marked gingivitis. Two weeks previously she was prescribed
cotrimoxazole (suphonamide and trimethoprim) for a UTI. Full blood count reveals a total WBC of 1.1X109/L with a total neutrophil count of
0.8X109/L. How would you manage her?
NB: Normal WCC (4-11x109) Normal Neutrophils (2-7.5x109)
Introductory Statement
Provisional Diagnosis: My impression for this patient is a drug-induced neutropenia secondary to co-trimoxazole use, resulting in
gingivitis
DDx: other potential causes of neutropenia
• Infections
o Viral infections HIV, EBV, CMV, Hepatitis, MMR
o Few Bacterial infections e.g. typhoid fever, shigella, brucellosis, tularaemia, TB
o Parasitic
• autoimmune disorders
• nutritional deficiencies e.g. B12, folate, copper
• haematological malignancies
• bone marrow failure (aplastic anaemia)
• congenital/familial/ethnic causes
Goals: My aims in this case would be:
• Initial assessment using ABCDE to ensure patient is HD stable
• Hx/Ex/Ix to determine underlying cause of neutropenia
• Manage underlying case
ABCDE Assessment
• Ensure the patient is hemodynamically stable according to ABCDE and exclude life threatening complications e.g. sepsis.
If septic this requires immediate IV abx and aggressive fluid resuscitation
History
Symptoms ± Complications
• Gingivitis
o Pain history, other associated symptoms (bleeding on tooth brushing, red and swollen gingiva, halitosis) and
infections
o Past infections/periodontal disease
• Neutropenia
o Other infective symptoms, identify other potential opportunistic infections including oral ulcers
Differentials
• Infection
o symptoms (fever, chills, cough etc.) sick contacts, travel history, previous/current infections e.g. HIV, hepatitis B,
EBV, malaria
• Bone marrow failure – easy bruising/bleeding, fatigue, hx of frequent infections
• Malignancy – weight loss, fever, night sweats, unexplained lymphadenopathy (+ past history)
• Hx of rheumatological/autoimmune disorders e.g. SLE
• Nutritional history
o B12 found in animal products such as red meat, fish, poultry, milk, and eggs.
o Folate (B9) found in leafy green vegetables, citrus fruits, dry beans, yeast, and vitamin-fortified cereals.
Risk Factors/Causes
• Current medications and dosage e.g. cotrimoxazole (usually 3 days, but can be mis-prescribed for much longer)
• FMHx of gingivitis
• Family history of neutropenia, bone marrow failure and other haematological disorders
Examination
• General observation + vitals
• Signs of infection
o Fever
o Oral exam – gingivitis, swelling, oral ulceration or dental pain
o Skin erythema, ulcerations, fissures
o Abnormal respiratory exam
• Haematological exam
o Lymphadenopathy and splenomegaly in haematological malignancies
o Ecchymosis and petechiae – bone marrow failure
Investigations
38
Bedside
• ± UA (infection)
Laboratory
• ± Gingival pus MCS difficult to obtain meaningful culture due to oral commensals. If abscess present, drainage + MCS
• FBC + peripheral blood smear
• ESR/CRP, EUC/CMP, LFT
• Septic screen – blood cultures (x2), urine MCS, sputum culture and chest X-ray (as febrile)
Differentials
• Consider (if clinically relevant)
o Vitamin b12, folate and copper
o Viral serology
o Autoimmune screening
o Bone marrow biopsy (if cause cannot be identified or suspecting malignancy, certain infections)
Management
Definitive
• Treat neutropenia
o Cease co-trimoxazole – neutrophil levels should return to normal within 2-3 weeks
o ± G-CSF – shown to reduce recovery + hospitalisation time + antibiotic use
• Treat infection
o Gingivitis
good oral hygiene practices
dentist
• removing plaque and calculus through debridement
• smoothing any irregularities on the teeth (e.g. rough edges of fillings) that allow plaque to
accumulate
± chlorhexidine 0.12% mouthwash BD for 5-10 days
o ± Febrile Neutropenia
Gram-positive bacteria being the most common cause of febrile neutropenia
Commence broad-spectrum IV antibiotics after cultures are taken
• IV tazocin or cefepime or ceftazidime
• + gentamicin + vancomycin If septic shock or requiring ICU
Supportive
• Isolation room with contact precautions
• Monitor for additional infections/deterioration

Long-term
• Re-do the neutrophil count

39
Acute Transfusion Reaction (2020)
2020 release: A 55 year old man is receiving a blood transfusion and develops rigors and vomiting. How would you manage him?
OR
A 67 year old woman with iron deficiency anaemia is admitted for a blood transfusion. Two hours after the transfusion commences, she is febrile.
How would you assess and manage her?
Introductory Statement
Provisional Diagnosis: My provisional diagnosis for this patient is an acute transfusion reaction, of which there are a number of
types that range from clinical benign to life threatening.
Goals:
• Immediate assessment with input from the transfusion service to identify the type of reaction
• Begin resuscitative measures immediately
• Closely monitor and manage this patient
DDx: Range of causes both immunological and non-immunological
Acute
• DDx of post-transfusion fever
o AHTR - ABO incompatible blood intravascular haemolysis
o Sepsis
o Febrile non-haemolytic transfusion reaction (After periods of long storage of blood products, cytokines may leak
from donor RBCs and subsequently cause a mild immunologic reaction in the recipient. In addition, preformed
recipient antibodies lead to lysis of the few remaining leukocytes within donor RBC concentrates, resulting in an
inflammatory reaction).
o Anaphylactic (Anti-IgA IgG in recipients with IgA deficiency bind to IgA on the surface of donor RBCs and trigger
mast cell degranulation)
• 2DDx of post-transfusion dyspnoea (not likely in this case without resp symptoms and presence of fever)
o Transfusion-associated circulatory overload (TACO), transfusion-related acute lung injury (TRALI), allergy
TRALI - Soluble factors (antibodies, certain lipids) in the donor blood lead to activation of the recipient's
granulocytes. Occurs particularly following transfusion of fresh frozen plasma (FFP) or platelets
TACO - fluid overload that can occur with transfusion of blood products. Occurs during or within 6
hours of transfusion.
• Other complications of transfusions
o Hyperkalaemia (K release from stored RBC)
o Hypocalcaemia due to citrate toxicity (occurs in liver patients; citrate binds Ca hypocalcaemia, treat with IV
calcium gluconate)
o Hypothermia (due to cold blood products)
o Minor allergic reactions Urticaria,
• NB: symptoms below
Delayed
• Immune Causes
o Delayed haemolytic (5-7 days after transfusion)
Occurs in patients who were previously sensitized to specific RBC antigens during transfusions,
pregnancy, or transplantations
Re-exposure to the antigens results in a rapid increase in antibodies that bind to donor RBCs and cause
extravascular hemolysis.
Presentation – fever, anaemia, mild jaundice, dark urine
Treatment – none required, antibody testing for future transfusion
o Transfusion-associated graft vs host disease (TA-GVHD)
Transfused T-lymphocytes recognise and react against “host” (recipient). Occurs 4-30d following
transfusion, most patients have severely impaired immune systems (e.g. Hodgkin lymphoma,
leukaemia)
Presentation – fever, diarrhoea, liver function abnormalities, pancytopenia
Can be prevented by giving irradiated blood products (Irradiation of blood products prevents
proliferation of T-lymphocytes, which cause TA-GVHD)
• Non-immune causes
o Iron overload due to repeated transfusions over long period of time (e.g. β-thalassemia major) → 2°
haemochromatosis. Treatment = iron chelators, phlebotomy if no longer requiring blood transfusion and not
anaemic
o Viral infection risk HBV, HCV, HIV (all rare), Other infections: Human T-lymphotropic virus (HTLV), EBV, CMV,
WNV (West Nile virus)
ABCDE Assessment
• ABCDE assessment as usual
40
Immediate management
• Stop the transfusion, save the remaining bag + tubing for potential analysis, maintain patent IV access (run normal saline)
• Check the transfusion bag and confirm the correct product was transfused to the correct patient,
• Look for gross colour changes/bubbles suggestive of bacterial contamination
• Contact the blood bank to seek advice about appropriate evaluation and management
History
Symptoms ± Complications
• Transfusion reaction history
o Confirm the rigors/vomiting are new onset + not related to the hospital admission
o Time the transfusion was initiated, symptoms began, and transfusion stopped
Differentials
• Fever and chills
o AHTR: rapid onset fever, chills, flank pain, burning pain at IV site, if in coma or GA [oozing from intravenous
sites (DIC)]
o FNHTR: fever in the absence of other systemic signs
o Sepsis: fevers, chills and hypotension
• Dyspnoea
o TRALI: sudden onset when transfused with platelets or FFP symptoms and imaging similar to ARDS including
fever, chills and respiratory distress
o TACO pulmonary oedema
o Anaphylaxis angioedema, wheezing + hypotension
o Minor allergic reaction Urticaria with no other allergic findings
Risk factors
• RF for TACO
Complications
• AKI (check urine output) and DIC (oozing line sites)
Other
• Underlying diagnosis/reason for transfusion
• Previous transfusion reactions
• Medications – especially any which have been started recently
Examination
• Vitals
• Cardiovascular and respiratory
• Skin examination (bruising/DIC, urticaria)
o Examine transfusion site – erythema, swelling, discharge and oozing
• Systems review Signs and source of infection
Investigations
Bedside
• UA (haemoglobinuria), ECG (arrythmias in hyperkalaemia), ABG
Laboratory
• Repeat ABO typing and cross matching on patient + donor sample to identify and record causative blood products
• Direct Coombs test confirm diagnosis
• ± Haemolytic screen
• Blood cultures on patient + donor sample sepsis
• DIC screen FBC (low platelets) + coagulation studies (↑ PT and APTT), d-dimer (↑)
• EUC assess renal function in massive haemolysis, hyperkalaemia
Imaging
• CXR: new infiltrates in TRALI/TACO
Management
• Consult haematology and blood bank for advice
• The key to most acute blood reactions is to stop the transfusion.
Definitive management
• AHTR aggressive rehydration (aid diuresis and manage hypotension) + diuresis to protect kidneys from free Hb
• FNHTR paracetamol if the patient feels uncomfortable, should be managed as AHTR until proven otherwise.
o Can consider running transfusion of same bag at lower rate if not too high fever and true diagnosis
• Sepsis adult sepsis kills pathway, immediate senior clinician review, septic workup + broads spectrum IV antibiotics
• Anaphylaxis IM adrenaline +/- bronchodilators +/- antihistamines
• TRALI supportive with oxygen, possible ventilation support
• TACO diuresis + oxygen

41
• Urticaria – antihistamines
Monitoring
• Monitor closely: however most severe reactions occur in the first 15mins
• ± Cross matched blood If patient still requires blood transfusion rather than O negative

Appendix
Massive Transfusion Protocol (Sydney Local Health District)
• Criteria for activation of MTP
o Actual or anticipated transfusion of 4 units of RBC in <4 hours + haemodynamic instability ± anticipated ongoing
bleeding
o Severe thoracic, abdominal, pelvic or multiple long bone trauma
o Major obstetric, gastrointestinal or surgical bleeding
• Activation - senior clinician contacting Blood Bank
• Blood product shipments - 4 units RBCs, 4 units FFP, 1 bag pooled platelets (varies depending on first/second and
subsequent shipments; may have cryoprecipitate)
Difference between group + screen & group + hold/cross-match
• Group + screen = blood group + antibody screen. Can prepare cross-matched blood in 10 min
• Group + hold is the same as group + cross-match = blood group + antibody screen + prepare 2 units of cross-matched
RBCs, held for 42 days (let them know if you don't need it, costs $300)

42
Pre-op warfarin management (2020)
An 87 year old male is admitted to hospital for elective inguinal hernia surgery. He had a metallic aortic valve replacement 20 years ago. His last
warfarin dose was 4 pm yesterday and his INR today is 3.4. How would you manage him?
INR targets
• mechanical aortic valve is 2.5 (2-3) [might be higher if other risk factors for VTE e.g. AF)
• mechanical mitral valve 3 (2.5-3.5)
Introductory Statement
Goals
1. Standard pre-operative work-up
2. Evaluate bleeding vs clotting risk in consult with cardiology & general surgery
3. Likely delay surgery until INR <1.5 with proper perioperative management of warfarin use including ceasing warfarin 5
days before surgery ± bridging therapy with LMWH
History
Surgical history
• Clarify emergency vs elective list (i.e. whether the surgery can be delayed)
o Since elective in the stem possibility to delay
• Verify need for surgery: Inguinal hernia complications? Impact on life?
Risk factors
• Warfarin history
o When & why started, compliance, INR levels (lability), any complications, diet (stable/unstable vitamin K intake)
o Importantly for mechanical valves, the target INR can be 2-3.5 or even higher if other risk factors
Dependent on which valve, number of prosthetic valves, age of prosthetic valve (older more
thrombogenic)
Therefore this patient is most likely at their therapeutic INR
• Consider thromboembolic risk: Use CHADS2 score to calculate (not CHA2DS2VASc which is AF specific)1, hx of strokes/TIA
(already covered in CHADS) presence and location of mechanical valve, AF, previous clots, VTE/PE risk
• Consider bleeding risk: Type of procedure (minimal, low, high risk which includes >45min procedure), use of regional
anaesthesia, bleeding diathesis, anti-thrombotics, other anti-thrombotic substances (fish oil), hx of bleeding (epixtaxis,
PR bleeding, bruising)
• Explore Hx of previous surgeries or periods of cessation of anticoagulation and what happened
Other
• Standard pre-operative workup of patient including assessment of patient factors, anaesthetic factors and surgical
factors (see Surgery: Pre and post-operative care)
• Complete rest of history systems review for pre-op assessment and important for discharge planning
Examination
• Vitals
• Observations: Ecchymoses (supratherapeutic INR)
• Inguinal hernia: Reducible? Strangulated? Small bowel obstruction?
• Cardiovascular exam: Listen for murmurs
• Respiratory exam: May impact anaesthetic
Investigations
• Standard pre-op workup
o ECG
o FBC (Hb and plt), EUC (CrCl <30 unsuitable for LMWH), LFT, BSL/HbA1C, coagulation studies, G+H
Imaging
• Abdo CT if suspecting inguinal hernia complications
Management
Anticoagulation
• Deciding pre-op anti-coagulation
o Dependent on bleeding risk vs thrombotic risk
o If thrombotic risk is transiently elevated: delay until risk returns to baseline
o If VTE in prior 3-4 weeks: delay or use a temporary IVC filter
o Consult cardiology & general surgery for advice and potential alternatives
Generally, must discontinue anticoagulation if surgical bleeding risk is high
Generally, must bridge if thromboembolic risk is high

1http://www.cec.health.nsw.gov.au/__data/assets/pdf_file/0006/458988/Guidelines-on-perioperative-management-of-
anticoagulant-and-antiplatelet-agents.pdf
43
• High thrombotic risk
o Cease Warfarin 5 days prior to operation
o Commence LMWH/UFH 4 days prior to operation
Enoxaparin 1mg/kg SC twice daily if CrCl>30, once daily if CrCl<30 (but consult haem/renal)
o Cease LMWH/UFH 24 hours/4-5 hours (respectively) prior to operation
o Check the INR the day before surgery
If INR>1.5, low dose PO Vitamin K 1-2mg (this is usually sufficient for reversal, rarely need other
reversal agents
• If too much K is given, will be unable to warfarinise him later so use a lower dose to bring it
down
Other reversal agents = Prothrombin complex concentrate (factors 2, 9, 10)
• Can use FFP (all factors) but this takes longer
o Recheck the INR the following day
If INR still supratherapeutic, consult haematology + repeat PO Vitamin K
o Following the surgery, review the wound + amount of fluid drainage + any sites of bleeding
Recommence Warfarin + LMWH/UFH together if satisfactory, typically 12-24h after surgery
o Cease LMWH/UFH when INR reaches therapeutic range
• Low/moderate thrombotic risk
o Cease warfarin 5 days prior to operation
o Bridging is not required
o Check the INR the day before surgery as per the “High VTE risk pathway”

Other management
• Other routine pre-op management

In this patient specifically!


• Delay surgery since it is elective follow the above algorithms
• If need urgent surgery can use reversal agents in liaison with haematology
• Warfarin counselling for future management (he may be above his INR target) - WARFARIN
References
• https://www.mja.com.au/journal/2000/172/12/consensus-guidelines-warfarin-therapy

44
Gastroenterology
Ascites (2020)
A 68 year old woman presents complaining of abdominal distension. On examination she has 10cm of shifting dullness but no organomegaly. How
would you assess her?
Introductory Statement
Provisional Diagnosis: clinical signs of ascites of which there are multiple differentials.
DDx: Most commonly liver cirrhosis and malignancy, but other causes including
• Ascites can be classified by the serum-ascites albumin gradient SAAG = serum albumin – albumin level of ascitic fluid
• High SAAG (Obsolete term: Transudate): ↑ portal vein pressure → ↑ hydrostatic pressure pushes fluid into peritoneal
cavity
o Portal hypertension
Pre-sinusoidal: splenic or portal vein thrombosis, schistosomiasis
Sinusoidal: cirrhosis (most common cause), alcohol-related liver disease, liver mets
Post-sinusoidal: RHF, constrictive pericarditis, budd-chiari syndrome (hepatic vein congestion)
• Low SAAG (Obsolete term: Exudate): ↓ intravascular osmotic gradient
o Hypoalbuminaemia nephrotic syndrome, severe malnutrition, protein-losing enteropathy (e.g. IBD, coeliac)
o Malignancy (can occur due to peritoneal carcinomatosis with lymphatic blockage, lymphoma obstruction by a
lymphoma, liver condition which may result in ↓ synthetic function on top of portal vein thrombosis e.g.
hepatocellular carcinoma)
o Infections TB (protein rich fluid from tubercles)
o Pancreatitis (accumulation of pancreatic fluid in peritoneal cavity)
Goals:
• Detect presence of potentially life-threatening complications (e.g. SBP and hepatic encephalopathy) as these would
require urgent treatment
• identify cause of ascites with main investigation being SAAG testing
• Manage appropriately with intraperitoneal antibiotics
History
Symptoms
• Sx of ascites
o Progressive abdominal distention duration and onset
Early satiety, weight gain, dyspnoea (elevated diaphragm)
o Associated symptoms
Abdominal pain
Abdominal wall hernias (due to ↑ pressure pushing on abdominal walls)
Peripheral and generalised oedema
Complications
• Spontaneous bacterial peritonitis abdominal tenderness, fever, altered mental status
o High suspicion for SBP if rapid or unexpected deterioration
• Haemoperitoneum sudden increase in ascites, symptomatic anaemia (occurs in 50% of HCC complicated by ascites)
Differentials
• Cirrhosis jaundice, easy bruising, leg swelling
• RHF known cardiac pathology, peripheral oedema
• Nephrotic syndrome known kidney pathology, DM, SLE etc.
• Infections TB (SOB, productive cough, systemic symptoms, sick contacts)
• Protein-losing enteropathy
• see DDx for others
Risk Factors/Causes
• Screen for risk factors or cause of cirrhosis (most common cause)
o Alcohol history - most common cause (60-70%). Need to know number of drinks per day (>80g/day for 10-20
years = high risk) and last drink
o Infection - hepatitis B/C (needle sharing, tattoos, multiple sex partners)
o Drugs - paracetamol toxicity, methotrexate
o Autoimmune hepatitis - Hx, history of other autoimmune causes
o Hereditary - haemochromatosis, Wilson disease, ɑ1 antitrypsin deficiency
o Prolonged cholestasis - Primary biliary cirrhosis (PBC), secondary biliary cirrhosis (due to stones, fibrosis, tumour
or parasitic infection)

Other
• previous episodes of ascites/ hospitalisations

45
Examination
• Vitals (exclude shock) + determine if altered mental state
• GIT exam
o Ascites
Flank dullness + shifting dullness
Organomegaly (liver + spleen)
Signs of SBP (generalised peritonitis)
• ± associated pleural effusion (decreased breath sounds, dullness to percussion)
o Signs of cirrhosis as a cause
General inspection – jaundice, altered mental stat
Hands – ecchymoses, petechiae, clubbing, leukonychia, palmar erythema, muscle wasting, hepatic flap
Face – scleral icterus
Chest – dilated veins, spider naevi, gynecomastia
Abdomen – caput medusae, dilated veins
• Cardiovascular examination
o Signs of right heart failure ↑ JVP, peripheral oedema, hepatosplenomegaly, ascites
Investigations
Diagnostic
• U/S guided Paracentesis ± culture
o Indications: new or worsening ascites or suspected SBP
Can do just U/S to detect
o Tests: appearance + colour, cell count + differentiations (↑ in low SAAG), albumin and total protein, ↑ LDH
(bacterial or malignant), MCS, acid-fast stain (TB), cytopathology (malignant cells)
o SAAG = serum albumin – albumin level of ascitic fluid.
Criteria High SAAG ≥ 11 g/L LOW SAAG < 11 g/L
Colour Clear, sometimes opalescent Cloudy (infection or malignancy)
Bloody (malignancy or traumatic paracentesis)
Milky (chylous)
Dark brown (bilirubin from potential perf)
Cell count and differentiation ↓ Cell count ↑ Cell count
May be elevated in SBP ± ↑ Neutrophil count (infection)
Protein concentration ?↑ Protein levels (> 2.5 g/dL) ↑ Protein levels (> 2.5 g/dL)
- Right heart failure - Hepatic malignancy
- Peritoneal carcinomatosis
↓ Protein levels (< 2.5 g/dL) - Pancreatitis
- Hepatic cirrhosis - Chylous ascites
- Tuberculosis

↓ Protein levels (< 2.5 g/dL)


- Nephrotic syndrome
o Complications: bleeding, perforation of bowel, bladder, liver, infection
Laboratory
• FBC (exclude ↑ WCC suggesting SBP)
• EUC (↑ urea and Cr in hepatorenal syndrome)
• LFTs (albumin, liver pathology)
• Coags (cirrhosis = coagulopathy and INRs used in child- -score)
• Aetiology of cirrhosis: viral hep screen, autoimmune screen (anti-nuclear anti-smooth muscle, IgG4), iron studies, TFT,
metabolic screen for NASH, 𝛼𝐹𝑃 for HCC
Imaging
• [alt.] Initial abdo U/S (cirrhosis) ± doppler
o confirm ascites, liver surface nodularity, patency of hepatic and portal veins
• ± CT abdo ± liver biopsy (if suspicious or malignancy, biopsy only if inconclusive)
• ± Fibroscan follow-up cirrhosis rather than initial (works on impedance and gives the density of the liver)
Management
• I would treat this patient in conjunction with a senior colleague. This involves management of life-threatening
complications + ascites itself
• Indications: Treat if symptomatic (tense ascites, dyspnoeic or early satiety), aim to improve symptoms rather than
completely remove fluid
Definitive
• Treat reversible causes
o Thrombosis anticoagulation
46
o TB tuberculostatics
• Sodium restriction (including drugs containing high amount of sodium)
• Water restriction or avoiding overhydration
• Diuretics (need to be careful of electrolytes and renal function)
o Useful if portal hypertension is cause
o Spironolactone PO, amiloride if spironolactone not tolerated.
o ± Frusemide if spironolactone alone is inadequate
• ± Albumin given to reduce complications related to fluid shifts and ↑ survival – dose = 20% IV at the rate of 6-8g per L of
drained ascites
• Therapeutic paracentesis – large volume drain of 5-10L over 1-6 hours
o ± repeated paracentesis if refractive
• [Refractory] Percutaneous insertion of a shunt between hepatic and portal veins (trans jugular intrahepatic
portosystemic shun; TIPS) to reduce portal HTN

Supportive
• Patient education
• Weight control
• Correct malnourishment (common; dietician referral)

Complications
• Spontaneous bacterial peritonitis
o Diagnostic if WCC >500 or ANC > 250 on ascitic tap. Likely pathogens = GN bacilli (E. coli, Klebsiella); s.
pneumonia.
o Treatment = IV ceftriaxone/cefotaxime + albumin to ↓ risk of hepatorenal syndrome (albumin 20% 100mL IV
BD for 3 days)
o Prophylaxis against SBP after a first episode – oral bactrim OR norfloxacin (if hypersensitive)
• Cirrhosis complications
o Hepatic encephalopathy
Lactulose 30mL PO Q1-2H to reduce GI absorption of toxic amines (converted to lactic acid by
intestinal flora neutralises NH3 to NH4 which is then excreted in faeces); add rifaximin (non-
absorbable antibiotic) if recurrent or chronic
o Hepatorenal syndrome
Correct precipitant
Pharmacology Midodrine (an alpha‑adrenergic agonist), causes a systemic vasoconstriction. When it
is used in combination with octreotide (a somatostatin analog, it functions as an inhibitor of splanchnic
vasodilatation) and albumin (plasma expander), systemic and renal hemodynamic status is improved
thereby improving renal function.
Surgical/interventional definitive management same as below

Long-term
• Can be assessed with Child-Pugh score, which estimates hepatic reserve in liver failure. Predictor of morbidity and
mortality

47
Hepatocellular Carcinoma (2020)
A 58 year old man presents with a two month history of right upper quadrant discomfort and weight loss. His liver function tests are normal but a CT
scan of his abdomen shows a solitary mass in the liver. How would you assess him?
Provisional and Differential Diagnosis
Provisional Diagnosis: My provisional diagnosis is hepatocellular carcinoma
DDx: Mass in the liver
• Benign – hepatic cyst (simple or complex), hepatic haemangioma, focal nodular hyperplasia, hepatic adenoma (most
commonly in premenopausal women >30y; associated with COCP), regenerative nodules, hepatic lipoma, hydatid cyst
• Malignant – hepatocellular carcinoma (most commonly in cirrhosis), intrahepatic cholangiocarcinoma, metastatic disease
(however usually multiple lesions), hepatic angiosarcoma
• Would also consider non-liver mass differentials like a colorectal malignancy
Goals:
• History and examination to determine aetiology of solitary mass
• Investigate with LFTs, AFP and triple phase CT to confirm a HCC
• Management options include surgical intervention, transplantation, non-surgical ablation, Sorafenib therapy or palliation
depending on stage (e.g. BCLC system)
History
Symptoms
• Sx of hepatocellular carcinoma usually asymptomatic apart from symptoms of underlying disease (chronic hepatitis or
cirrhosis). In advanced disease
o Local – pain, mass, jaundice, ascites
o Constitutional symptoms of malignancy – weight loss, fever, night sweats
o Symptoms of metastases – bone pain, dyspnoea, signs of raised ICP
Risk factors
• Risk factors for HCC – cirrhosis (80% of cases), hepatic pathologies independent of cirrhosis (chronic HBV/HCV, AAT
deficiency, hepatic adenoma: 1% risk of malignant transformation, NASH, wilson’s disease, hepatic autoimmune diseases
etc.)
o alcohol, smoking, drugs including IVDU
Other
• Medications – in particular those affecting liver function e.g. paracetamol
• ECOG status (ranges from 0 = fully active; 4 = bedbound; 5 = dead)
Examination
• Often nil findings except for underlying pathology (cirrhosis or hepatitis commonly)
• Check vitals
• General observation – cachexia, LOC (hepatic encephalopathy)
• Gastro exam
o palpable mass, symptoms of cirrhosis (e.g. palmar erythema, spider naevi, gynaecomastia, coagulopathy),
evidence of hepatic decompensation (e.g. variceal bleeding, ascites, hepatic encephalopathy)
• DRE if considering colorectal malignancy
Investigations
Laboratory
• Routine: FBC, EUC, CMP (potentially paraneoplastic syndromes)
• LFT + Coags (synthetic function)
o Child-Turcotte-Pugh score can be calculated to stratify underlying liver disease
• HBV/HCV serology
• G+H (pre- surgical workup)
• AFP (tumour marker)
o In patients with cirrhosis of the liver or chronic hepatitis B/C infection, AFP is used as a screening test for HCC!
o Also helpful in monitoring response to therapy
Imaging
• US: best initial test
• triphasic CT: confirmatory
o non-contrast, arterial and portal venous phases. Helps differentiate lesions based on vascular enhancement.
Works on the basis that the liver has dual blood supply (hepatic artery + portal vein)
o HCC – mass with increased vascularity during arterial phase, with washout in later phases (due to angiogenesis).
Solitary mass in a liver that appears cirrhotic.
o Metastases – often multiple
o Haemangioma – small peripheral enhancing lesion in a young healthy female with a normal appearing liver
• Staging
48
o CT chest/abdo/pelvis ± PET scan. TMN staging or BCLC staging (helps to guide treatment)
NB: BCLC = Barcelona clinic liver cancer
• Surgical planning – MRI with gadolinium-based contrast or MRA
Invasive
• Biopsy not required for diagnosis of HCC. If diagnostic uncertainty after imaging, consider FNA or surgical resection
(surgical resection preferred if HCC cannot be excluded)
o FNA often non-diagnostic when evaluating hepatic adenomas and focal nodular hyperplasia. Also has risk of
bleeding or seeding neoplastic cells.
Management
Management of this potential HCC involves an MDT approach, ideally in a specialised unit. I would consult general surgery and
oncology
• The choice of therapy is based on the number, size and location of tumours, the presence of vascular invasion or
metastatic disease, the severity of underlying liver disease and the performance status of the patient.
Definitive
• Surgical
o Surgical resection
Liver resection – 10% have resectable tumour. May be offered to patients with compensated cirrhosis
and without portal HTN, but these patients remain at risk of developing further HCC (60-80% have
recurrent tumours in 5 years)
o Liver transplantation
Liver transplantation if diagnosis made early – offers the best chance of long-term survival. Should be
considered in patients fulfilling criteria (e.g. Milan criteria)
• Non-surgical
o Ablation
If unresectable (most patients) – options to shrink and scar tumour include:
• Thermal ablation
• Irreversible electroporation
• Embolization
• Percutaneous injection therapy
• External beam radiation therapy
o Systemic Therapy
Consider Sorafenib – if advanced HCC and compensated cirrhosis, consider sorafenib (multikinase
inhibitor; prolongs survival)
Follow-up
• HCC surveillance in patients with cirrhosis (from any cause) or chronic Hepatitis B is recommended
o Perform liver ultrasound every 6 months
o If a nodule smaller than 10 mm is detected, repeat ultrasound every 3 months to check for an increase in
nodule size; if a nodule 10 mm or larger is detected, further investigation with contrast-enhanced dynamic
imaging (i.e. computerised tomography or magnetic resonance imaging) is required.
• Vaccination against HBV in high-risk individuals

Palliation
• if decompensated cirrhosis and advanced HCC, give supportive care only.
• Allied health
o counselling, social work, palliative care if non resectable

49
Massive Haematemesis (2020)
A 36 year old man is admitted following a haematemesis. His condition is stable and he is awaiting endoscopy. However, he has a further bleed and
his BP falls to 80/65 mmHg. How would you manage him?
Introductory statement
Provisional Diagnosis:
• This patient is likely in haemorrhagic shock following further upper GI bleeding. This is a medical emergency which
requires simultaneous assessment and management.
DDx: The upper GI runs from the mouth up to the ligament of Treitz (2nd part of duodenum). Causes of a upper GI bleed include:
• Oesophageal vascular (gastro-oesophageal varices), Mallory-Weiss tears, inflammatory (oesophagitis), malignancy
(oesophageal cancer can invade aorta and cause massive bleed)
• Gastroduodenal inflammatory [crohn’s, gastritis, gastroenteritis (shigella, e.coli)], vascular (angiodysplasia), PUD,
malignancy (Zollinger-ellison syndrome), aortoenteric fistula (rare)
• Non-GI causes epistaxis
Goals of Management
• A to E assessment with aggressive fluid resuscitation and blood transfusions to stabilise the patient
• Brief H/E/I to identify the possible causes of upper GI bleeding
• Manage appropriately with referral to gastroenterology for an urgent endoscopy
ABCDE Assessment
• Call for senior help – activate rapid response/MET call
• Airway – ensure airway is patent and protected, assess for possible aspiration with haematemesis
• Breathing – assess RR and saturation, give oxygen if desaturating
• Circulation
o Assess HR, BP, cap refill, peripheral pulses and warmth
o Insert 2x large bore cannulas and start fluid resuscitation with 500mL fluid boluses
o Send bloods
VBG, FBC, EUC, LFT, Coags, Group and crossmatch (if not done already)
o ± blood products with transfusion of packed RBC
In variceal bleeding, aim for Hb between 70-80
• Disability – assess GCS/AVPU, Alcohol withdrawal scale (if relevant)
• Exposure – temperature, prevent hypothermia
Immediate management
• Stop the bleeding
o Cease + reverse anticoagulation if on anticoagulants
o PPI infusion or boluses
o If variceal bleeding IV Terlipressin (or octreotide) to reduce bleeding and antibiotic prophylaxis with
ceftriaxone
Terlipressin = vasopressin reduces portal vein pressure reduces pressure in oesophageal varices
(risk of peripheral vasoconstriction)
• ± prokinetics e.g. erythromycin or metoclopramide to increase clearance and improve view for endoscopy
History
Symptoms
• Review notes for initial presentation + previous notes
• Bleeding
o Haematemesis vs PR bleeding (stem just says ‘further bleed’)
o Haematemesis
Colour – coffee ground vs bright red (more severe)
Volume/number of episodes
Associated with significant vomiting/retching/coughing beforehand
Previous episodes
o PR bleeding
bright red (brisk upper GI bleed or lower GI bleed), dark, melena
o Other symptoms – epigastric pain
Differentials
• Hx of Chronic liver disease/Alcohol use/HCV or HBV infection
• Dry retching/excessive vomiting (Mallory-weiss tears)
• Hx of PUD, known H. pylori infection
• Hx of oesophagitis/gastritis/GORD
• constitutional symptoms of malignancy
Risk Factors

50
• Bleeding risk Previous bleeds, bleeding diathesis (including inherited bleeding disorders, antiplatelets or
anticoagulants, others including NSAIDs)
• RF for CLD and mallory-weiss tear alcohol intake
Other
• Comorbidities influencing management
o CAD/Pulmonary disease more susceptible to adverse effects of anaemia need to be maintained at higher Hb
and anaesthetic issues pre-procedure
o Renal disease/CHF risk of fluid overload with resus
o Coagulopathies/liver disease more difficult to control bleeding, may need transfusion of prothrombinex or
FFP
o Dementia/encephalopathy higher risk of aspiration
• Previous endoscopies
Examination
• Re-check vitals
• Abdo Signs of peritonism – may suggest perforation, Signs of chronic liver disease
• DRE PR bleed
o If frank blood concerning as upper GI bleed is transitting so fast that it is not exiting as malaena
Investigations
Diagnostic
• [1] Upper GI endoscopy (oesophagogastroduodenoscopy EGD)
o As below (diagnostic and therapeutic)
• [2] NGT to retrieve stomach contents and confirm that the bleed is in the upper GIT
Laboratory
• Same as resuscitation
Imaging
• ± CXR (free air under diaphragm) or CT If suspected perforation
Management
• Consulting the gastroenterology registrar to arrange a scope
Emergent
• As above
Definitive
• [1] Emergent (within 12 hours) Upper GI endoscopy (oesophagogastroduodenoscopy)
o Diagnostic modality + gold standard treatment. Early endoscopy (within 24h) recommended.
o Treatment: Clip, rubber band Ligation (especially for varices), thermal coagulation, balloon tamponade, Injection
therapy (epinephrine more for ulcers), Sclerotherapy
o Risks of endoscopy pulmonary aspiration, GI perforation, increasing bleeding while attempting therapeutic
intervention, aesthetic risk
• [2] Surgical exploration (laparotomy)
o Usually resect the section that’s bleeding.
• Options:
o Variceal bleeding
[Alt.] Sengstaken-Blakemore tube
• Gastric balloon + oesophageal balloon
o Inflate gastric balloon first and apply traction which will stop bleeding at gastro-
oesophageal junction which is most common site of bleeding
o ± inflate oesophageal balloon if still unstable but risk of pressure necrosis.
[Alt.] Interventional radiology for something like TIPS (transjugular, intrahepatic, portosystemic shunt)
Supportive
• Analgesia as required
• Prep for scope
o NBM prep for scope
o ± prokinetics (erythromycin, metoclopramide) – improves gastric visualisation
Follow-up
• Monitor for continued bleeding
• Monitor for alcohol withdrawal and treat appropriately
• Address cause
o If variceal consider β blocker (splanchnic vasoconstriction), portosystemic shunt or transplant
o If PUD cease NSAIDs, test for H. pylori and eradicate
o If alcohol intake educate how this contributes to increased risk of bleeding

51
Ulcerative colitis (2020)
A 38 year old woman presents with a two day history of profuse bloody diarrhoea with fever. She was diagnosed as having ulcerative colitis 10 years
ago and has been treated with salazopyrine and corticosteroids. She has not had any symptoms for the past three years and ceased all treatment six
months ago. How would you assess and manage her?
Introductory Statement
Provisional Diagnosis: Considering this patient’s history of UC which is a chronic relapsing remitting inflammatory bowel condition,
she most likely has developed an acute flare.
Goals:
• This is a medical emergency treated promptly with supportive measures in addition to pharmacotherapy.
DDx:
• Infectious
o Bacterial invasive (salmonella typhi, yersinia), inflammatory (salmonella (non-typhoidal), shigella,
enterohaemorrhagic e. coli, campylobacter jejuni), Note: secretory e.g. S. aureus, vibrio cholerae will not
produce blood
o Viral (not usually bloody) CMV, ?norovirus, ?rotavirus
• Neoplastic (CRC), vascular (ischaemic colitis), diverticulosis/itis, pseudomembranous colitis (c. diff), angiodysplasia,
perianal fissure, autoimmune (crohn’s), invective colitis
ABCDE assessment
• A-E assessment, aiming to ensure this lady is not in hypovolemic shock or have a colonic perforation secondary to toxic
megacolon (severe pain, marked abdo distension, fever, N/V, peritonitis)
History
Symptoms ± Complications
• Assess severity of flare
o #bowel habits (≥6 loose bloody stools), quantify blood, cramps, temperatures, weight loss/appetite, abdominal
pain, N&V
Complications
• Severe bleeding, fulminant colitis + toxic megacolon
o Fulminant colitis: >10 stools/day, continuous bleeding, abdominal pain, distension, acute severe toxic symptoms
(fever, anorexia)
• Extra-intestinal manifestations for IBD (both UC and crohn’s)
System Extra-intestinal manifestations
Dermatalogic Erythema nodosum (more likely crohn’s), pyoderma gangrenosum (more likely UC), aphthous stomatitis
Rheumatologic Ankylosing spondylitis, sacroiliitis, OA Rheumatologic EIM are the most common
Fistulas Enterocutaneous, gastrocolic, enterovesical (have pneumouria), rectovaginal/anovaginal
Liver Primary Sclerosing Cholangitis – more common in UC
Eyes Iritis, Uveitis, Episcleritis (benign), Scleritis (severe) – more common in Crohn’s
Malabsorption More common in Crohn’s. can occur in UC due to backwash ileitis
Cholesterol stones, kidney stones, vitamin deficiencies e.g. B12 anaemia
Differentials
• Sick contacts, travel, food, recent broad Abx use, fevers, diverticular disease
Risk Factors/Causes
• Contributing factors for UC flare – recent cessation of treatment
Other
• PMHx UC -extent (proctitis, pancolitis), assess severity (Montreal classification), treatment (biologic naïve, previous
treatment failure, surgical resection including amount of remaining bowel as this affects absorption), course of
remission, why ceased medications, gastroenterologist
• Factors affecting treatment pregnancy (anti-folates cannot be used), chance of being pregnant, sexually active (drugs)

Examination
• General: anaemic, cachexic, temp
• Abdominal examination:
o Distention, masses (?abscess)
o Exclude red flags
Fulminant colitis/toxic megacolon – severe lower quadrant pain or diffuse abdominal pain with
diarrhoea, abdominal distension, fever, hypovolemia
Exclude perforation – peritonitis
• DRE for perianal disease
• Extra-intestinal signs of IBD:

52
o As above
Investigations
Bedside
• Urine B-HCG
• Stool microbiology – Stool MCS, C. diff toxin, faecal calprotectin (marker of acute flare)
Laboratory
• FBC (anaemia, ↑WCC), EUC (electrolyte losses, AKI), LFT (hypoalbuminemia – part of severity assessment, primary
sclerosing cholangitis → cholangiography, during ERCP), CRP/ESR (severity of inflammation)
Imaging
• AXR:
o Toxic megacolon (sequalae of fulminant colitis) diameter ≥6cm + systemic toxicity. Will require surgery
(colectomy)
o Ischaemic colitis unspecific mild diffuse bowel dilatation, thumbprinting due to bowel wall thickening
Invasive
• ± Flexible sigmoidoscopy to confirm presence of UC, visualise severity, extent of inflammation, and to obtain biopsies to
exclude presence of infection (e.g. CMV, HSV, C.diff) can inform management
• Full colonoscopy should be avoided as may precipitate toxic megacolon
Management
• Consider inpatient vs outpatient (montreal severity scale), MDT approach, aggressive medical therapy, though some
surgery may be required
Supportive:
• NBM + IV fluid and electrolyte replacement + IDC (monitor UO)
• ± nutritional support
• VTE prophylaxis vital as in IBD there is a loss of anti-thrombin III. If renal function sufficient, clexane (enoxaparin)
40mg SC once daily
• Stool charting
• In severe IBD disease avoid, NSAIDS, opioids, anti-diarrhoeal (loperamide), anticholinergic medications (relieves
cramping) as these can precipitate toxic megacolon
o Can use anti-diarrhoeal and anticholinergic in mild disease
Definitive
• Acute for severe IBD
o [1] IV hydrocortisone 100mg q6h or methylprednisolone
reassess after 3-5days → PO glucocorticoids (prednisolone 1mg/kg max 60mg)
o ± Antibiotics ciprofloxacin and metronidazole
o [2] ± Consider prompt medical salvage therapy (cyclosporin or infliximab)/colectomy for patients with acute
severe UC who fail to respond/continue to deteriorate despite 3-5d of IV corticosteroid
• Maintenance
o 1) prevent relapse 2) prevent bowel damage 3) detect dysplasia, prevent carcinoma
o these medications allow for induction in the case of non-severe IBD
o [1] high dose PO and rectal 5-ASA e.g. mesalazine (top ± tail 5-ASA)
Sulfasalazine bad A/E so use its derivatives
+ Wean steroids no role in long-term maintenance Tx
o [2] ± immunomodulators (MTX OR mercaptopurine or azathioprine) azathioprine breaks (more A/E) down into
mercaptopurine
o [3] ± biologics (infliximab or vedolizumab)
• Surgery
o Surgical resection of bowel, potentially curative as it removed the focus of disease and reduces cancer risk
o Proctocolectomy with either ileal pouch–anal anastomosis or end-ileostomy
Complications
• Fulminant colitis
o NBM, give IV glucocorticoids, broad spectrum ABx (amp + gent + metro). If not responding, give
immunomodulators or biologics, or refer for an urgent colectomy
Appendix
See Paediatrics: IBD for comparison of ulcerative colitis and crohn’s

53
Renal
Acute Kidney Injury – AKI (2020)
2020 (case 1): A 78 year old man has been hospitalised for one week following a stroke complicated by pneumonia. Routine biochemistry shows that
his serum creatinine has increased to 280umol/L (60-120) from 110umol/L on admission. How would you manage him?
Case 2: A 66 year old man is hospitalised with worsening congestive cardiac failure. His dose of frusemide is increased and is also commenced on
enalapril. The next day his serum creatinine has increased from 115umol/L (60-120) on admission to 175umol/L. How would you assess and manage
him?
Phases of AKI
• 1) Initiating event hours to days
o Symptoms of the underlying illness causing AKI may be present.
• 2) Oliguric or anuric phase (maintenance phase) 1-3 weeks
o Progressive deterioration of kidney function
↓ urine production, ↑ retention of urea and creatine
o Complications: fluid retention, hyperkalaemia, metabolic acidosis, uraemia, lethargy, asterixis
• 3) Polyuric/diuretic phase ~2 weeks
o GFR returns to normal while tubular reabsorption remains disturbed
o Complications: loss of electrolytes and water (dehydration, hyponatraemia, hypokalaemia)
• 4) Recovery phase months to years
o Kidney function and urine production normalises
Definitions
• Oliguria = < 0.5mL/kg/hour of urine production (i.e. <20-30mL/hr in a person of normal weight)
• Anuria = no urine output for >6 hours
Introductory Statement
Provisional Diagnosis: This man has AKI, which is defined by the KDIGO guidelines as 1) increase in serum Cr by ≥26.5μmol/L
within 48h, increase in serum Cr to ≥1.5x baseline within 7 days or urine volume <0.5mL/kg/h for 6 hours.
DDx:
• Case 1 In an elderly patient with stroke + pneumonia, I am most concerned about pre-renal causes such as sepsis and
dehydration.
• Case 2 In this patient, his AKI is likely pre-renal due to a combination of over-diuresis (reducing renal perfusion) and
ACEi use. Also may be contributed to by cardiorenal syndrome.
o Note: ACEi should not be used in acute setting as it can acutely worsen renal function, good for mortality benefit
in the long term but not useful in acute phase.
Goals:
• Rapid A to E assessment to Identify and treat life threatening complications of AKI including fluid overload,
hyperkalaemia, uraemia (e.g. pericarditis), metabolic acidosis ± sepsis (case 2)
• Take a targeted H/E/I to identify the underlying cause ± renal consult
• I would then identify and reverse the underlying cause [insert overview of relevant therapy]
Full list of differentials for AKI
• Pre-renal causes
o Hypovolemia
Volume depletion: e.g., haemorrhage, vomiting, diarrhea, sweating, burns, diuretics, poor oral intake,
acute pancreatitis
Decreased circulating volume: e.g., hepatorenal syndrome, cirrhosis, liver failure, nephrotic syndrome,
congestive heart failure
o Hypotension: e.g., sepsis, dehydration, cardiogenic shock (decreased cardiac output), anaphylactic shock
o Drugs affecting GFR – NSAIDs (constrict afferent arteriole), ACEI/ARBs (dilate efferent arteriole), cyclosporine,
tacrolimus
o Renal artery stenosis
• Renal
o Tubulointerstitial
acute tubular necrosis (ATN) [85% of renal AKI’s] caused by sepsis, infection, ischaemia and/or
nephrotoxins (e.g. contrast, medications, rhabdo myoglobinuria, haemoglobinuria)
acute tubulointerstitial nephritis (AIN) caused by drug-induced, infectious, immunological
o Vascular – vasculitis, malignant HTN, HUS, TTP
o Glomerular – glomerulonephritis (usually cause CKD though) e.g. Goodpasture, Wegener granulomatosis, post-
streptococcal GN, lupus etc.
• Post-renal
o Congenital malformations (e.g. posterior urethral valves)
o Acquired obstructions

54
Benign prostatic hyperplasia (BPH), Iatrogenic/catheter-associated injuries, Tumours, Stones, Bleeding
with blood clot formation
o Neurogenic bladder (e.g., multiple sclerosis, spinal cord lesions, or peripheral neuropathy)
ABCDE Assessment
• Focus on life threatening complications
o B fluid overload (APO)
o C assess for hypovolaemia, sepsis, signs of hyperkalaemia, uraemic pericarditis
History
Symptoms
• Symptoms of AKI
o May be asymptomatic
o Monitor urine output oliguric or anuric, may have pain over bladder or flanks
Ask if they are passing urine e.g. IDC vs. toilet vs. incontinent
o Signs of fluid depletion/overload (depending on aetiology) weight change if known, SOB
o Signs of uraemia anorexia/nausea, encephalopathy, pericarditis, pruritis
o Fatigue, altered mental state
• Symptoms of CHF for case 2 LHF, RHF
Differentials
• Determine cause of AKI by asking symptoms of the relevant aetiology
o pre-renal systems review for infection, vomiting, diarrhoea, dehydration, medications e.g. NSAIDs/ACEI
o renal causes of glomerular disease e.g. diabetes, infections, exposure to nephrotoxic substances e.g.
contrast, gentamicin
o post-renal history of difficulty passing urine or incomplete emptying, pain over bladder/flanks, hx of kidney
stones or carcinoma
Examination
• Vital signs
• Fluid status – fluid chart, input + output
o signs of overload e.g. pitting oedema, raised JVP, bibasal crackles
o signs of hypovolemia e.g. sunken eyes, decreased skin turgor, dry mucous membranes, cool peripheries,
capillary refill
• Assess IDC + urine bag if present to check for obstruction
• Cardioresp exam
• GIT exam renal mass, palpable bladder (urinary retention)
Investigations
• KDIGO guidelines as 1) increase in serum Cr by ≥26.5μmol/L within 48h, increase in serum Cr to ≥1.5x baseline within 7
days or urine volume <0.5mL/kg/h for 6 hours.
Bedside
• ECG (hyperkalaemia = peaked T waves + others)
• VBG sepsis (metabolic acidosis and lactate)
• Urine MCS + urinalysis; look for cells, casts, crystals
o Renal RBC casts, fatty casts (nephrotic syndrome), muddy brow and/or epithelial and/or granular casts (ATN)
± Proteinuria suggests an intrinsic renal cause (glomerular injury)
o Pre-renal hyaline casts
• Urine sodium (↓ urine sodium concentration and ↓ fractional excretion of sodium) pre-renal (since kidneys working
normally to rectify hypovolaemia)
• Urine ACR
• Bedside bladder scan
Laboratory
• FBC + differential (sepsis), EUC(↑Cr ,BUN), CMP
o ↑ BUN:Cr pre-renal
o ↓ BUN: Cr renal
• ± Blood cultures/septic screen if febrile
Imaging
• ± CXR APO
• ± Renal US for obstructive cause, consider CT abdo/pelvis if US shows an abnormality
Special Tests
• ± Renal biopsy – useful if suspecting acute GN or acute allergic interstitial nephritis
o Indications – diagnosis not certain, pre-renal azotemia or ATN unlikely, oliguria persists >4 weeks
Differentials

55
• Consider additional investigations based on clinical suspicion – septic screen, serology for HBV/HCV/HIV, autoimmune
screen including ANA/ANCA/dsDNA; ASOT (PSGN), serum EPG (multiple myeloma)
Management
• My goals of management are to call for senior help. I would give supportive treatment, reverse the underlying cause and
manage complications of AKI
Definitive
• Consider and reverse underlying cause
o Pre-renal causes
fluid challenge (with 250mL) + fluids
• NS rather than Hartmann’s (in context of AKI since K may exacerbate issues)
• Aim for >0.5mL/kg/hr and check if urine output has picked up within 4 hours
• If not improved, repeat fluid challenge.
± diuretics to ↑ flow (if multiple fluid challenges attempted and not anuric and patient is stable,
hypervolemic and no cardia/renal issues)
Septic shock antibiotics
o Renal - treat underlying cause (e.g. steroids, cyclophosphamide glomerulonephritis)
Replete volume with NS in ATN or contrast-induced renal dysfunction
ATN Remove offending agent e.g. NSAIDS, ACEI/ARB, nephrotoxic medications including
aminoglycosides and contrast + supportive
RPGN or interstitial nephritis corticosteroid or immunosuppressive therapy, ± treat infection
Vascular treat accordingly
± trial of diuretic to increase urine flow
o Post-renal
catheterization to decompress + monitor urine output.
± ureteral stenting or percutaneous nephrostomy.
• Correct electrolyte disturbances EUC/CMP every 4-6 hours
o ± Hyperkalaemia See Hyperkalaemia case
• Metabolic acidosis - give bicarbonate; dialysis indicated if volume overloaded + pH <7.1

Supportive
• Review medications
o Cease nephrotoxins (antibiotics including gentamicin)
o cease drugs affecting GFR (diuretics, ACEI/ARBs, NSAIDs)
o avoid hyperglycaemia (causes osmotic diuresis)
o Adjust dosages of renally cleared medications
• ± IV fluids if depleted for pre-renal and some intrinsic causes, but avoid volume overload (which is likely), fluid balance
daily including urine output
o Consider fluid challenge
o For fluid overload can consider salt restriction and diuretic
• Monitor for good urine output and reassess patient in 3-4 hours to ensure patient is not overloading
• Nutritional support (catabolic state)

Complications (some are life threatening)


• Hyperuricaemia can cause encephalopathy, pericarditis or bleeding diathesis (mast cells cannot degranulate in the
presence of uraemia)
• APO (due to volume overload) oxygen/ventilation support
• Infection (occurs in 50-60%)
o common (e.g. pneumonia, UTI, wound infection, sepsis). Multifactorial, uraemia impairs immune function.
• Indications for dialysis
o If the following are refractory to medical therapy (AEIOU):
o Acidosis, electrolyte imbalance (hyperkalaemia), intoxication (methanol, ethylene glycol, lithium, aspirin),
overload (fluid), uraemia (encephalopathy, pericarditis, bleeding)

56
Urinary retention (2020)
An 82-year-old man is admitted to hospital for investigation of falls. He does not pass urine for the next 24 hours. How would you manage him?
• NB: Note the context of anuria not just decreased urine output
• see Surgery: Post-op urinary retention
Differences
• PD:
o ? Traumatic injury
o Potential cause of AKI is rhabdomyolysis due to fall ± long lie, but consider other pre-renal, renal and post-renal
causes.
• History
o Should try to come to practical side of assessment and management first e.g. what can you ask the nursing staff
o Falls workup including baseline function

57
Hyperkalaemia post-op (2020)
You receive a call from the surgical ward about a patient who is 2 days post small bowel resection for adhesions and ischaemic gut. You are told the
patient is looking well but their blood tests show a potassium of 6.5 mmol/L (NR 3.5- 5.2 mmol/L). How would you manage this?
Introductory Statement
Provisional Diagnosis: This patient has hyperkalaemia with a potassium of 6.5 mmol/L which is concerning and requires immediate
assessment and management to prevent life-threatening complications e.g. arrythmias and muscle paralysis leading to respiratory
failure. In this case, I would consider a broad range of differentials but most pertinently iatrogenic fluids, insulin deficiency
including:
DDx:
• increased intake (diet, massive blood transfusion, iatrogenic fluids)
• change in distribution (insulin deficiency, acidosis, B2 antagonists/alpha agonists, cell lysis (burns, rhabdo, TLS),
metabolic acidosis)
o NB: ischaemic gut could cause cell lysis
• decreased excretion (Na+ pump in distal nephron [e.g. 1o adrenal insufficiency, spironolactone, Addison’s], AKI/↓ GFR)
• pseudohyperkalaemia (prolonged tourniquet, haemolysis 2o delayed sample processing, extreme leucocytosis or
thrombocytosis)
Goals
• Rapidly assess and manage this patient to prevent complications of hyperkalaemia e.g. life-threatening arrythmia.
• Targeted H/E/I to identify and reverse the hyperkalaemia as well as the underlying cause

ABCDE Assessment
• Senior help + call ICU for monitored cardiac bed
• B: potential hypoventilation if significant weakness
• C:
o obtaining an immediate ECG looking for ECG changes associated with hyperkalaemia (tall tented T wave, P wave
widen + flatten, P-R interval lengthens, QRS widens, sine wave with blending of QRS and T wave pre-terminal
rhythm (ventricular arrythmias) → cardiac arrest (VF, asystole, PEA)) continuous cardiac monitoring
o ± IV calcium gluconate 10mL of a 10% solution (1g) over 2-3mins if >6.5 stabilise the cardiac membrane
immediately
Not used if digoxin toxicity as the cause digoxin immune Fab + shifting intracellular methods below
o Short-term shift of K intracellularly
Insulin/dextrose– takes 15min, lasts for a few hours.
• Dosage = 10-20U, followed by 50mL of 50% dextrose (25g)
Nebulised salbutamol (B-agonist) – takes 90 min
• Dosage 10-20mg nebulised in 4mL of saline over 10 min (4-8 times the normal dose for
bronchodilation).
± IV sodium bicarbonate 50-200mmol (limited efficacy) for metabolic acidosis
o ± Haemodialysis – if refractory (potentially method of choice in the patient, as they have CKD)
o Bloods FBC, EUC/CMP (renal function, repeat K to exclude pseudohyperkalaemia due to ↑WCC artefact from
blood sample handling), BSL, VBG (acidosis can cause hypo, alkalosis can cause hyperkalaemia)
• D: GCS (uraemia can ↓), worst case scenario intubate + ICU
History
Surgical history
• Review patient notes, Records from operation (any complications, blood transfusion given, progress since surgery)
• Fluid status (IV fluid prescription, input + output)
• Medication review
Symptoms
• Symptoms of hyperkalaemia (NB: patient is likely tolerant if CKD) usually only present if severe e.g. >7.0
o cardiac arrhythmias (palpitations), MSK (ascending muscle weakness, flaccid paralysis, paraesthesia decreased
deep tendon reflexes); Respiratory (respiratory failure); GI (Nausea/vomiting, intestinal colic, diarrhoea)
o Note: hypokalaemia muscle paralysis/weakness, cardiac arrhythmias/palpitations (± syncope), constipation
• Past episodes of hyperkalaemia
Differentials
• Determine cause of hyperkalaemia see DDx
o ↓ excretion CKD, potassium sparing anti-hypertensives
o Change in distribution Medication review, uncontrolled DM (insulin deficiency)
o ↑ intake → recent transfusion
Examination
• Vitals
58
o HR, BP, temperature (for fever)
• Systems Review
o Hydration status – volume depletion can cause electrolyte imbalance
o Signs of infection (could cause acidosis) – respiratory exam (crepitations), skin examination for pressure sores,
gastro exam (peritonitis), examine wound
• LL + UL neuro
o Characterise the ascending weakness
Investigations
• as per resuscitation
Bloods
• consider septic screen if infection suspected
Management
• Hyperkalaemia is a medical emergency, so I would consult a senior colleague or renal physician for help.
Immediate
• Hyperkalaemia
o Potassium level ≤ 6.5 mEq/L and no signs of cardiotoxicity
Discontinuation of drugs that ↑ K+ e.g. ramipril
Low K diet
Potassium binding agents either PO or rectal enema (preferred for less A/E of bowel ischaemia from
resonium) – (Calcium resonium OR Kayexalate) + lactulose to avoid constipation (not acute! Takes a
long time)
• NB: also known as calcium polystyrene sulfonate and sodium polystyrene sulfonate
respectively
o Moves into the colon releases Ca2+ and Na+ picks up K+ passed out of the gut
Potassium wasting diuretic e.g. thiazide or Loop diuretics – (not useful if patient as has CKD)
± fluids + drink water if volume depletion
o Potassium level > 6.5 mEq/L or cardiotoxicity
See resuscitation above
• Underlying cause of hyperkalaemia
o ± treat infections if present, CKD (consult renal), medications, uncontrolled DM etc.
Supportive
• IV hydration if volume depleted
• Monitoring of electrolytes

Appendix: Hyperkalaemia ECG changes

59
Hyperkalaemia in CKD
A 74 year old man has CKD stage 3 with eGFR 58. He is treated for hypertension with ramipril and amlodipine. In recent weeks he has been dizzy and
his blood pressure is 98/56 mmHg. His serum potassium is 6.8 mmol/L. How would you manage him?
Introductory statement
Provisional Diagnosis:
• This patient has 2 issues, severe hyperkalaemia and symptomatic hypotension likely secondary to ramipril (which ↑ K)
and amlodipine use, both very concerning due to risk of life-threatening cardiac arrhythmias and hypovolaemic shock.
Goals:
• Rapidly assess and manage this patient to prevent complications of hyperkalaemia e.g. life-threatening arrythmia.
• For hypotension I’d like to rule out sepsis, over-diuresis, other forms of shock and manage with fluid resus, medication
review and potentially Abx if sepsis.
DDx: other causes for hyperkalaemia include:
• See above Post-op hyperkalaemia
Resuscitation
• See above Post-op hyperkalaemia
• + fluid resus for hypotension
History
• See above Post-op hyperkalaemia +
Symptoms
• Sx of hypotension
Differentials
• Determine cause of hypotension
o recent change in dose
o hydration status (avoid dehydration)
o exclude autonomic causes of hypotension – (DM, neurodegenerative diseases including PD)
• Determine cause of hyperkalaemia Post-op hyperkalaemia
o Change in distribution Most likely due to ramipril
o ↓ excretion Particular focus as patient has CKD, last dialysis, missed sessions, urine output
Risk Factors
• CKD
o CKD symptoms (see below), who manages, medications, baseline function (e.g. Cr, GFR, K +, complications,
causes of acute worsening of CKD (infection, recent nephrotoxins)
• Cardiovascular comorbidities – history of stroke, CVD, PVD, DM, hypercholesterolaemia, smoking, diet, exercise.
Examination
• See above Post-op hyperkalaemia +
• Hydration status – volume depletion can cause electrolyte imbalance, especially in patients with CKD; will also affect
whether frusemide can be given
• Renal exam
o e.g. encephalopathy, itch, pitting oedema, pleural effusion, CKD – fistula (dialysis)
• Cardiovascular + peripheral vascular exam (comorbidities)
Investigations
• See above Post-op hyperkalaemia
Management
• See above Post-op hyperkalaemia +
• Hypotension treatment
o Potentially due to over-treatment ± lower dose or ceasing one anti-hypertensive (ideally keep the ramipril,
protects kidneys in CKD)
o Potentially due to sepsis treat accordingly
Long term
• Treat other CV factors smoking cessation, diabetes, HTN, cholesterol etc.
• Treatment of CKD see appendix

60
Appendix: Treatment of CKD
Clinical features of Chronic Kidney Disease (CKD)
include CMADHUNGER
Cardiovascular disease, metabolic acidosis, dyslipidaemia, hyperkalaemia, uraemia, Na+/H 2O, growth retardation, EPO failure,
renal osteodystrophy
Treatment
• Lifestyle
o Diet - below
o Smoking cessation
• Medications
o CVD
Glycaemic control (if diabetic) prevents worsening of proteinuria
o Metabolic acidosis
treat renal failure
± oral bicarbonate replacement
o Dyslipidaemia
statins
o Hyperkalaemia/electrolyte abnormalities
Correct hyperphosphatemia with calcium citrate (phosphate binder)
Diet – restrict K
o Uraemia
Pruritis - try capsaicin cream or cholestyramine and UV light
Uraemic coagulopathy - Desmopressin (stimulates release of vWF from endothelial cells) used if clinical
bleeding or invasive procedure
Uraemic pericarditis
Uraemic neuropathy
o Na+/H20
BP control – decreases rate of disease progression (ACEI preferred, multiple drugs potentially required)
• ACEI – dilate efferent arteriole of glomerulus. If used early on, can reduce the risk of
progression to ESKD because it slows progression of proteinuria NOTE: can cause
hyperkalaemia!
Fluid overload management e.g. pulmonary oedema
Diet – ↓ salt
o Growth retardation
paediatrics
o EPO failure
Anaemia – treat with erythropoietin injections (target haematocrit 33-36%)
o Renal osteodystrophy
NB: due to inability to activate vitamin D ↑ PTH Ca resorption from bones
long term oral calcium and vitamin D to prevent secondary hyperparathyroidism and renal
osteodystrophy
o General measures
Adjust doses of renally excreted medications
• Definitive
o Dialysis if indicated
o Transplantation is the only cure

61
Hypokalaemia 2o to Vomiting (2020)
A 44 year old woman is receiving chemotherapy for lymphoma. She has been vomiting frequently and her serum potassium is 3.0 mmol/L. How
would you manage her?
the most common electrolyte abnormality in hospitalised patients.
Introductory Statement
Provisional Diagnosis: My impression is this patient has hypokalaemia secondary to chemotherapy induced nausea and vomiting.
My main concerns are myocardial excitability resulting in arrhythmias, and dehydration, muscle paralysis and other electrolyte
abnormalities from profuse vomiting.
DDx:
• Rule out important complications of chemotherapy
o Febrile neutropenia (neutrophil count <0.5, fever) as may be an underlying cause of vomiting
o tumour lysis syndrome although I would expect her K to be elevated and thus this is less likely.
• DDx of vomiting
o GIT, neurological, iatrogenic, psychological
• Causes of hypokalaemia
o Transcellular shift
Alkalosis (hyperkalaemia leads to acidosis and vice versa), hypoosmolality
Medications Insulin (INto the cells), B2 agonists etc.
o ↑ loss
GI losses diarrhoea, vomiting, laxatives etc.
Renal losses e.g. mineral/glucocorticoid excess, thiazide/loop/osmotic diuretics (non K sparing),
hypomagnesaemia, type I and II renal tubular acidosis
Goals
• A-E assessment to ensure HD stable and address hydration status. Also includes doing an urgent ECG with view to
manage hypokalaemia urgently if worrying signs
• Manage hypokalaemia and monitor for life-threatening consequences
• Manage vomiting
ABCDE Assessment
• Call for senior help
• A/B hypokalaemia can cause diaphragmatic paralysis which may then require O2
supplementation and intubation/invasive ventilation
• C – HR. BP, ECG, 2IVCs
o ECG ST depression, T wave flattening, U waves + arrhythmias (premature
atrial complex, premature ventricular beats, AV block, paroxysmal atrial or
junctional tachycardia, VT/VF)
o Cannulas
IVC 1 VBG (acidosis can cause hypo, alkalosis can cause hyperkalaemia, electrolytes), EUC (confirm
K, other electrolytes because of vomiting), CMP/urea (hypomag can occur concurrently and need to
treat it to resolve hypoK, exclude tumour lysis syndrome – PUKE Ca), ± digoxin level (hypokalaemia can
cause digoxin toxicity as digoxin competes with K+ for Na/K ATPase pump)
• PUKE Ca Phosphate, urea, K elevated; Ca decreased
IVC 2 fluid resus + maintenance + ongoing losses
History
Symptoms ± Complications
• Hypokalaemia
o Ascending muscle paralysis/weakness ± diaphragm, muscle cramps
o Cardiac arrhythmias/palpitations (may lead to syncope)
o N/V, constipation
o Fatigue
o Polyuria
o Note: hyperkalaemia muscle paralysis/weakness, cardiac arrhythmias/palpitations (± syncope), diarrhoea
• Vomiting
o quantity, character, content (bilious, faeculent), management (prophylactic anti-emetic)
o onset (relation to chemotherapy) Acute (within 24 hours), delayed (4-5 days), anticipatory (subsequent to
first experience of chemotherapy, precipitated by chemotherapy associated sensations)
• Complications digoxin use (hypok can cause digitalis toxicity)
Differentials
• DDx for hypokalaemia
o GI losses (vomiting, diarrhoea)
62
o Medications (insulin, beta2 agonists, diuretics)
RF
• Past history of chemo induced N/V
Examination
• Vitals BP/HR for shock (hypovolaemia), temp (exclude fever for febrile neutropenia)
o If the patient is hypertensive, excessive aldosterone activity is likely.
o If the patient is normotensive, either GI or renal loss of K+ is likely
• Hydration assessment consider IDC to monitor urine output if v dehydrated and need aggressive fluid resus
Investigations
• Important see resus
• ± urinary K if cause uncertain (not in this case)
o >20mEq/L = renal loss
o <20mEq/L = extrarenal loss
Management
• Call oncologist and ideally HDU setting

Hypokalaemia
• Mild-moderate (3.0-3.4) with no symptoms/red flags oral replacement (preferred)
o KCl sustained release 1200-3600mg (16-48 mmol) PO, daily in divided doses. Initial dose dependent on deficit
and adjust dose accordingly to response
o If unable to swallow tablet effervescent formulation
• Severe (<3.0), or unable to tolerate oral (possible in this case because of vomiting) or ECG changes IV potassium
o KCl 20-40mmol/L IV mixed with an infusion
Slowly and dilute as can cause local irritation (burns) + cardiac arrhythmias (acute changes in K)
Max rate 10-20mmol/hr which raises serum K 0.1-0.2mEq/L
o ± 1% lidocaine to bag to reduce pain (K burns)
o General considerations for safety
Ideally done in HDU with ECG monitoring and serum K measured every 2 hours

Vomiting/supportive management
• This is important as the vomiting is most likely the cause for the hypokalaemia
• Anti-emetic
o dexamethasone (good for chemo induced N/V)
o ondansetron (can use wafers and good for chemotherapy induced vomiting)
o aprepitant/fosaprepitant (good for chemotherapy induced N/V prophylaxis)
o metoclopramide (antiemetic and prokinetic) blocks dopamine receptors ± serotonin receptors
• Rehydration and monitor urine output via IDC if necessary
• Discontinue any meds that might worsen hypokalaemia (e.g. diuretics)
• Correct hypomag if present otherwise contributes to ↑ K loss

Complications
• Digoxin toxicity digoxin competes for spots with potassium on the Na/K exchanger pump. Without K less
competition and increased risk of toxicity!
• Treatment
o Digoxin-specific antibody (œ) fragments
o Atropine for symptomatic bradycardia
o Class IB antiarrhythmics
o Temporary cardiac pacing

Appendix:
• K+ acts like H+: Hyperkalaemia leads to acidosis and vice versa!

63
Hyponatraemia (2020)
An 86 year old woman admitted with pneumonia becomes confused. Her serum sodium is 119mmol/L (135-145). How would you assess and
manage her?
Background on sodium
• Most important extracellular cation and plays an important role in maintaining the body's extracellular fluid volume.
• Sodium imbalances typically reflect a dilution or concentration of extracellular fluid rather than an actual loss or gain
of sodium.
o These changes in extracellular fluid volume are mainly due to an increase or decrease in ADH serum levels
(which causes the retention and loss of free water respectively).
• In certain cases, however, sodium imbalances may be the direct result of sodium loss (e.g., following diarrhea,
vomiting, or the use of antidiuretics) or excessive sodium intake.
Introductory statement
Provisional: this woman’s confusion is most likely delirium 2o to severe hyponatraemia and her infection (causing SIADH). I am
most concerned about her risk of cerebral oedema
Goals:
• Rapidly assess using an ABCDE assessment with a quick screen for the pneumonia as the cause looking at the pulse
oximetry. Resuscitate the patient following the sepsis pathway if appropriate with a view of starting empirical antibiotics
urgently or broadening cover
• Targeted H/E/I to determine aetiology and exclude other causes
• Treat the hyponatraemia urgently
• Reverse the identified causes and also treat the pneumonia
DDx: The patient’s hyponatraemia could be:
Causes of hypotonic hyponatremia (Low serum osmolality normal <280 with low serum Na)
Hypovolemic hypotonic Euvolemic hypotonic Hypervolemic hypotonic hyponatremia
hyponatremia hyponatremia
Description Low extracellular fluid volume Normal or minimal changes High extracellular fluid volume
in extracellular fluid volume
Renal - Acute or chronic renal - SIADH (mood stabilisers, - Acute or chronic renal failure with
causes failure with high urine output antiepileptics, cerebral or low urine output (i.e., failure to excrete
(polyuria) pulmonary pathology, free water)
- Diuretics malignancy SCLC)
- Mineralocorticoid deficiency - Medication use
(Addison disease) - Exercise-associated
- Recovery phase of acute tubular hyponatremia (EAH)
necrosis - Acute or chronic renal
- Cerebral salt wasting syndrome failure
(post intracranial injury damages - Glucocorticoid deficiency
sympathetic nervous stimulation (adrenal insufficiency)
of kidneys) - Severe hypothyroidism (↑
ADH)
Extrarenal - Diarrhea - Decreased salt intake (e.g., - Congestive heart failure
causes - Vomiting “tea and toast” diet) - Liver cirrhosis
- Dermal fluid loss (e.g., burns, - Water intoxication - Severe hypoproteinemia (e.g., nephrotic
sweating) (dilutional hyponatremia) syndrome)
- Third space fluid - Excessive infusion of
loss (e.g., peritonitis, ascites) hypotonic (e.g., 0.45% NaCl)
- Bleeding/haemorrhage or sodium-free isotonic IV
fluids
- Primary polydipsia
- Beer potomania
- Reset osmostat syndrome

• Other causes
o Isotonic hyponatraemia (pseudohyponatremia) Serum osmolality normal (280- 295) with low serum Na.
TURP syndrome
Pseudohyponatremia
• Asymptomatic laboratory artifact falsely indicating hyponatremia when sodium has not been
reduced or diluted

64
• Due to very high amounts of protein or lipids in the plasma (e.g., hyperlipidaemia, multiple
myeloma), which then alter the plasma water concentration
• Clinical features may include: Pancreatitis, Diabetic ketoacidosis, Obstructive jaundice, CRAB
criteria in multiple myeloma
o Hypertonic hyponatremia Serum osmolality high (>295) with low serum Na.
Caused by the presence of osmotic substances, which cause an osmotic shift of water out of the cells
and diluting [Na] in the ECF
E.g. hyperglycaemia, mannitol, sorbitol, glycerol, maltose
• Would also consider do a quick screen for other causes of delirium see Psychiatry: Delirium e.g. pneumonia
ABCDE Assessment
• Rapid ABCDE assessment and commence sepsis 6 pathway if appropriate
o Empirical Abx for CAP is ceftriaxone + azithromycin IV, consider broadening cover if already on
o Oxygen supplementation + IVF resus
• Should consult renal or endocrine regarding management and ideally in HDU or ICU environment
History
• Ideally get collateral from nursing staff e.g. how acute confusion has been
Symptoms
• Pneumonia Sx dyspnoea, chest pain, productive cough, altered mental state
• Hyponatraemia look at trend on eMR, rate at which sodium has fallen, if chronic, may want to consider other cause of
delirium since it is usually better tolerated
o Sx of hyper/hyponatraemia
Mild generally asymptomatic, potentially anorexia, N/V, headache, muscle cramps
Moderate weakness, lethargy, confusion
Severe seizures, altered consciousness, coma
• NB: if severe hyponatraemia, can cause cerebral oedema that can lead to herniation
• NB: Worst if acute hyponatremia, chronic hyponatremia tends to be better tolerated.
Differentials
• Identify contributing factors to hyponatraemia
o Hypovolemic Fluid loss (excess vomiting, diarrhoea, NG suction, third spacing), medications acting on the
kidneys (diuretics, ACEI)
o Euvolemic SIADH (see ddx), excess free water intake (e.g. IV administration of hypotonic solutions)
o Hypervolemic CCF, nephrotic syndrome, cirrhosis
o Other causes: Note other co-morbidities such as hyperlipidaemia and diabetes mellitus.
Examination
• Vitals
• Fluid status examination
o Review fluid chart – input vs. output, weight change if recent weight known
o Hypervolaemic → peripheral oedema, ↑JVP, basal crepitations
o Hypovolaemic → ↑HR, ↓BP with postural drop, decreased skin turgor and dry mucous membranes.
• Respiratory examination assess pneumonia
• Rule out other causes of confusion in particular look for signs of infection
Investigations
Bedside
• ABG (exclude sepsis, hypoxia, electrolytes), ECG, Urinalysis (glucose, UTI),
Laboratory
• Redo the serum biochemistry (EUC) plus serum osmolality
o First step: serum osmolality helps to decide the aetiology of the severe hyponatremia and is always the
Serum osm (mmol/L) = 1.86 (Na + K) + glucose + urea + 10 Approximates to 2x the sodium
o Serial required rate at which the sodium has fallen is important because a rapid fall over 24-48 hours can
cause cerebral oedema, herniation and death.
• Exclude non-hypotonic hyponatraemia lipids, proteins, BGLs (+ history of recent glucose medications e.g. mannitol)
• Urinary sodium and osmolality @same time as serum osmolality to determine renal or extra renal issue
o In patients taking diuretics, urinary sodium concentrations should be interpreted with caution. A FEUa < 12 %
can provide more diagnostic accuracy than UNa to differentiate hypovolemia from euvolemia.
• Cortisol (exclude Addison’s)
• LFT (cirrhosis can cause hyponatraemia)
• Blood cultures if febrile
Imaging
• CXR (pneumonia)

65
Management
• Since severe hyponatraemia, most likely requires ICU setting
Definitive
• Treat underlying cause
o E.g. stop the drug responsible e.g. diuretics, SSRI’s, SNRI’s, carbamazepine etc.
• Correct sodium levels
o ± Hypertonic saline (3% NaCl) specifics depends on acute vs. chronic (can be managed with more
conservative measures), associated intracranial pathology (when there is risk of ↑ brain herniation)
o ?Salt tabs (unconventional but used sometimes)
o Hypotonic
Isotonic saline

66
o Euvolemic
Fluid restriction (not for hypovolaemic) (as a guide 500 mL less than daily urine output or roughly
<800mL/day)
NB: some aetiologies this is not ideal like when hypovolaemic due to cerebral salt wasting, want to give
IV fluids
o Hypervolemic
Fluid restrictions as above
± Diuretics in CHF and cirrhosis, these can help
• Careful correction of sodium levels:
o Rapid increase in sodium levels risk of osmotic demyelination (dehydration damage)
Acute hyponatraemia ↑ 1-2mEq/L until 4-6mEq/L within 6 hours
Chronic min ↑ 4-6 mEq/L within 24 hours, max 8-12 mEq/L within 24 hours
o Rapid fall in serum sodium risk of cerebral oedema
Supportive
• Monitoring for complications
• Serial EUCs to monitor progress e.g. in 12 hours

Appendix: Hypernatremia
(Always hypertonic)
Differentials (either losing water or retaining too much Na)
Hypovolemic hypernatremia (high serum Na+ levels with decreased extracellular volume as a result of hypotonic fluid loss)
• Extrarenal cause (manifests with oliguria due to dehydration)
o Gastrointestinal loss (e.g. diarrhea, vomiting, drainage from nasogastric tubes, fistula)
o Dermal fluid loss (e.g., burns, excessive sweating)
o Third-spacing (peritonitis, ascites)
• Renal cause (leads to dehydration due to polyuria)
o Diuretics
o Osmotic diuresis (e.g., hyperglycemia, mannitol, uraemia, high-protein tube feeding, osmotic diuretics)
o Recovery (polyuric) phase of acute tubular necrosis
Euvolemic hypernatremia (high serum Na+ levels with normal or minimal changes in extracellular volume as a result of pure water
deficit)
• Extrarenal causes (manifests with oliguria due to decreased water intake)
o Lack of access to water (Altered mental status, immobilization, physically restrained patients, quadriparesis
o Impaired thirst mechanism: primary hypodipsia
o Mechanical ventilation
• Renal cause (causes increased thirst due to polyuria)
o Diabetes insipidus (central or nephrogenic)
Hypervolemic hypernatremia (high serum Na+ levels with increased extracellular volume as a result of intake of hypertonic water
or retention of sodium in excess of water)
• Extrarenal causes (initially manifests with polyuria due to fluid overload, followed by dehydration due to polyuria)
o Iatrogenic: excessive infusion of NaCl, sodium bicarbonate solutions, or hypertonic saline; haemodialysis
o Seawater consumption
• Renal causes (causes hypertension and hypokalaemia with normal urine output and no fluid overload)
o Primary hyperaldosteronism
o Cushing syndrome
Diagnostics

67
Treatment
• Correct free water deficit (Free water refers to water without electrolytes. Free water deficit = Total body water (body
weight x 0.5 for ♀/0.6 ♂) x [(serum Na/140) - 1] in <24 hours (max rate 10mEq/L/day)
• Acute IV 5% dextrose
• Chronic IV 5% dextrose or enteral H2O if stable

68
Hypercalcaemia (2020)
A 59yo female attends a diabetes clinic annually for review. You notice her serum calcium concentration has been above the upper limit of normal for
the last 3 years, currently 2.79mmol/L (NR 2.1-2.6). How would you investigate and manage this?
Introductory Statement
Provisional Diagnosis: My impression is that this patient has hypercalcaemia
DDx:
• the most common causes include
o hyperparathyroidism (primary e.g. parathyroid adenoma; secondary in vitamin D deficiency; tertiary e.g. chronic
CKD with ↓ vitamin D results in autonomous activation)
o hypercalcaemia secondary to malignancy (paraneoplastic syndrome PTHrP, osteolytic lesions/mets)
o NB: these 2 causes account for >90% of hypercalcaemia – malignancy causes will usually result in a more acute
and higher calcium compared to hyperparathyroidism
• Other causes that I would consider include
o PTH mediated Hyperparathyroidism above
o Non-PTH mediated hypercalcaemia of malignancy, granulomatous disorders (e.g. sarcoidosis)
o Other
Medications
• vitamin D intoxication (increases Ca absorption in the gut)
• esp. thiazide diuretics – increase Ca reabsorption in kidneys
Metabolic disorders (hyperthyroidism – increased osteoclast activity and bone resorption; adrenal
insufficiency – unknown mechanism)
Pseudohypercalcaemia secondary to hyperalbuminaemia in the context of dehydration as a
confounder for the results.
Goals
• Confirm that they have true hypercalcaemia by looking at a corrected calcium/ionised calcium
• History/examination to determine whether they are symptomatic`
• Investigations to determine underlying cause
• Manage appropriately by correcting the hypercalcaemia and treating the underlying cause as well as managing any
osteoporosis/osteopenia
History
• Confirm true hypocalcaemia on the blood results. If no corrected calcium, will need to do an albumin as part of her
diagnostic work-up to exclude pseudohypercalcaemia
Symptoms ± Complications
• Calcium stabilises resting membrane potential of neuron by regulating opening of Na channels for depolarisation more
calcium = less Na channels open less spontaneous depolarisation
• Hypercalcaemia (same as vit D overdose) stones (renal stones in last 6 months), bones (bone pain, arthralgia,
fractures), thrones (urinary frequency (polyuria), groans/abdo pain (3 causes: constipation, PUD, pancreatitis),
psychiatric overtones (confusion, anxiety, depression, fatigue, stupor)
o More severe muscle weakness, arrhythmias
• ± Osteoporosis history of fragility fractures
Differentials
• Systems review for malignancy
o Paraneoplastic PTHrP causes squamous cell cancers of lung, head/neck; breast, ovarian, bladder, renal;
lymphoma, leukaemia
o Osteolytic malignancy multiple myeloma, breast (mixed), renal, prostate
• Primary hyperparathyroidism
o Usually chronic hypercalcaemia in an otherwise asymptomatic patient
• Medications thiazide
Risk Factors
• Malignancy risk factors
• History of PTH or thyroid dysfunction
Examination
• Generally vague examination findings. Think decreased neuronal excitability
• Neurological examination
o Slow or absent reflexes, muscle weakness
• Bones
o Bony tenderness
• Endocrine
o Thyroid exam – may feel a parathyroid nodule
69
• Systems examination for malignancy
Investigations
Bedside
• ECG (arrhythmias, short QT interval, bradycardia, AV block)
Laboratory
• In reality probably do PTH and CMP first and then reassess based on PTH level
• PTH (see below for interpretation), PTHrP (paraneoplastic syndromes – order if PTH normal), vit D, CMP (calcium level,
can consider measuring ionised calcium directly without having to workout corrected calcium, phosphate low in primary
hyperparathyroidism), albumin (for corrected calcium), EUC (renal impairment and MM), FBC (anaemia in MM), TFTs
• Other can consider work up for MM (serum and urine electrophoresis looking for monoclonal antibodies), malignancy
screen depending on history/examination
Imaging
• Dependent on above results
• If suspect parathyroid adenoma neck U/S + functional study (Technetium 99m sestamibi scan to see the functioning
adenoma – only do if planning for surgery)
• Consider DEXA scan for osteoporosis
Management
Supportive
• Mild hypercalcaemia (<3mmol/L)
o Oral rehydration
o Avoid medications that can cause hypercalcaemia e.g. thiazide, lithium
o Avoid high calcium diets (>100mg/day)
• Moderate (3-3.5)
o Asymptomatic treat as above
o Symptomatic treat as below
• Severe (>3.5)
o IV saline infusion (often adequate) treats the dehydration caused by hypercalcaemia and prevents further
deterioration of kidney function that would ↑ Ca further
o Additional treatment
Bisphosphonate (Zoledronic acid 4mg IV OR pamidronate 60-90mg IV)
± IV calcitonin (salmon calcitonin - salcatonin) infusion if non-responsive to above
o Dialysis in very severe cases
o Cause based therapy
Excessive bone resorption: Consider IV bisphosphonate infusion
• Long term
o Same as mild hypercalcaemia + encourage exercise (reduces bone resorption) + avoid vit D overdose (i.e.
maintain a moderate vit D diet)

Definitive
• Malignancy and paraneoplastic syndromes treatment of malignancy
• Hyperparathyroidism
o Definitive treatment is surgical resection of adenoma
o If unable to undergo surgery (e.g. poor surgical candidate), conservative treatment includes
Bisphosphonates
Cinalcalcet (activates calcium sensing receptor on parathyroid gland causing PTH release inhibition)

70
Appendix
Understanding calcium and albumin (from osmosis)
• Calcium forms in the body
o In bone 99%
o Extracellular 0.99%
o Intracellular 0.01%
• Within the extracellular compartment (i.e. in plasma and interstitium) calcium exists mainly in 2 forms ionised and
protein bound (albumin)
o The ionised form is more active and actually the one that causes symptoms of hyper/hypocalcaemia
• When we take a total serum calcium, we are measuring the protein bound calcium and ionised calcium. In states of
hyperalbuminaemia (e.g. dehydration – albumin becomes concentrated) – the amount of protein bound calcium
increases but the amount of ionised calcium remains roughly the same (this is hormonally regulated). Therefore, the
total serum calcium will be increased but in reality the ionised calcium or ‘corrected calcium’ will be normal!

AMBOSS table of interpreting PTH levels

Hyperparathyroidism
• Primary hyperparathyroidism (↑ Ca and ↓ PO4)
o Adenoma (sporadic or in multiple endocrine neoplasia)
o Excess PTH increased active vitamin D (calcitriol) production via stimulation of 1-alpha-hydroxylase synthesis
in the kidneys
• Secondary hyperparathyroidism
o Renal insufficiency ↓ production of 1,25-dihydroxyvitamin D ↑ PTH
o May also be caused by vitamin D deficiency or hypocalcaemia
• Tertiary hyperparathyroidism
o Renal failure chronic secondary hyperparathyroidism autonomous activation of one or more parathyroid
glands
References
• Francis (Elizabeth notes)
• Amboss
• https://www.uptodate.com.acs.hcn.com.au/contents/diagnostic-approach-to-
hypercalcemia?search=hypercalcaemia&source=search_result&selectedTitle=1~150&usage_type=default&display_rank
=1#H9
• https://www.uptodate.com.acs.hcn.com.au/contents/clinical-manifestations-of-
hypercalcemia?search=hypercalcaemia&source=search_result&selectedTitle=3~150&usage_type=default&display_rank
=3#H9
• https://www.uptodate.com.acs.hcn.com.au/contents/treatment-of-
hypercalcemia?search=hypercalcaemia&topicRef=836&source=see_link#H1248559
• https://www.uptodate.com.acs.hcn.com.au/contents/primary-hyperparathyroidism-
management?search=hypercalcaemia&topicRef=850&source=see_link#H14

71
Hypocalcaemia post-denosumab injection (2020)
An 82y/o male complains of cramps and tingling in his fingers. He received a subcutaneous injection of denosumab by his GP for osteoporosis 7 days
prior. He has stable chronic kidney disease with an eGFR of 58ml/min and his current biochemical tests show a low plasma corrected calcium of 1.58
mmol/L (NR 2.1-2.6 mmol/L). How would you investigate and manage him?
• NB: similar case to Surgery: post-op hypocalcaemia. In that case, the differentials are different since it is post-op and stem
symptoms are different
Introductory Statement
Provisional Diagnosis: My impression is that this man has symptomatic hypocalcaemia secondary to denosumab (he is at higher
risk of this side effect given his CKD) [the nadir in serum calcium occurs ~10days after denosumab administration in patients that
get hypocalcaemia].
DDx: My differentials would include other causes of hypocalcaemia including
• Primary hypocalcaemia - high PTH vitamin D deficiency, CKD (↓ vitamin D, ↑ phosphate precipitates with Ca. NB:
hypocalcaemia can also cause hypercalcaemia if chronic), calcium loss (e.g. acute pancreatitis, tumour lysis syndrome,
sepsis), hyperphosphataemia (precipitates the Ca)
• Others – variable PTH hypomagnesemia, hyperventilation, medications (loop diuretics), hyperventilation, osteoblastic
mets
• Hypoparathyroidism - low PTH (post-surgical, autoimmune, hungry-bone syndrome, genetic, infiltration).
Goals
• Conduct a focussed clinical assessment including ECG to confirm my diagnosis and investigate life threatening sequelae
incl. arrhythmias, seizures and laryngeal spasm
• Manage with calcium replacement and endocrine consult
History
Symptoms ± Complications
• Symptoms of hypocalcaemia
o Tetany which is ↑ neuromuscular excitability (perioral and acral/extremities paraesthesias,
myalgias/cramps/spasms in any muscle)
o Seizures (generalised tonic-clonic)
o Cardiovascular (palpitations, hypotension, arrhythmias)
o Abdo (biliary colic, abdominal cramping and diarrhoea)
• Denosumab history duration of treatment, recently started/changed?, problems in the past?, did they check vitamin D
level prior (>40mmol/L of 25(OH)D is the cut-off for giving an anti-resorptive)
• ± CKD symptoms and management
Differentials
• Other medications e.g. vitamin D (vit D deficiency can cause hypocalcaemia)
• Post-surgical (parathyroidectomy), tumour lysis syndrome (active malignancy), pancreatitis (classical epigastric pain
radiating to the back, history of pancreatitis, risk factors e.g. EtOH, gallstones)
Examination and Investigations
• Identical to Surgery: post-op hypocalcaemia
• Vital signs
• Neurological exam
o muscle fasciculations, hyperreflexia, tingling, Trousseau’s sign (ipsilateral carpopedal spasm occurring several
minutes after inflation of a blood pressure cuff to pressures above the systolic blood pressure = spasmodic
contraction of the hands and feet with flexion of the wrist and metacarpophalangeal joints, extension of the
fingers, dorsiflexion of the ankles, and plantarflexion of the toes), Chvosteks sign (twitching of facial muscles
when you tap over the facial nerve approx. 2cm ventral to the ear lobe)
o should be done hourly post-op
• Cardiovascular exam
o Arrhythmias
• Resp exam
o Stridor in laryngeal spasm
Investigations
Bedside
• ECG (arrhythmias, prolonged QT interval)
Laboratory
• Check EMR for trend (if possible)
• CMP (low Ca, magnesium levels), albumin (for corrected calcium), PTH (should be low post-op), EUC (exclude renal
failure), Vit D (differentials)
o Corrected calcium every 1g/dL of albumin below normal (4.0g/dL) equates to 0.8mg/dL loss of calcium

72
• Can consider other tests e.g. lipase, glucose (acute and chronic pancreatitis)
Management
Definitive
• Endocrine consult to advise Ca replacement
• Cardiorespiratory monitoring ± HDU review
• Hypocalcaemia
o Replace can give oral or IV but in this patient, who is symptomatic you would likely give IV
o IV
Indicated if life threatening e.g. rapid and progressive decrease in [Ca], symptoms of hypocalcaemia
(tetany, seizures, arrhythmias, laryngeal spasm), ionised [Ca]<1.0mmol/L, total serum [Ca] <1.9
IV calcium gluconate 1-2g, infused over 10-20mins (do not infuse more rapidly as risk of arrhythmias,
arrest and vein irritation). Make sure cannula is in the right position
Can also use IV calcium chloride
Can induce arrythmia need monitored environment for rapid infusion
o Oral
PO calcium supplementation (e.g. calcium carbonate 1.5g BD or calcium citrate) once patient is able to
swallow
Consider activated vit D (calcitriol) supplementation

Supportive
• Manage other electrolyte abnormalities
o Hypomagnesaemia (decreases PTH secretion if low) and hypophosphataemia (ONLY if extremely low as PO4
precipitates with Ca)
• Discuss with endocrine re ongoing denosumab therapy
o Common to get some degree of hypocalcaemia with denosumab (usually asymptomatic)
o Can get symptomatic hypocalcaemia in patients with CKD + vit D deficiency
Consider activated vit D supplementation e.g. cholecalciferol
Appendix
Calcium homeostasis

Other
• Issue with denosumab is stopping it results in catch-up fractures brings into question the sequencing of the anti-
resorptives

References
• Surg hypocalcaemia case + Francis tute (15th)
• https://www.uptodate.com.acs.hcn.com.au/contents/denosumab-for-
osteoporosis?search=denosumab%20hypocalcemia&source=search_result&selectedTitle=1~150&usage_type=default&d
isplay_rank=1#H6

73
• https://www.uptodate.com.acs.hcn.com.au/contents/diagnostic-approach-to-
hypocalcemia?search=hypocalcaemia%20differentials&source=search_result&selectedTitle=1~150&usage_type=default
&display_rank=1#H9

74
Peritoneal Dialysis Peritonitis (2020)
A 52 year old woman is being treated by peritoneal dialysis for end-stage renal failure due to chronic glomerulonephritis. She presents with an acute
abdomen. How would you assess and manage her?
Introductory Statement
Provisional Diagnosis: My impression is this woman has an acute abdomen, and in the context of peritoneal dialysis, my
provisional diagnosis would be PD peritonitis. My major concerns are sepsis and that the PD has not been working well for a while
electrolyte abnormalities, and end organ dysfunction.
DDx: I would also consider other causes of an acute abdomen in particular
• SBO/LBO
• perforated viscus
• inflammation/infection (e.g. appendicitis, pyelonephritis, pancreatitis, cholecystitis)
• mesenteric ischaemia
• gynae ruptured ectopic pregnancy
Goals
• Ensuring the patient is HD stable with an ABCDE assessment, particularly nothing circulation and temperature
• Focussed hx/ex/Ix notably including examination and MCS of dialysate fluid to confirm presence of PD peritonitis and
exclude differentials
• Managing with abx and supportive care
Rapid Assessment
• Looking for signs of septic shock and HD instability
• A/B as normal
• Circulation
o ECG monitoring
o BP/HR (if shock fluid resus but be cautious of fluid overload in context of renal failure)
o Establish IV access with 2 large bore cannulas and take bloods
VBG (lactate, acidosis), FBC/CRP, EUC (renal function electrolyte abnormalities may need urgent tx
e.g. K+, also impt for abx prescribing), LFT, amylase/lipase, blood cultures x2
o IDC insertion to monitor UO
• D/E
o Signs of end organ dysfunction
o Temperature (febrile)
History
Procedure
• Type of dialysis (CAPD vs APD see appendix – influences dosing of abx), concentration of dialysate, dialysate appearance
usually, considered haemo?, does she have fistula? history of clots/catheter failure
Symptoms ± Complications
• Peritonitic symptoms SOCRATES (ask specifically about worsening when going over car bumps), fever, malaise, N/V,
bowel habit
• PD specific
o cloudy dialysate? (if yes sig likely PD peritonitis),
temporal relation of cloudy bag (if before illness PD peritonitis, if after likely secondary cause)
o potential contamination of bag/when was last dialysis
Differentials
• Rule out DDx (see other notes)
Other
• Medications/allergies particular to Abx
Examination
• Vitals looking for signs of shock
• Examine the dialysate bag colour/turbidity (turbid = infection, green = ?pseudomonas)
• Abdo
o Peritoneal site – signs of infection (erythema)
o peritonism, rule out ddx (masses, localisation, tinkling bowel sounds)
Investigations
Diagnostic
• Send dialysis bag for cell count, MCS (need to send whole bag! Usually 2L)
o Will need to be inside patient for 2 hours to get positive growth
o Common pathogens (skin stuff) in PD peritonitis are coag –ve staph (e.g. staph epidermidis), staph aureus,
strep, enterococcus, enteric gram –ve bacilli. Fungi uncommon
75
o If polymicrobial infection with gut bacteria concerned about perforation
• Swab of catheter site if possible
Laboratory
• Other bloods taken in resus
Imaging
• Can consider CXR looking for pneumoperitoneum for perforated viscus (can also get that in PD due to leaving the tube
open)
Management
• Renal/PD unit consult
Supportive
• Analgesia, fluids (careful as ESKD), DVT prophylaxis
• Patient education + assessment re aseptic technique
• Adjust PD regimen as required + consider heparin in dialysis bag if concerned re clotting (fibrin strands found or previous
clot)
• Indications for catheter removal refractory peritonitis (doesn’t respond to tx within 5 days), relapsing peritonitis,
fungal infection, line infection
Definitive
• Add abx to dialysate. Intraperitoneal route preferred as delivers local high concentrations. Bag should start to clear after
48-72hrs
o Empiric gent + cefazolin
o If concerned about MRSA gent + vanc
o If suspecting bowel perf add metronidazole
o Targeted tx once organism isolated
• Continue abx tx for at least 14 days until dialysate clears and then extra 7 once dialysate clears. For staph aureus,
enterococci and gram -ve bacilli continue for 21 days
Long-term
• Discuss L/T tx with renal and dialysis team, consider switch to HD

Appendix:
• Criteria to diagnose PD peritonitis (2/3)
o Abdo pain/cloudy dialysate
o WCC in dialysate
o Positive growth of organism
• CAPD vs. APD
o Continuous ambulatory peritoneal dialysis (CAPD; daytime multiple session) vs automated peritoneal dialysis
(APD; night only)
• Dialysis solutions
o 1.5%, 2.5%, 4.25%. Higher concentration drains more water (ultrafiltration)

76
Appendix: Haemodialysis
Can occur using 1) fistula 2) graft 3) central vascath

Fistula or graft causes ↑ BP and blood flow through the vein causing the vein to enlarge capable of delivering the amount of
blood flow necessary to provide adequate haemodialysis

77
Rhabdomyolysis (2020)
A 42 year old male recommenced weight training after a hiatus of 10 years. He presents 48 hours after his first session, which included 2 hours of
heavy weightlifting, complaining of severe pain and tenderness in his back, shoulders and biceps muscles. He describes “blood” in his urine for the last
24 hours and his urinalysis shows +++blood and +++protein. His serum Creatinine is 169 mmol/L (NR < 120 mmol/L). How would you investigate and
manage him?

Pathophysiology
• Rhabdomyolysis → release of the following substances:
o CK and serum myoglobin → pigment nephropathy → acute tubular necrosis → acute kidney injury (intrinsic)
o Potassium → cardiac arrhythmia
o Lactic acid → metabolic acidosis
Causes
• Traumatic: crush injury, direct injury
• Non-traumatic: long lie, seizures, overexertion (e.g. ultramarathon), intoxication, skeletal muscle ischaemia, infection,
medication induced, malignant hyperthermia
Introductory Statement
Provisional Diagnosis: My impression in this man is AKI secondary to rhabdomyolysis in the context of non-traumatic exertion.
DDx:
• Renal colic (however shoulder and biceps would not be sore)
• Other causes of myopathy
o Inflammatory myopathy (usually chronic however and only 1/3rd have muscle tenderness)
Polymyositis (PM): inflammatory myopathy affecting the proximal skeletal muscles
Dermatomyositis (DM): inflammatory myopathy that presents similarly to polymyositis, with the
addition of skin involvement
Inclusion body myositis (IBM): inflammatory myopathy affecting both the proximal and distal skeletal
muscles
• Muscle fatigue/strain + concurrent causes of haematuria (e.g. infection)/haemoglobinuria (haemolysis)
o Other causes of AKI
Pre-renal, renal, post-renal
o Glomerulonephritis
Goals
• Ensure the patient is haemodynamically stable via an ABCDE assessment
• Conduct a focussed H/E/I to confirm rhabdo and exclude differentials looking for the classic triad of symptoms (myalgia,
generalised weakness, darkened urine) + marked acute elevation of CK and possible AKI
• Manage with fluid and electrolyte management and if refractory, consider haemodialysis and monitoring for
complications including AKI and compartment syndrome
ABCDE Assessment
• Ensure the patient is haemodynamically stable (most worried complications including cardiac instability due to
hyperkalaemia)
History
Symptoms
• Symptoms of rhabdomyolysis
o Classic triad (50% of cases): myalgia (commonly proximal), generalised weakness, darkened urine (red to brown)
o Nonspecific symptoms: fever, nausea, vomiting, abdominal pain
• Complications
o AKI (ask about urine output), Compartment syndrome (pain disproportionate with injury, pain with passive
flexion, tense/swollen compartments, neurovascular compromise), DIC (infrequent), arrhythmias (secondary to
electrolyte disturbances)
Differentials
• AKI causes dehydration or underperfusion (e.g. CCF, APO), intra-renal, obstructive (e.g. stones, malignancy)
• Renal colic loin to groin pain
• Myopathies
o Inflammatory myopathy chronicity
o Statin induced myopathy medication review
• Screen for other causes of haematuria/haemoglobinuria
Risk Factors
• Screen for other causes of rhabdomyolysis (above)
Other
• Complete rest of medical history including PMHx, SHx

78
o Past history of rhabdomyolysis

Examination
• Vitals
• MSK
o Inspection (skin changes due to aetiology potentially), palpation (muscle tenderness and swelling), move
(muscle weakness)
o Compartment syndrome see history for examination findings
Investigations
Bedside
• Urinalysis (macroscopically dark, +++blood due to myoglobinuria, +++proteins due to myoglobin + other proteins released
by damaged myocytes) Urine MCS (exclude RBC to differentiate the + blood on urinalysis), ECG
o Remember that stem only says ‘dark urine’ which can be either blood or myoglobin – hence need for urine MCS
Laboratory
• FBC + CRP (Hb and infective cause), CK/EUC + eGFR/ CMP (CK usually >5x, other electrolyte abnormalities such as ↑ K
and ↑ PO4, ↓ Ca, ↑ lactate and MA)
o Doing a serial CK is the most important investigation for this patient

Management
Supportive
o Continual monitoring of CK
o Comfort care and symptom management
o Graded exercise program once discharged

Definitive
• Eliminate the cause (not so relevant for this patient)
o Crush injury management, toxins etc.
• Early and generous IV fluid administration
o Monitor fluid output (maintain urine output at 1-2ml/kg/hr) + electrolyte levels
o 1-2L per hour for the first couple of hours reduces concentration of myoglobin which is nephrotoxic
Duration of fluids until trajectory of CK is down-trending, doesn’t have to be back to normal
o Other (not great evidence for the following)
± diuretics (frusemide) if urine output not being maintained – probably shouldn’t use as first line
therapy
• Unable to find robust evidence suggesting use of diuretics
Might also consider alkalising urine (pH 6-6.5) with sodium bicarbonate to prevent precipitation of CK
• Evidence is scarce on this
• Correct electrolyte and metabolic levels as necessary
o Hyperkalaemia (see Hyperkalaemia in CKD),
o Although hypocalcaemia calcium phosphate precipitation means calcium should not be administrated unless
very necessary e.g. cardiac stabilisation or significant symptoms
o Hyperuricaemia allopurinol
• Consider haemodialysis if worsening kidney function cannot be managed with above
o May require for weeks in some cases
• Monitor for complications and manage accordingly
o Compartment syndrome referral to orthopaedics for fasciotomy

References
• Francis tute
• https://www.uptodate.com.acs.hcn.com.au/contents/clinical-manifestations-and-diagnosis-of-
rhabdomyolysis?search=rhabdomyolysis&source=search_result&selectedTitle=1~150&usage_type=default&display_rank
=1#H25330880
• https://www.uptodate.com.acs.hcn.com.au/contents/prevention-and-treatment-of-heme-pigment-induced-acute-
kidney-
injury?search=rhabdomyolysis&source=search_result&selectedTitle=3~150&usage_type=default&display_rank=3#H253
99006
• https://tgldcdp.tg.org.au.acs.hcn.com.au/viewTopic?topicfile=toxicology-general-
approach&guidelineName=Toxicology%20and%20Wilderness#toc_d1e1608

79
UTI (Recurrent)
A 23 year old woman presents with a history of recurrent UTIs. How would you assess and manage her?
Definition
• Recurrent UTIs are defined as ≥ 2 infections in 6 months or ≥ 3 in 1 year according to UpToDate
• See Primary Care: UTI
Introductory Statement
DDx: Causes of recurrent UTI
• Anatomic abnormality of urinary tract resulting in obstruction, stasis or reflux
o Causes of reflux - vesicoureteral reflux
o Causes of stasis - hypotonic bladder (more common in older population), voiding dysfunction (e.g. multiple
sclerosis)
o Intraluminal obstruction - calculi, neoplasms
o Intramural obstruction - ureteral stenosis, urethral strictures, prostatic obstruction (common in old men; not
this patient)
o Extramural obstruction - neoplasm, fibrosis
• Pregnancy
• Behavioural factors - sexually active women especially with spermicide use (local irritation of the urethral meatus;
common cause in young women), wiping back to front
• Iatrogenic - indwelling bladder catheter, nephrostomy tube, ureteral stenting, radiation cystitis
• Functional abnormality (less likely in young woman) - cystocele, incontinence
• Immunocompromised state (less likely if isolate recurrent UTIs only)
Goals
• First conduct a targeted H/E/I to exclude an active urinary infection and septic shock
• I would then determine if there was an underlying cause for recurrence, reverse it if possible and give prophylactic
antibiotics.
History
• Same as Primary Care: UTI
o Focus on RF for recurrence using DDx
Examination
• Same as Primary Care: UTI
Investigations
• Same as Primary Care: UTI
o Would have a lower threshold for imaging including Renal and bladder U/S, KUB CT, cystoscopy, intravenous
pyelogram (X-ray with contrast)
Management
Management of current UTI
• Same as Primary Care: UTI

Management of recurrence
• Education
o Toilet hygiene wipe from urethra to anus
o Adequate hydration
o Urination post sexual intercourse
o Finish course of antibiotics
• Non-pharmacological
o Intravaginal oestrogen
o Consider methenamine (inconsistent evidence)
• Pharmacological
o Antibiotics prophylaxis post-coital OR at onset of symptoms OR continuous

80
Neurology
Status Epilepticus/Seizure 2o to Viral Encephalitis (2020)
A 24 year old woman is admitted to hospital with suspected viral encephalitis and is commenced on high-dose acyclovir. On the first night of her
hospitalisation she has a generalised seizure which is persisting for more than 10 minutes. How would you manage her?
Status epilepticus criteria
• ≥ 5 min of continuous seizures (technical definition is >30mins but since it is rare for seizures to last for more than 5
mins, this is the criteria and also when active treatment is commenced)
o Can be convulsive or non-convulsive/absence (variable motor signs)
• OR ≥ 2 seizures with consciousness not being fully regained in the interictal period
NB: Most normal seizures are brief (1-3mins) and do not require drug treatment
Introductory Statement
Provisional: Given that this is an unremitting seizure lasting ≥ 5 mins continuous, this woman is in status epilepticus which is a
medical emergency, likely 2o to her viral encephalitis and a potential A/E of acyclovir. I am most concerned about anoxia, cerebral
ischemia, cerebral oedema and rhabdomyolysis.
Goals: My priorities would be
• 1) MET call, ABCDE assessment with simultaneous resuscitation to terminate the seizure 1) benzodiazepines 2)
antiepileptic drug 3) general anaesthetic + transfer
o Also important to cease the acyclovir
• 3) Manage long term by reversing causes and possible anti-epileptics as per neurology’s recommendations.
DDx: Causes of seizures could be
• Epilepsy (unprovoked)
• Provoked seizures
o Extracranial
Infections Untreated meningitis (i.e. got the diagnosis wrong), Brain abscess
Drugs/medications
• A/E e.g. acyclovir
• Intoxications cocaine, lithium, lidocaine, theophylline, metal poisoning (e.g. mercury, lead),
carbon monoxide poisoning
• Withdrawal alcohol, benzodiazepines, barbiturates
Metabolic and electrolyte disturbances – hypoxia, hyponatremia, hypoglycaemia, hypocalcaemia,
kidney failure (uraemia), thyroid storm, hyperthermia
o Neurological stroke, TIA (less likely as young), hypoxic injury, trauma, tumours (1° or 2°), haemorrhage,
hypertensive encephalopathy
o Miscellaneous
Pseudoseizures – not true seizures but are psychiatric in origin; are often difficult to distinguish from
true seizures without an EEG
Eclampsia – only definitive treatment is delivery. Also give Mg infusion
Resuscitation
• Met call + senior help. Patient needs to go to ICU and needs to be treated for status epilepticus
• Start stopwatch
• A: as usual +
o All convulsions have the capacity to involve the respiratory musculature and upper airways
o Ensure no aspiration by clearing secretions
o If trismus (locked jaw) then nasopharyngeal airway
o most status epilepticus patients need to be intubated
• B: as usual +
o O2 usually given after >5 and active treatment commenced
• C: as usual (assess, monitor e.g. ECG, vascular access)
o Insert IV cannulas, send bloods
VBG (fast) EUC, CMP, BSL, LFT, ± toxicology screen, ± anti-epileptic levels, ± ABG, ± septic screen
o [1] ± (>5mins) short acting-benzo IV over 2 minutes e.g. midazolam (0.15-0.2mg/kg up to 10mg), diazepam (0.1-
0.25mg/kg up to 10mg) or clonazepam (0.25-0.5mg up to 1mg). If IV access unable to be obtained quickly, can
give midazolam IM/buccally/intranasal
Continue with rest of resuscitation and if after 5 minutes patient still seizing, repeat dose of
benzodiazepine
• D: as usual +
o Quick neurological exam
o First aid for seizure – stay with patient, time seizure, protect the head and put in the left lateral position if
worried about vomiting

81
o If BSL<3 give 10% glucose IV at 2mL/kg as a bolus then an infusion after, recheck in 5 mins
o ± Setup EEG monitoring important for post-seizure to exclude non-convulsive status epilepticus
• E: as usual
o Look for causes of provoked seizure, e.g. rash for meningococcal meningitis
• Other immediate management
o Cease the acyclovir in the short term (A/E potentially causing the seizure)
Continued management
• [1] Benzodiazepine
• Wait 5 minutes and consider repeated dose for another cycle
• Treat potential reversible causes e.g. glucose, thiamine, naloxone ± order relevant investigations e.g. LP
• [2] Long-acting anti-epileptic
o phenytoin sodium 20mg/kg IV (no faster than 25-50mg/minute – slower for children and elderly)
o sodium valproate 40mg/kg IV up to 3000mg over 5-10 minutes
o phenobarbitone 20mg/kg IV (no faster than 1mg/kg/min) only for children
o NB: Levetiracetam can be used in children but supporting evidence is limited
o NB: Phenytoin and phenobarbitone (ICU ideally due to resp depressions after benzo) especially need ECG and BP
monitoring due to risk of arrhythmias, hypotension etc.
• [3] (after 15 mins usually) Rapid sequence induction with general anaesthetic (thiopentone or propofol) ± airway
management (if not already stable airway) and transfer to ICU
o Other 3rd line agents recommended by RCH for children instead of rapid sequence induction midazolam
infusion, ketamine, pyridoxine

If seizures stop
• Management in the post-ictal phase
o Recovery position. Post-ictal non-responsiveness typically lasts 10-20 minutes. Close monitoring of vitals
required, treat ABCs.
• Check EEG monitoring in case there is a continuation of non-convulsive seizure and to track response to medications

History
• Will need a collateral
• Since it is first seizure, important to classify the seizure and work out the causes. If patient was on anti-epileptics would
want to know more about triggers and compliance
Symptoms ± Complications
• Detailed chronological history of events before, during and after the seizure
o During: Classify seizure based on whether motor (focality of limb or eye movement) OR non-motor onset (e.g.
aura), aware/impaired awareness.
o After: Ask about complications of seizure e.g. trauma, post-ictal phase/hemiparesis
• (this case specifically) Ask about symptoms of viral encephalitis (headache, fever, aches, confusion)
Differentials
• Look for underlying cause of a seizure
o Symptoms of meningism (headache, photophobia, neck stiffness)
o Medication/drug history – use of alcohol, benzodiazepines prior to hospital (withdrawal) or acute intoxication
o Previous seizures and anti-seizure medication (management plan if in place)
o Neurological comorbidity (e.g. VP shunt, structural brain abnormality) symptoms of ↑ ICP
o Renal failure (hypertensive encephalopathy)
o Endocrinopathy (electrolyte disturbance)
o Trauma or non-accidental injury
Other
• PMHx history of seizures
Examination
• check vitals + GCS again; until return to baseline status
• Neurological Exam
o Post epileptic seizure unlikely to find any abnormalities
o Provoked seizure Look for focal neurological deficits, signs of raised ICP (papilledema, Cushing’s reflex: HTN,
bradycardia, irregular breathing), signs of meningitis
• Complications (according to eTG)
o Complications of acute seizures – vomit, aspiration, trauma (e.g. posterior shoulder dislocation)
o Complications of prolonged seizures – CNS injury (focal neurological deficits), non-cardiogenic pulmonary
oedema (listen to lungs), rhabdomyolysis (myalgia, weakness, darkened urine), acidosis and kidney failure
(check urine output + do bloods), hyperthermia

82
Investigations
• As per resuscitation
Imaging
• CT brain to investigate if first seizure
• ± LP
Management
Short Term
• Treat of any of the complications mentioned above
• Monitoring with regular GCS and vital checks
Long Term
• Identify cause of seizures Reverse these
o Meningitis – dexamethasone + antibiotics
o HSV encephalitis - acyclovir
o Delirium tremens
o Drug intoxication
• Consider adding anti-epileptics relevant to classification of seizure (generally started after 2 or more unprovoked but
started if patient is in status epilepticus)
o Focal (partial) carbamazepine
o Generalised or unknown sodium valproate
o Specific childhood epilepsy syndromes consult therapeutic guidelines
• Counselling
o Meds Compliance, A/E, Drug interactions, missed dose regimen
o ADL Avoiding triggers, dangerous situations (swimming, climbing), driving restriction, wary of mood changes
o Emergency When to call an ambulance + Educating family members on seizure first aid (protect the head,
± emergency medication e.g. buccal midazolam)
o Prognosis immediate initiation of pharmacotherapy ↓ short term risk of recurrent but not long-term

83
TIA (2020)
A 70 year old man with type II diabetes presents complaining of an episode of facial asymmetry and weakness which lasted 5 minutes. How would
you assess and manage him?
Introductory Statement
Provisional Diagnosis: given the transient facial droop my provisional diagnosis is a (transient ischaemic attack) defined as
transient period of neurologic dysfunction caused by focal brain, spinal cord, retinal ischemia, without acute infarction
DDx: Other causes to consider (although unlikely given the temporary nature):
• Central
o Neurological – Ischaemia (stroke), hemiplegic migraine, hypertensive encephalopathy, head trauma, post-ictal
Todd’s paresis
o Metabolic – hyponatraemia, hypoglycaemia
o Toxins – alcohol, benzodiazepines
o Malignant – CNS tumour/abscess
• Peripheral Facial nerve palsy
o Idiopathic bell’s palsy
o Secondary infection (HZV, HIV, HSV), DM, sarcoidosis, GBS, other causes
Goals: My aims would be to
• assess the patient via a rapid assessment
• conduct a targeted H/E/I for a stroke workup including finding source of TIA e.g. carotid doppler (likely atherosclerosis of
carotid arteries embolus causing TIA)
• I would then like to manage the pt via non-pharmacological + pharmacological means to treat the cause + aggressively
manage stroke risk factors.
o eTG The risk of a stroke after a TIA is about 10% at 2 weeks, and half these events occur within 48 hours
ABCDE Assessment
Assuming haemodynamically stable
History
• Although it was temporary, focus is to exclude a stroke!
Symptoms ± Complications
• Sudden onset of focal neurological deficits. FAST: facial drooping, arm weakness, speech problems, time
• Characterise the facial asymmetry
o Timing, upper facial and/or lower (helps to differentiate central vs. peripheral)
o Peripheral sensory disturbances, dry mouth, ocular features (Lagophthalmos – can’t close eyes, ↓
lacrimation, ectropion), synkinetic involuntary movements (facial spasms while closing the eyes)
Risk Factors/Causes
• Stroke RF
o Demographics age, gender
o Medical previous stroke/TIA, AF, antiphospholipid syndrome, carotid stenosis
o Lifestyle smoking, HTN, cholesterol, diabetes
• In particular, need to know severity and if the risk factors are treated
Differentials
• malignancy (constitutional), migraine (headache, aura), TP (seizure), peripheral (viral exposure and infective symptoms +
differentiating symptoms above)
Other
• ABCD2 score Estimates risk of subsequent stroke after TIA (2,7,90 day risk) – estimated using the
o Age, BP, Clinical features of TIA, Duration of symptoms, Diabetes hx
Examination
• Recheck vitals, in particular BP
• Neurological examination
o looking for focal neurological deficits (facial paresis, arm drift/weakness, dysarthria/aphasia)
• Cardiovascular exam
o Causes of stroke (bruits, murmur, AF)
Investigations
Diagnostic
• Stroke package: CT brain non contrast, CT perfusion, CT angiography
o MRI since more sensitive?
• Determine cause of stroke – echo (TTE), carotid duplex US
Bedside
• ECG (AF, old MI), BSL (exclude hypo), urinalysis, ABG/VBG

84
Laboratory
• FBC (platelets), EUC (Na), coagulation studies (PT, INR, APTT), fasting BSL, HbA1C, fasting lipids
Other
• If patient is young, consider thrombophilia screen and vasculitis screen
o Thrombophilia screen – protein C, protein S, antiphospholipid antibodies (lupus anticoagulant, anti-β2-
glycoprotein, anti-cardiolipin), factor V Leiden
o Vasculitis screen – ANA, ENA, rheumatoid factor
Management
• Consult neurology registrar.
• Usually patient admitted for work-up of stroke for safety and timelines, although most investigations can be done as an
out-patient

Non-pharmacological
• Monitoring – 4hrly obs + Holter monitor (exclude AF), NBM until swallowing checked

Pharmacological
• antiplatelet (aspirin 300mg PO/NG/rectal 1st day, then reduce to 100mg OD indefinitely)
o Very important due to ↑ risk in in following weeks + importance of secondary preventions
o for atherosclerosis– after haemorrhage excluded
o Can consider DAPT
• ± Anticoagulation (warfarin or NOAC) - indicated in patients with AF (as per AF case; assess CHADSVASC and HASBLED).
Not indicated in patients without AF or another source of cardiogenic embolism

Surgical
• ± Urgent carotid endarterectomy indicated if ipsilateral carotid stenosis > 70% (benefit greatest within 2 weeks of TIA or
mild stroke)

Long-term
• Prevention – SNAP (dietician/exercise physiologist)
• Co-morbidities
o BP
o Cholesterol - statin to reduce risk of further ischaemic stroke. Started regardless of cholesterol levels as there is
presumed atherosclerosis
o Diabetes

85
Chronic Daily Headache (2020)
A 36 year old woman has been experiencing almost daily headaches for the past three months. She is otherwise asymptomatic. How would you
assess and manage her?
Epidemiology
• Tension-type headache: 40-80% of cases
• Migraine: 10% of cases
NB: ↑ ICP headaches are normally worse in morning and worsened by factors that ↑ ICP e.g. coughing, sneezing, recumbency
or exertion
Introductory Statement
Provisional Diagnosis: Given this patient with a chronic daily headache (defined as ≥ 15 days per month for ≥ 4 hours a day). This is
most likely due to tension headaches or potentially migraines considering the patient is female and their prevalence.
DDx: include:
• Primary headaches
o tension headache, migraine, trigeminal autonomic cephalgias e.g. cluster headache (multiple episodes/day,
more severe and short-lived, trigeminal neuralgia)
• Secondary headaches
o Drugs/medications - medication overuse headache (pre-existing headache disorder), medication headache,
substance induced withdrawal
o Cranial pathology - rhinosinusitis or abscess, SAH, space occupying lesion or raised ICP - e.g. tumour,
hydrocephalus
o Other less likely causes
Traumatic brain injury
Infection (less likely > 3 months and asymptomatic) - meningitis, encephalitis
Rheumatological disease (e.g. giant cell arteritis) usually never occurs <50
Ophthalmological - acute glaucoma
• NB: Can also split up into structural and non-structural useful for neurosurg ddx
Goals:
• perform a targeted history and examination to classify the headache, determine any contributing factors and exclude
life-threatening differentials
• manage the patient appropriately with analgesia ± anti-emetics or prophylactic migraine medications
History
Symptoms ± Complications
• Sx of headaches Pain history (SOCRATES)
o age of onset, triggers (stress, financial/relationship issues, exams, work), relieving factors, timing/frequency,
associated symptoms (aura, photophobia, phonophobia, tearing)
Differentials
Primary headaches
Tension headache Migraine headache Cluster headache
Sex ♂<♀ ♂<♀ ♂ > ♀ (3:1)
Duration 30 minutes to a couple of 4–72 hours 30–180 minutes
days Short, recurring attacks
Frequency Occasionally to daily Occasionally to several 1–3 episodes ever 24 hours
Episodic or chronic times a month Usually occur in a cyclical pattern
(clusters)
Localization Holocephalic or bifrontal 60% are unilateral. Mostly unilateral
Character Dull, nonpulsating, band- Pulsating, Localized to the periorbital and/or
like or vice-like pain boring/hammering pain temporal region
Intensity Mild to moderate Moderate to severe Severe, agonizing pain
Additional symptoms - No autonomic - Nausea, vomiting - Ipsilateral autonomic
symptoms (vomiting, - Hyperacusis symptoms: conjunctival injection
nausea, phonophobia, - Photophobia and/or lacrimation, rhinorrhoea and
or photophobia) - Phonophobia nasal congestion
- Tightness in - Preceding aura - Partial Horner
the posterior neck - Prodrome syndrome: ptosis and miosis, but
muscles no anhidrosis
- Pericranial tenderness

86
Triggers/exacerbating - Stress - Stress Alcohol
factors - Lack of sleep, fatigue - Fluctuation in hormone
- Routine activities (e.g., levels: oral
climbing stairs) do not contraceptives,
exacerbate symptoms. menstruation
- Certain types of food
(e.g., those containing
tyramines or nitrates such
as processed meat,
chocolate, cheese)
- Exacerbated by exertion
- Improved with
sleeping/darkness
The typical migraine headache is “POUND”: pulsatile, one-day duration, unilateral, nausea, disabling intensity.

Red flags for life-threatening causes causes


• Space occupying lesions causing raised ICP - early morning headache, worse when lying down, nausea/vomiting, changes
in cognition or personality, seizures
• Constitutional features of cancer - weight loss, malaise, night sweats
• Infection - fever, immunocompromised state (e.g. HIV), meningism (light sensitivity)
• Cerebrovascular event - neurological deficits (weakness, loss of sensation, facial droop), sudden onset thunderclap
headache (SAH)
• Trauma - head injury
• Giant cell arteritis – unilateral, visual disturbance, jaw claudication
• SNOOP: S (systemic, secondary), N (neurologic), O (onset is sudden/abrupt), O (older >50yo), P (positional, prior headache
now different in quality, papilloedema)
Other causes of headaches
• Sinusitis - feeling of fullness behind eyes or over cheeks/forehead
• Medication history
o Medication overuse headache – used ≥ 15 days/month. (ergotamine, triptans, opioids, simple analgesics) in a
patient with a pre-existing headache disorder
Patients taking analgesia for other reasons (for example, arthritis) are only at risk of developing
medication overuse headache if they also have a history of headaches.
o withdrawal from other substances, drugs which cause headaches as a side effect (CCBs, nitrates, dipyridamole)
o COCP (risk of stroke), anticoagulants (risk of bleeds)
Risk Factors/Causes
• Psychiatric considerations mental health issues, depression, stress, somatic symptom disorder, effect on ADLs
Examination
• Vital signs (BP, pulse, GCS)
• Full neurological exam (CN, UL, LL)
o exclude focal neurological deficits, including cranial nerve palsies.
o Palpate temporal arteries + assess for jaw claudication (giant cell arteritis)
o Fundoscopy – exclude raised ICP which is seen as papilledema (indicates mass or infection).
Signs of papilloedema include venous engorgement (early), loss of venous pulsation, haemorrhages
over or adjacent to the optic disc, blurring of optic margins, elevation of the optic disc, Paton’s lines.
o Rule out less likely causes (stroke, meningitis)
• Neck and scalp tenderness: palpate temporal arteries and assess jaw claudication (giant cell arteritis)
Investigations
• chronic headache is a clinical diagnosis. Investigations should only be performed if a secondary cause is suspected.
Imaging
• ± contrast CT or MRI Brain make a decision based on red flag symptoms (e.g. malignancy)
Management
• The management of chronic daily headache depends on the specific headache type

Non-pharmacological
• Encourage headache diary (useful for assessment) – document headache episodes per day, associated symptoms,
perceived triggers
• Avoid triggers stress, alcohol, cheese, chocolate, lack of sleep, COCP
• Behavioural therapy Rest in a quiet, dark room and avoid movement or any activity including reading and television
(particularly for migraines), good sleep hygiene, routine meal schedules

87
• Physical therapy regular exercise, cold packs over the forehead or back of the skull (targeting the supraorbital and
greater occipital nerves), hot packs over the neck and shoulders (targeting the innervation of the scalp), neck stretches
and self-mobilisation
• Cognitive behavioural therapy – coping skills, relaxation techniques, gentle exercise in between attacks, correction of
posture. 

• Modification of SNAP risk factors and discourage over-reliance on caffeine.

Definitive
• Medication overuse headache
o patient education, withdrawal of offending medications, bridge therapy aimed at symptomatic relief during
medication withdrawal, relapse prevention.
• Chronic migraine – Limit the use of acute headache medications to prevent medication overuse headache (<10-15
days/month depending on medication)
o Acute treatment
[1] Non-opioid analgesic (paracetamol or NSAID) ± anti-emetic (metoclopramide preferred) take
early cause they may get N/V which will limit absorption)
[2] triptan (serotonin receptor agonist) e.g. eletriptan, naratriptan, rizatriptan, sumatriptan,
zolmitriptan (needs to be limited to <10 days/month due to risk of MOH; MOA = 5HT agonists)
[3] for status migrainosus (>72hrs) ± chlorpromazine or dexamethasone or dihydroergotamine
• Ergotamines + triptans - Do not use within 24h of each other. both act as 5-HT agonists, and
cause excessive vasoconstriction.
o Prevention - required if headaches occurring >2-4 days/ month
[1] - TCA (amitriptyline, nortriptyline) OR candesartan OR propranolol OR pizotifen OR sodium
valproate OR topiramate OR verapamil
• Look at eTG table which helps to choose between these based on co-morbidity
[2] If first drug not effective after reasonable trial (max tolerated dose for 8-12 weeks), different drug
• Tension-type headache
o Acute treatment
[1] - Non-opioid analgesic (paracetamol or NSAID)
o Prevention - given if increasing frequency
[1] TCA (amitriptyline, nortriptyline). Continue for 6 months if effective, then consider trial of
withdrawing therapy.
[2] mirtazapine, venlafaxine.
• Cluster headache
o Acute treatment
[1] PO or subcut triptan (e.g. sumatriptan, rizatriptan, zolmitriptan) ± high flow O2 15L/min
[2] For severe refractory cluster headache, indomethacin or dihydroergotamine (if the patient has not
had a triptan in the preceding 24 hours)
o Prevention
[1] = verapamil. Can also use glucocorticoids, lithium, topiramate.
Appendix

88
Back Pain (2020)
A 40 year old man develops acute back pain after heavy lifting three weeks ago. He now presents with persistent low back pain, sciatica and
weakness of ankle dorsiflexion. He asks about whether he should have spinal manipulation. What would you advise him?

When considering radiculopathies, it is end-point of pain that is the point of dermatomal distribution. Therefore,
sciatica can represent a radiculopathy despite pain covering multiple dermatomes
Pathophysiology of disc herniation
• The intervertebral disc consists of a dense outer ring (annulus fibrosus) and a gelatinous core (nucleus pulposus).
• Compression, tension, shear, and torque stresses on the spinal disc degenerative changes (e.g., dehydration,
annular tear) disc protrusion or herniation and surrounding inflammatory response adjacent nerve root
impingement sensorimotor deficits in affected nerve root
o It is this inflammatory response that settles in 6 weeks and why conservative management is the mainstay
early
• Usually, the affected nerve root is the one below the level of disc herniation (e.g., L4-L5 disc herniation leads to L5
radiculopathy).
• Disc protrusion/herniation almost always occurs posterolaterally because the longitudinal posterior ligament is
thinner than the longitudinal anterior ligament.
Introductory Statement
Provisional Diagnosis: My provisional diagnosis is of acute lumbosacral radiculopathy due to a L5/S1 lumbar disc herniation or
spondylosis most likely affecting nerve root L5
DDx: I want to exclude:
• Pathologies
o Traumatic/Degenerative (Non-specific muscular/ligamentous strain, intervertebral disc herniation,
spondylolisthesis, arthritis, vertebral fracture)
o Spinal column or cord infection (e.g. epidural abscess, discitis, vertebral osteomyelitis)
o Neoplasm (commonly metastatic, multiple myeloma, primary e.g. chondrosarcoma)
o Demyelinating diseases e.g. transverse myelitis, multiple sclerosis or GBS
• These can cause:
o Radiculopathy (commonly from disc herniation)
o Spinal cord compression causing incomplete spinal cord syndrome
o Conus medullaris / cauda equina compression (spinal emergency) due to fracture, trauma, malignancy etc.
NB: Cauda equina is comparatively asymmetric (due to mix of fibres that can be affected) and late onset
of bladder/bowel involvement)
• Visceral organ disease (AAA, PUD, pancreatitis, renal disease)
Goals
• Assuming the patient were clinically stable, I would proceed with a focused history and examination to determine
aetiology as well understand what ‘spinal manipulation’ means to the patient
• Educate the patient about their condition and about what treatments are recommended based on their symptoms
History
Symptoms ± Complications
• Mechanism of injury
• Sx of back pain SOCRATES pain, progression of symptoms, acute (<12 weeks) vs chronic (>12 weeks)
o Worse with activity or rest (inflammatory should be rigorously investigated)
• Complications
o See red flags below
Differentials
• Red flags
o Spinal cord compression low back pain radiating to buttocks and legs, neurogenic claudication (aggravated
by spinal extension including standing, walking downhill, or even at rest. Relieved by spinal flexion including
sitting, leaning forward, walking uphill)
o Cauda equina compression progressive bilateral, other lower limb motor/sensory deficits especially saddle
area (changes to sensation when on toilet), urinary (overflow) or faecal incontinence
o Malignancy nocturnal pain, constitutional symptoms, past or current history of malignancy
o Others significant trauma, fever, worse when recumbent
o Can use TUNAFISH acronym: trauma, unexplained weight loss, neurological deficits, ↑ age, fever,
immunocompromised, steroids, hx of HIV/TB/Cancer
• Risk factors for fragility fracture (corticosteroids, malignancy etc)
Risk Factors/Causes
• Previous spinal injuries
Other
89
• Psychosocial history focus on occupation, level of physical activity, ADLs, RFs for chronic pain (fear of activity,
compensation issues, low mood etc.)
Examination
• Spine MSK exam
o Look, feel, move
inspection of anatomic abnormalities (lordosis, scoliosis, kyphosis)
palpation for vertebral or soft tissue tenderness non-specific finding
decreased ROM
• LL Neuro exam of
o Tone, motor, reflexes, sensation
check for any motor/sensory deficits; Dermatomal/myotomal distribution? Bilateral?
• See appendix (knee strength and reflexes L4, great toe and foot dorsiflexion L5, foot plantar
flexion and ankle reflexes S1)
Straight leg raise test
• Straight leg raise test (Lasegue's sign): straight leg of patient is raised → ↑ pain in the
ipsilateral leg with radiation to the motor or sensory area of the affected nerve root more
sensitive
• Crossed straight leg raise test: opposite straight leg of patient is raised → increased pain in
affected leg with radiation into the motor/sensory area of the affected nerve root.
Walk on heels (check dorsiflexion) more specific
• Abdo exam
o ± DRE and anal reflexes for effect on sphincter if worried about spinal cord/cauda equina compression
o Check for intra-abdominal causes (e.g. AAA)
Investigations
• Often not needed
Diagnostic
• Radiological findings are poorly correlated with symptoms in patients with low back pain. Many commonly reported
abnormalities on lumbar spine imaging are physiological and not related to the experience of pain.
• ± Lumbar X-ray
o Can be useful if fracture, degenerative changes, malignancy, spondylolisthesis
• ± MRI spine
o early imaging (before 6 to 8 weeks) is unnecessary unless there are severe or progressive neurological deficits
in this case definitely worthwhile considering
• ± nerve conduction studies, EMG and sensorimotor testing for differentials
Management
• Start of with conservative management (non-specific back pain below) for 6 weeks unless:
o Failure of conservative treatment
o So bad they can’t function
o New onset motor symptoms within 24-hour period

Non-specific back pain


Non-pharmacological
• Counselling
o Explanation of nature of pain, the lack of need for investigations, prognosis, clarification of patient
misconceptions and unhelpful coping strategies that may contribute to poor prognosis
o Most patients have an acute course that improves rapidly (usually within 4 weeks) with no ongoing limitation in
daily activities may be painful but will recover
o Remaining active rather than bed rest including encouragement to return to work ASAP
• Refer to physio for strengthening of core muscles and graded exercise programs
• Education re safe lifting, positions of ease, physical activity to reduce nerve root impingement and avoid exacerbation
• Weight loss if they have an elevated BMI
• Hot packs and manual therapies such as massage
• ± CBT help development of adaptive coping behaviours
• Relaxation techniques
• ± TENS evidence is limited
• NB: spinal manipulation in uncomplicated back pain (not this man) has limited benefit – should be done by a licenced and
experienced professional
o Avoid neck manipulations!
Pharmacological
• simple analgesia ([1] NSAIDs, [2] paracetamol);

90
• avoid opioids (if use, limited, close follow up) see NPS module: opioid analgesics in chronic non-cancer pain
o Options: controlled release morphine OR controlled release oxycodone OR transdermal buprenorphine
o Avoid codeine since it short acting
Follow-up
• Any new/worsening (fever, numbness, bladder/bowel, sexual dysfunction) re-present to ED
• Persisting pain 6 weeks re-assess for serious pathologies and consider more invasive management (below)

Symptomatic lumbar disc herniation


• similar conservative management to non-specific lower back pain above
• ± epidural glucocorticoids or short course PO course re-assess after 2 weeks, may provide modest benefit but no
benefit beyond 3 months
• ± TCAs, gabapentin, pregabalin could be useful for neuropathic pain (evidence of lacking)
• ± Surgery to remove the herniated disc / spinal decompression e.g. microdiscectomy
o for those with persistent or progressive motor deficit, failure of conservative treatment, severe radicular pain
o useful in subset of patients when there is a high correlation between pathology and symptoms
o can be done minimally invasively
o surgery results in more rapid recovery than conservative management; however, long-term outcomes are
similar

Symptomatic spinal cord stenosis


• spinal emergency!!!
• Conservative treatments above are of unproven benefit
• Surgical consultation

91
Parkinson’s Disease
A 60 year old man has had Parkinson’s disease for six years and has been taking levodopa/benserazide for the last four years. He complains that the
control of his symptoms is not as good as it was. How would you manage him?
Introductory Statement
Provisional Diagnosis: My impression is that this man is suffering from motor fluctuations due to the long term use of Levodopa,
as the underlying symptoms of Parkinsonism can manifest in the “off” periods, occurs in about 50% of long-term users. As
Parkinson’s progresses, dopaminergic neurons die and there is an increasing dependence of the basal ganglia on plasma
Levodopa.
DDx: I would also like to consider
• Poor medication compliance
• Levodopa induced dyskinesia
• Infections exacerbating PD
Goals: My aims in this case are:
• Perform a targeted hx and examination to qualify his worsening symptoms
• Implement conservative, pharmacological and even potentially surgical additions to his current treatment to target the
motor, non-motor and cognitive disturbances
History
Symptoms ± Complications
• Motor (usually unilateral onset with asymmetrical course) resting tremor (subsides with voluntary movements but ↑
with stress e.g. pill-rolling tremor), rigidity, akinesia/bradykinesia (slow movements/difficulty initiating movements),
postural instability (late sign), shuffling gait, dysarthria, micrographia, dystonia
• Non-motor autonomic dysfunction (postural hypotension falls, bladder dysfunction esp. overactive bladder,
constipation, hyper/hyposexuality), psychiatric (depression, anxiety, psychosis, cognitive decline)
• Timing when symptoms occur, do they occur with levodopa dose?, are they predictable in onset? (wearing off
phenomena occurs ≤4hrs after dose)
• Complications
o falls
Differentials
• Quick screen for infective symptoms
• Complications of levodopa drug induced dyskinesia (chorea) or dystonia e.g. in the context of levodopa
Risk Factors/Causes
• Medication hx what meds - levodopa + others?, compliance?, neurologist?, other meds/interactions?
Other
• Psychosocial hx ADLs and supports?, falls risk?, cognitive status?
• Collateral history cognitive function/mental state, functional status etc
Examination
• Vitals postural BP
• Neuro
o Observation flat affect/masked facies
o Gait postural instability, shuffling gait, reduced arm swing
o UL and LL resting tremor, cogwheel rigidity
o Akinesia/bradykinesia finger tapping, hand grip, pronation/supination, toe tap
• Systems review infection
• Cognitive screen MMSE/RUDAS
• Psych screen depression questionnaire if necessary
Investigations
• Septic screen if think infection otherwise no investigations necessary
Management
• Conservative and pharm methods to treat motor, non-motor and psychiatric manifestations
Conservative
• Low protein diet protein significantly affects levodopa absorption
• Smaller, more frequent, regular dosing times
• Patient education, support groups, exercise/physio
Pharmacological
• Modify existing drug regimen
o (as above) or change
o sustained release delivery
• Add on a 2nd dopaminergic agent
92
o COMT inhibitors (e.g. entacapine) ↓ metabolization of dopamine
o dopamine agonists [non-ergot] (e.g. bromocriptine)
o MAOI-B inhibitors (e.g. selegeline) ↓ metabolization of dopamine
o NMDA antagonist (e.g. Amantadine) antagonise at the glutamate NMDA receptor dopaminergic effect
especially useful to ↓ levodopa-induced dyskinesias

± Surgical
• Deep brain stimulation

Management of non-motor side effects


• Orthostatic hypotension non-pharm (avoid heat, slow to get up), pharm (fludrocortisone)
• Bladder dysfunction fluid restriction, anticholinergics
• Sexual dysfunction sildenafil
• Psychiatric AP for psychosis, consider anti-depressants for depression/anxiety
• Cognitive decline GP and MDT management, pharmacist for med compliance, can consider portable programmable
pump for administration of meds

Appendix:
• Parkinsonism Parkinsonism is a syndrome featuring bradykinesia and either resting tremor or rigidity (or both).
• Secondary parkinsonism: parkinsonism with secondary causes such as medication, intoxication, and head trauma
• Parkinson disease: parkinsonism for which no cause can be determined (idiopathic)
o Unilateral onset is characteristic of Parkinson disease!

Appendix:
• Benserazide
o DOPA carboxylase inhibitor (same as carbidopa) Decreased peripheral conversion of L-DOPA to dopamine
which can’t cross BBB → reduced peripheral side effects of L-DOPA (e.g., orthostatic hypotension, nausea and
vomiting) and allows dopamine to ↑ concentration in the brain

Treatment algorithm
• Patients over age of 65 or multimorbid patients
o [1] levodopa + decarboxylase inhibitor (carbidopa or benserazide)
• Patients under the age of 65 with no significant comorbidities
o [1] (Non-ergot dopamine agonists ± L-dope/carbidopa) OR MAO-B inhibitors OR COMT inhibitors
o Alternatives
Ergot dopamine agonists
NMDA antagonists
Anticholinergics/muscarinic antagonists (useful in <65 with tremor as main complaint)

93
Recurring dizziness with autonomic failure (2020)
A 72-year-old female presents with recurring dizziness. She has postural hypotension, thought to be due to autonomic failure and is taking multiple
medications for other conditions. How would you approach this situation?
Introductory Statement
Provisional Diagnosis: My impression for this patient is likely recurring dizziness due to postural hypotension secondary to
autonomic failure. My major concern is regarding her falls risk.
DDx: However, I would like to rule out other causes of postural hypotension including:
• Hypovolemia
• Medications (antihypertensives, diuretics, alcohol)
• Cardiac pathology e.g. aortic stenosis, arrythmia, anaemia, CCF
• Autonomic failure
o Neuropathies (Diabetes, amyloidosis, Sjogren’s)
o Neurodegenerative diseases e.g. Parkinson’s and Parkinson’s plus syndromes (including dementia with Lewy
bodies)
• Other conditions that may be described as “dizziness” such as causes of vertigo (BPPV, Meniere’s, posterior stroke)
Goals
• Targeted history, examination and investigations to exclude other causes of dizziness and stratify her risk of falls
• Provide appropriate supportive and definitive management to address her dizziness and minimise her risk of falls
History
Symptoms ± Complications
• General onset + duration of symptoms
• Characterise what the patient means by dizziness
o Presyncope usually lasts for seconds and minutes. Described by patients as “nearly blacking out/fainting” or
light-headedness, associated with diaphoresis, nausea and visual blurring
o Vertigo most commonly described as a spinning sensation (“ground spinning out beneath me”)
Never lasts for more than a few weeks!
o Disequilibrium sense of imbalance that occurs primarily when walking
Can be caused by a variety of things including peripheral neuropathy, MSK disorders disturbing gait,
vestibular disorders and cerebellar disorders
• Timing of dizziness and any triggering events e.g. postural, positional
• Associated symptoms palpitations, chest pain, nausea and vomiting, weakness
Risk Factors/Causes
• Autonomic failure when diagnosed, causes, symptoms (gastroparesis, anhidrosis, urinary/bowel issues)
• History of cardiac disease aortic stenosis, arrythmias
• Volume depletion dehydration, diuretic use, recent vomiting/diarrhoea
• Medications e.g. vasodilators, b-blockers, diuretics, sedatives
Other
• Falls history
o Number of falls in the last 12 months
o Complications/injuries following falls
o Other risk factors DAME (drugs and alcohol, age-related changes, medical, environment)
• Complete the rest of the medical history
o Big focus on psychosocial history to assess home environment, supports and services and baseline ADLs
Examination
• General appearance + vitals
• Postural BP looking for drop in systolic BP by >20mmHg or diastolic BP >10mmHg
o Lack of reflex tachycardia in response to drop in BP is a clue that autonomic failure is the cause (presence does
not exclude autonomic failure)
• Cardiac exam
o Pulse (rate + rhythm), murmurs (valvular pathology), carotid bruits
• Neurological exam
o Gait
o Cerebellar exam
o If vertigo HiNTS (distinguish between central and peripheral causes of vertigo) and Dix-Hallpike if positional
vertigo
o Anhidrosis (rub pen along patient’s forehead and it won’t have resistance)
Investigations
Bedside
• ECG (arrythmias)
94
Laboratory
• FBC (anaemia), EUC, LFT
Imaging
• Consider CT brain if suspecting central cause and Echo if concerned about cardiac pathology
Management
• Manage this patient in an MDT setting with input from geriatrics, neurology and allied health to minimise symptoms and
manage falls risk

Pharmacological
• Medications to increase blood pressure fludrocortisone (glucocorticoid) 0.1mg daily, midodrine (alpha-1 receptor
agonist, not registered for use in Australia)
• Optimise co-morbidities e.g. cause for her autonomic failure

Non-pharmacological
• Review and rationalise medications to minimise polypharmacy + reduce use of medications that can exacerbate
orthostatic hypotension + increase risk of falls e.g. opioids, sedatives (consult pharmacist)
• To reduce orthostatic hypotension
o Avoid extreme heat, alcohol, large meals, straining and standing up rapidly
Educate about proper ‘getting up’ technique e.g. go from lying to sitting, wait a few minutes and then
try stand if feel comfortable
o Increase sodium and water intake
o Sleep with head of the bed raised
• Manage falls risk
o Address DAME risk factors
o May require referral to physiotherapy for gait and balance and strength training + referral to OT to optimise
home environment
o Consider services based on ADLs
o Consider whether she is safe at home or whether she needs increased care

References
• https://www.uptodate.com/contents/mechanisms-causes-and-evaluation-of-orthostatic-hypotension
• https://www.uptodate.com/contents/approach-to-the-patient-with-dizziness
• https://www.uptodate.com/contents/falls-prevention-in-community-dwelling-older-persons
• Francis Tute

95
Rheumatology
Statin Induced Myopathy (2020)
A 63 year old woman complains of aching shoulders and hips for the past month. She has not noticed any weakness but the pain is affecting her
activities. Apart from the pain she feels tired but her appetite is good and her weight is stable. Her past health is unremarkable expect for
hypercholesterolemia which was diagnosed three months ago and is being treated with simvastatin. How would you assess and manage her?
Introductory Statement
Provisional Diagnosis: Given her muscle pain after introduction of a statin, my provisional diagnosis is statin-induced myopathy,
which affects 10% of patients treated by statins. It is more common with statins that are extensively metabolised by CYP450 3A4
(simvastatin carries greatest risk, atorvastatin)
DDx: Include:
• Drug-induced myopathies (steroids, PPIs, CCB, CYP4503A4) – includes myalgia (muscle discomfort, aches, sore, stiffness;
normal CK), myopathy (muscle weakness ± elevated CK), myositis (muscle inflammation), myonecrosis, rhabdomyolysis
(myonecrosis, myoglobinuria, AKI)
• Autoimmune
o Polymyalgia rheumatica (treat with short course steroids) associated with GCA and more pain than weakness
with no raised CK
Joint involvement correlates well with polymyalgia rheumatica
o Fibromyalgia
o Inflammatory myopathies (commonly pelvic and shoulder girdle) – polymyositis, dermatomyositis (associated
with malignancies has polymyositis clinical features with extra cutaneous manifestations)
• Endocrine
o Hypothyroidism
• Arthritis of shoulders and hips – mechanical (e.g. OA) or inflammatory (e.g. RA)
Goals:
• assess the patient through a targeted hx + examination and consider investigations e.g. serum CK/urinary myoglobin, and
then
• manage the patients by predominantly pharmacological means e.g. temporarily ceasing statins + correcting reversible
causes.
History
Symptoms ± Complications
• Symptoms of statin-induced myopathy
o classically proximal symmetric muscle weakness (can be painful and thighs>arms) and/or functional deficits
(e.g. inability to raise arms), time course of symptoms (usually weeks to months)
• Complications
o Rhabdomyolysis (rare) classic triad = muscle pain, weakness, dark urine; others include malaise, fever,
tachycardia, N/V, abdominal pain
Risk Factors/Causes
• Statin – when commenced + reason for taking statin (e.g. primary prevention, hypercholesterolaemia). In particular, I
would want to know her LDL levels
• Risk factors for developing statin induced myopathy
o 1) Medical conditions which increase risk of myopathy (hypothyroidism, CKD, obstructive liver disease, vitamin
D deficiency)
o 2) drug interactions: a) taking a statin extensively metabolised by CYP450 3A4 (e.g. atorvastatin, lovastatin,
simvastatin); b) taking drugs that interferes with CYP3A4 (e.g. clarithromycin, verapamil, diltiazem, anti-HIV
drugs).
Differentials
• Rule out DDx – onset of pain, SOCRATES
o Polymyalgia rheumatica inflammatory rheumatic condition. Aching and morning stiffness in shoulders, hip
girdle and neck. Need to consider giant cell temporal arteritis (there is an association)
GCA temporal pain, vision disorders, jaw claudication
Less weakness than myopathy and more pain
o Fibromyalgia chronic widespread MSK pain, fatigue, cognitive disturbance, psychiatric and somatic problems
o Inflammatory myopathies proximal muscle weakness more than pain
Examination
• MSK exam to identify affected joints, tenderness, restricted ROM, inflammatory signs e.g. synovitis, boggy swelling
• Neurological exam - looking for power (proximal myopathies)
Investigations
• Clinical diagnosis based on temporal association for both onset with initiation of statin therapy and resolution with statin
withdrawal.
96
Laboratory
• Creatine kinase (spectrum of statin-induced muscle will have different), EUC + urinary myoglobin (rhabdomyolysis)
o CK will not be raised in polymyalgia rheumatica
• Risk factors for statin induced myopathy vitamin D + TFTs (can present similarly) + LFT
• Lipid profile + other cardiovascular workup
• HMG-CoA reductase antibody (highly sensitive)
Imaging
• ± MRI thighs
Special
• Biopsy, EMG
Management
Pharmacological
• Cease statin
o Wait for symptoms to resolve and CK to return to baseline
• Correct reversible causes
o vitamin D deficiency, hypothyroidism, drug interactions, AKI, biliary obstruction (consider alternatives)
NOTE: if patient had rhabdomyolysis and there is no obvious reversible aetiology statin therapy cannot
be resumed
• If persistent/progressing after statin stopped consider immunosuppression therapy
o Oral corticosteroid (e.g. prednisolone) + immunosuppressant (e.g. azathioprine or methotrexate + folic acid)
• Resume statin when symptoms have resolved
o Consider switching to pravastatin or fluvastatin (lower muscle toxicity than other statins)
Earlier generation statins usually lower myopathy inducing
o Consider alternate day or less frequent dosing
• NOTE: For most primary prevention patients who do not tolerate statins, non-statin lipid-lowering medications are not
indicated as they have not been shown to affect outcomes such as risk of death or major cardiovascular event (e.g. MI,
stroke)
o Potential alternative interventions include lifestyle modification and, in higher- risk patients, antiplatelet
therapy.
o However, non-statin lipid lowering medications may be indicated in patients with very high LDL-C levels and
high CVD risk
Options – cholesterol absorption inhibitors (e.g. ezetimibe) or fibrates

97
Rheumatoid arthritis
A 33 year old woman presents with a three month history of pain in her hands, wrists and feet. The pain is worse in the morning and associated with
stiffness in the joints lasting for about an hour. On examination there is swelling of the wrist and MCP joints with limited movement in the joints. How
would you assess and manage her?
Arthritis/Arthralgia overview
• Pathology: Mechanical or inflammatory
• Type: Large or small joint
• Symmetry: Symmetrical or asymmetrical
• Joints involved: Monoarthritis vs oligoarthritis vs polyarthritis
• Location: axial vs peripheral (or both)
• Occurrence: additive vs migratory vs all at once
Introductory Statement
Provisional Diagnosis: My impression is rheumatoid arthritis given the symmetrical polyarthritis of the small joints (always spares
DIPs) and her risk factors of being a female.
DDx:
• seronegative spondyloarthropathies (psoriatic arthritis – DIP involvement is distinctive feature, reactive arthritis,
ankylosing spondylitis, IBD related)
• other CT diseases (SLE, scleroderma/CREST, sjogren’s, mixed)
• septic arthritis (viral Parvovirus B19, HIV, HBV, HCV, Lyme disease) (bacterial chlamydia, gonorrhoea)
• crystal arthropathies (gout peripheral small joints in lower extremities especially but can be large joints/pseudogout
mostly large joints frequently knee or wrist)
• non-inflammatory (OA)
Goals: My aims in this case are
• Focussed hx/ex/Ix to confirm diagnosis of RA and rule out differentials as well as determine baseline function
• Appropriate management with simple analgesia, steroids and DMARDs with the goals of reducing symptoms, preventing
disease progression and maintaining function
History
Symptoms ± Complications
• RA
o Timeframe RA >6 weeks, shorter may indicate septic arthritis
o Joints involved
symmetrical polyarthritis of small joints with DIP sparing (feet MTPs, cervical C1, C2 leading to
instability, ± others wrists)
if atlanto-axial stability pain/stiffness of neck, neurological deficits, deformity
o morning stiffness (>30 mins compared to OA which has evening stiffness),
o extra-articular manifestations
constitutional (fever, weight loss, malaise etc), resp (SOB in pleural effusions, nodules, pulmonary
fibrosis), cardio (chest pain in pericarditis, myocarditis), skin (nodules), msk (carpal tunnel syndrome,
bursitis), eyes (episcleritis, scleritis, 2o sjogren’s), hands (Raynaud’s)
Differentials
• OA (weight bearing joints, insidious onset, brief morning stiffness, minimal swelling), ankylosing spondylitis (back pain,
usually male), psoriatic arthritis (rash), reactive arthritis (recent infection esp. STIs), viral arthritis (recent infection, RF for
infection with above virus’), SLE (malar rash), gout (RF for gout, hx of gout), IBD arthritis (history of IBD symptoms)
Risk Factors/Causes
• RA Fam hx, personal hx of inflammatory arthritis/chronic inflammatory conditions, smoking, obesity
• DDx as above
Other
• Psychosocial hx and functioning particularly important! Need to know baseline function so can monitor disease
progression/response to treatment
Examination
• Vitals low grade fever
• MSK full rheumatological examination of hands and other joints
o RA
Soft, boggy swelling that is symmetrical, excludes DIPs, classical hand deformities (boutonniere, swan
neck, ulnar deviation of MCPs, ‘z’ thumb)
Atlanto-axial and subaxial subluxation (c-spine instability, neurological impingement)
Complications ruptured tendons, ruptured bakers cyst, carpal tunnel (tinels and phalens)
Extra-articular manifestations + exclude differentials

98
• cardio (pericarditis, murmurs in valvular incompetence), lungs (pleurisy, effusion), skin
(nodules, rash for ddx), eyes (scleritis, episcleritis)
o Exclude differentials (joint involvement, amount of swelling, presence of crepitus)
Investigations
• No single test to rule in or out RA
Most important diagnostically
• RF and anti-CCP (↑ specificity), CRP/ESR (raised), FBC (anaemia of chronic disease)
o High RF is a poor prognostic factor and more likely to have extra-articular manifestations
• X-ray of affected joints, typical changes in RA;
o Periarticular osteopenia, symmetrical joint space narrowing, marginal joint erosion, soft tissue swelling, classic
deformities
o Can differentiate from OA which has asymmetrical joint space narrowing, osteophytes, subchondral sclerosis,
subchondral cyst
Bedside
• ECG (can get conduction abnormalities)
Laboratory
• ANA (exclude ddx, can be +ve in 1/3 of RA patients consider ENAs if +ve to test for SLE + others)
• ± serology (esp. parvovirus B19)
• Starting on immunosuppressive therapy EUC/LFT (baseline), test for infections that can be exacerbated (e.g. TB, HIV),
B-HCG
Imaging
• consider MRI or US of joints
Special
• ± synovial aspirate if worried about septic arthritis
Management
• Aim to control symptoms, prevent disease progression, maintain function/QoL
• Referral to rheumatology + MDT management e.g. OT
Non-pharm
• Exercise!, stop smoking!, optimise other risk factors (SNAP)
• Optimise comorbidities
• Functional help at home if required
• Fish oil
• Ensure vaccinations up-to-date (both RA and treatment ↑ risk of infection)
• Counsel has to be on 2 contraceptive methods, but if she wants to get pregnant, use contraception
Pharm
• Induction
o NSAIDs (pain) ± corticosteroids (PO or IM) + DMARD (methotrexate usually first line)
Consider coxibs (celecoxib) COX-2 selective and useful for RA
Wean off steroids
Counsel when starting methotrexate (see Methotrexate Monitoring (2020)
• Use folic acid with methotrexate
• Maintenance
o [1] NSAIDs PRN and DMARD (essential, reduced mortality and morbidity)
o [2] Consider other DMARDs (e.g. leflunomide, hydroxychloroquine etc which are not standard
immunosuppressants) if methotrexate CI
o [3] + Add 2nd DMARD specifically biologics (e.g. anti-TNF, IL-1 inhibitors which are immunosuppressives) if not
controlled on 1xDMARDs
o [4] Consider surgery in v severe cases (arthroscopic synovectomy, joint replacement)

Follow-up
• Patient and clinician assessment of symptoms + functional status (DAS-28 score)
• Physical examination to determine ROM and function
• Lab markers (CRP/ESR, Hb)
• Imaging x-ray every 2 years

99
Appendix: variants of RA
• Felty syndrome - triad of RA, neutropenia, splenomegaly
• Juvenile RA - begins before 18 years of age. Extra-articular manifestations may predominate (Still disease) or arthritis
may predominate
• Caplan syndrome - RA associated with pneumoconiosis

Appendix: Joint involvement

Appendix: OA vs. RA Imaging


Osteoarthritis Rheumatoid
1. Irregular joint space narrowing 1. Symmetrical joint space narrowing
Destruction of articular cartilage by pannus and ↑ production
of cartilage MMP destroying extracellular matrix
2. Osteophytes 2. Soft tissue swelling
Due to inflammation of the joint and surrounding tissue leading
to an inflammatory joint effusion and synovial hypertrophy
3. Subchondral sclerosis 3. Periarticular osteopaenia/osteoporosis
Bone resorption formation due to systemic inflammation
(activating osteoclasts and increasing bone resorption),
immobility and glucocorticoid therapy
4. Subchondral cysts 4. Marginal Joint Erosion
Local bone necrosis due to joint stress Due to activation of osteoclasts (2’ to interaction between
inflammatory cytokines and RANK)
5. Classic Deformities
Due to destruction of tendons, ligaments and joint capsule

marginal

Appendix: Extra-articular manifestations

100
101
Methotrexate Monitoring (2020)
A 38 year old woman has been treated for rheumatoid arthritis for 2 months with diclofenac. Her arthritis is poorly controlled and recent X-rays of her
hands show erosive changes in the metacarpophalangeal and proximal phalangeal joints. She is to be commenced on methotrexate. How would you
monitor her therapeutic response and possible drug toxicity?
For general assessment and management of Rheumatoid arthritis see Rheumatoid Arthritis case.
Introductory statement
Methotrexate is a DMARD which is a folic acid antagonist. It has cytotoxic and immunosuppressant effects.
• Interferes with folic acid metabolism through inhibition of dihydrofolate reductase (DHFR) ↓ nucleotide synthesis
↓ DNA synthesis ↓ clonal expansion of T and B cell
• Much lower dose in RA compared to chemotherapy
Goals:
• Assess severity of RA (baseline)
• exclude contraindications to methotrexate prior to administration
• Counsel on indications and adverse effects of methotrexate
• Appropriately investigate and monitor patient’s response and side effects

Note: the following is done prior to starting therapy as well as during follow-up minus a few components
History
Symptoms
• Symptoms of RA see Rheumatoid arthritis case ± compare to baseline for therapeutic response
o Assess severity of disease
Consider DAS 28 (disease activity score 28) – number of swollen and tender joints (28 joints), ESR/CRP,
‘global assessment of health’ made visually on a 10cm line by the patient
Duration of early morning stiffness
o Extra-articular manifestations of RA including systemic signs fever, anorexia, malaise, weight loss, and symptoms
of cardioresp disease
• ADLs
• A/E of medications if already commenced
Other
• Assess contraindications to methotrexate
o Pregnancy or breastfeeding (need to stop 3-6 months prior to trying for a baby) - ensure that the patient is on
adequate contraception
o Alcoholism, alcoholic liver disease or other chronic liver disease
o Immunodeficiency
o Pre-existing haematological issues - bone marrow hypoplasia, leukopenia, thrombocytopenia, significant
anaemia
o Latent infection – Hep B/C, HIV, TB
• Medications drug interactions that may alter concentration
• Vaccination status ideally >1 month before commencement however live vaccines such as MMR, polio, shingles and
yellow fever are no longer CI in low dose methotrexate users but suitability decided on an individual basis
Examination
Regular intervals depending on disease activity and severity
• MSK
o examine all joints + look for extra-articular manifestations of RA see Rheumatoid arthritis case
Examined joints include the hands, wrists, elbows, shoulders, and knees and feet
The joints should be evaluated for the presence of swelling, tenderness, loss of motion, and deformity.
o assess changes in previously affected joints or the appearance of inflammation in previously uninvolved joints.
Investigations
Prior to starting, resuming, or significantly increasing therapy with nonbiologic or biologic disease-modifying antirheumatic drugs
(DMARDs) the following is ideally done according to UpToDate:
Bloods
• Initially + follow ups FBC (anaemia), EUC, LFT, B12/folate, ESR
o every 2-4 weeks for first 3 months or after increasing the dose, then every 8-12 weeks for months 3-6, then
every 12 weeks after that
• Initial workup ONLY screen for hep B and hep C (HBV core antibody, HBV surface antigen), Quantiferon gold (screen
for latent TB), HIV
o Note: reason for screening is ↑ risk of infection
Imaging

102
• repeat X-rays of hands/wrists/feet every 2 years for patients in remission or with low disease activity. Look for signs of
progression (periarticular osteopenia, irregular joint space narrowing, bone erosions, soft tissue swelling)
• initial workup ONLY ± CXR (TB)
Special tests
• ± pulmonary function tests (methotrexate can cause restrictive lung disease), if using hydroxychloroquine
ophthalmologic screening
Management
• MTX (IM or oral) + Folic acid
o Advise patient that much of the joint damage that ultimately leads to disability occurs early in the disease, so
early treatment with DMARDs is critical and can taper off corticosteroids + NSAIDs. 1st Line DMARD =
Methotrexate (MTX)
o Dosage – taken once per week (initially 10mg once weekly, then adjust dose gradually to optimal response while
avoiding toxicity (max dose 25mg once weekly) PO or Subcutaneously
+ folic acid 5-10mg per week PO (usually 12 hours after dose) up to even once daily except MTX day
reduces hematologic, gastrointestinal, and hepatic adverse events
Alternative = folinic acid
o Slow onset of action (initial improvement seen in 4-6 weeks, latest within 12 weeks)
o Advise patients of side effects – related to cytotoxicity
Common side effects – GIT (nausea, vomiting, mouth ulcers, GI upset), abnormal LFTs, increased
susceptibility to infection, mild alopecia
Uncommon side effects – bone marrow suppression (pancytopenia) ± macrocytic anaemia (due to
folate antagonism), liver cirrhosis, renal failure, interstitial pneumonitis which may lead to pulmonary
fibrosis
o Drug education ‘patient information leaflets’
• Changing drug regimen
o Most patients should receive a DMARD as soon as possible after diagnosis
o If treatment resistance, consider combination treatment with other drugs (see Rheumatoid Arthritis)
o If GI side effects occur, consider taking a smaller dose bi-weekly

103
SLE and Pleuritic Chest Pain (2020)
A 36 year old woman with SLE is hospitalised for investigation of declining renal function. She complains of pleuritic retrosternal chest pain. How
would you assess her?
Introductory Statement
Provisional Diagnosis: lupus nephritis (can be nephritic or nephrotic) and likely pericarditis secondary to an acute flare of SLE.
DDx: My differentials would include other causes of chest pain:
• (Life-threatening) clot – AMI (SLE 5x risk), PE (esp. antiphospholipid)
• (Most common) infection – pneumonia + pleurisy (considering immunosuppressive therapy)
• Other SLE complications – myocarditis, pneumonitis, diffuse alveolar haemorrhage
• Other chest pain (non-SLE) – cardiac tamponade, pneumothorax
Goals: My aims in this case are
• Conduct a rapid A-E assessment + immediate assessment/management to ensure HD stable
• Take a targeted H/E/I to confirm provisional diagnoses, exclude differentials and monitor SLE activity to direct
appropriate management
ABCDE Assessment
• Breathing – RR, O2
o Concerns – diffuse alveolar haemorrhage
• Circulation – HR/BP, ECG
o Concerns – MI, PE
• Exposure – temp
o Concerns – infection
History
Symptoms ± Complications
• Lupus nephritis foamy urine ± haematuria, oedema
o Red flags (requiring dialysis) – AEIOU esp. fluid overload, hyperkalaemia, uraemic encephalopathy
• Pericarditis worse on inspiration/leaning back, alleviated by sitting up/leaning forward
• SLE MDSOAPBRAIN to gauge severity see appendix
Differentials
• AMI – radiating down L arm, diaphoresis
• PE – haemoptysis, unilateral calf swelling, clotting RF
• Infection – fever/constitutional, purulent sputum
Risk Factors/Causes
• SLE flare ceasing medications/compliance, infection, UV exposure, stress (physical/mental), hx of flares
• RF for ddx
Other
• SLE hx when diagnosed?, medications?, rheumatologist/nephrologist, what manifestations do you usually have?
(MDSOAPBRAIN), antiphospholipid testing? (increases risk of PE), ever diagnosed with lupus nephritis?
Examination
• Vitals
• Cardio
o Pericardial friction rub (best heard at left sternal edge)
o Becks triad for tamponade (increased JVP, muffled heart sounds, hypotension)
• Resp
o Decreased vocal resonance (effusion), focal consolidation (pneumonia)
o Inspect calves for signs of DVT
• SLE (consider)
o Neuro + MSK + derm + lymph nodes
• Renal AKI – look for signs of uraemia (encephalopathy, bleeding)
Investigations
Diagnostic – lupus nephritis
• Urine
o UA – (proteinuria, haematuria)
o MCS – protein, red cells ± dysmorphia
± formal 24h protein (if >120 on formal urine study)
o If red cells >10 on MCS or protein >500mg on 24h renal biopsy for lupus nephritis
• EUC – renal function

Diagnostic – chest pain (these would all be done to R/O ddx for SLE pts because of their increased risk)
104
• Bedside
o ECG – pericarditis (widespread ST elevation + PR depression), exclude MI
• Lab
o Troponin – exclude MI
o FBC – exclude pneumonia
• Imaging –
o Echo – pericarditis/pericardial effusion
o CXR – exclude pneumonia
o CTPA or V/Q – exclude PE

Monitoring lupus activity


• FBC – anaemia, acytopenia, lymphopenia, platelets
• Flare ↑ESR (CRP often normal), ↑anti-dsDNA (correlate with disease activity), ↓C3/C4
• CRP
o If SLE only normal/mildly elevated (B cell mediated)
o Significantly raised more likely infectious ddx
Management
• Consult rheumatology (always for SLE)
Definitive

Mild symptoms, no vital organs Severe symptoms, no vital organs Organ damage
affected affected
Basic therapy Hydroxychloroquine [Plaquenil] which is not a standard immunosuppressive
If flaring on long term meds check compliance
Induction Low-dose , short- Medium-dose , oral glucocorticoids High-dose, IV glucocorticoids
therapy term, oral glucocorticoids
Immunosuppressive
agents (e.g., azathioprine, mycophenolate, cyclophosphamide)

• Lupus nephritis
o Ensure to add 2nd agent: immunosuppressive (mycophenolate or cyclophosphamide)
o Corticosteroid (directed by renal biopsy)
Whilst waiting for renal biopsy oral prednisone 1mg/kg or 2 days (pulse) IV methylprednisolone (if
severe e.g. class III-IV lupus nephritis)
• Serositis (pericarditis and pleuritis)
o colchicine (for serositis) – over weeks/months based on pt progress
• Other cause for chest pain (still very important in this case)
o PE antiphospholipid syndrome screen (even if already done; can change)
+ve heparin lifelong warfarin
-ve heparin for 6mo with repeat APS screen @3mo
o Infection abx as directed
can be used to treat pericarditis if mild, however in this case the steroids should cover it
Supportive
• Cease nephrotoxic agents, fluids, pain management, consider VTE prophylaxis
• Pt counselling + education
o Avoid UV exposure
o Optimise comorbidities
o Smoking cessation

Appendix: SLE Diagnosis


• 4/11 diagnostic criteria either serially or simultaneously in order to be diagnosed with SLE. Of the 4, you need at least 1
clinical and 1 lab criteria
4 Lab Criteria 7 Clinical Criteria
• Blood/haematological features (haemolytic anaemia • Mucocutaneous signs (>70% cases)
with reticulocytosis, leukopenia, lymphopenia, o Malar rash with sparing of the nasolabial
thrombocytopenia due to autoantibodies to folds
RBCs/WBCs) o Discoid rash
• Renal disorder (proteinuria >0.5g/day, cellular casts) o Oral/nasopharyngeal ulcers (painless)
• ANA +ve (most sensitive but not specific i.e. not o Photosensitivity
diagnostic on its own) • Arthritis (non-erosive involving >2 joints i.e.
polyarthritis)
105
• Immunological features (presence of other auto- • Serositis (pleuritis or pericarditis)
antibodies e.g. +ve anti-dsDNA, anti-Smith, • Neurologic disorders (seizures or psychosis)
antiphospholipid abs)

Appendix: Other investigations for SLE


• ENA other specific auto-antibodies
• Antiphospholipid antibodies e.g. lupus anticoagulant, anticardiolipin, anti-B2-glycoprotein1
• Imaging of affected joints
Appendix: Treating lupus nephritis
• Depends on type of lupus nephritis
o I-II do not treat
o III-IV treat with immunosuppression
1st line: mycophenolate or cyclophosphamide
o V-VI (sclerosed/scarred; too late!) transplant or dialysis

106
Palliative Care
End of Life Care (2020)
A 78 year old woman has with terminal cancer and is expected to die within a few days and is documented as not for resuscitation. She is
semiconscious but seems distressed, tachypnoeic and agitated and is making gurgling noises which are distressing to the family. How would you
manage this situation?
Introductory Statement
Provisional: These clinical findings of progressive deterioration are consistent with the terminal phase of illness characterised by
terminal secretions, which are noisy, rattled breathing caused by an accumulation of secretions when the patient is close to
death. (hours-days, highly variable)
Goals:
• I would consult with the multidisciplinary palliative care team regarding holistic care for this patient and her family
depending on the setting hospital/hospice/home.
• Clarify the goals of care with the patient and family (on the spectrum of full treatment through to just symptom
management), likely to be mainly comfort
• symptomatically manage the patient ± treat reversible causes of pain and distress.
• I would then counsel the family members so the process of death is comfortable, dignified and aligns addresses
psychosocial and spiritual viewpoints or patient
History
• Identify capacity + confirm extent of NFR / advanced care directives mechanical ventilation, antibiotics, feeding tubes,
CPR, intubation, ICU, artificial hydration
• Gather concerns Encourage patient and family to express their concerns so we can address them
Symptoms
• Identify common end-of-life symptoms
o Pain, agitation, respiratory secretions, breathlessness, nausea, vomiting, depression, anxiety
o Clarify what symptoms are distressing, when they are occurring, are they new.
• Consider if there are any potentially reversible causes of distress constipation, urinary retention (very common)
Other
• Medication review reassess indications and potential benefits of all medications. Only continue medication required
for symptom management. Minimise PRN’s with better choices
• Psychosocial issues
o Preferred place of care and/or death – include needs and wishes of patient, family and carers
If they are bedbound and wish to die at home they need at least 2 family members to support them
o Emotional issues – depression, anxiety, unresolved issues within family
o Spiritual – religious vs non-religious + views on death
Examination
• Aiming to determine the source of pain.
o Hydration status
o Face look in mouth (dry), candidiasis, expression (pain)
o Chest listen to chest to determine upper vs lower airway secretions
o Abdomen feel for palpable bladder suggesting urinary retention
o ± Skin Pressure sores (don’t really cause pain)
• NB: Vitals can be observed but often not needed (or less frequent) as no treatment will be implemented (consider RR, HR,
sats)
Investigations
• no observations, no bloods, do not do investigations.
• ± Bladder scan if suspected retention
Management
• Can consult the palliative care nurse for advice, all treatment is supportive!
Setting
• Single room allows family and loved ones to grieve and avoid distressing other patients.
• Clarify ideal setting is home vs. hospital vs. hospice if out of hospital, need to organise with community palliative care
team
• Ward cohesiveness make sure staff on ward staff understand situation and goals of care e.g. NFR if after hours

± Treating reversible causes of pain


• Unlikely in this patient considering their terminal phase, depends on goals of treatment that were set.
o Urinary retention consider catheter
o Constipated aperients/suppositories. Consider enema but can be painful

107
o Oral candidiasis nilstat drops
o Pressure sores reposition patient
Medical issues
• Aiming to treat common symptoms: pain, agitation, respiratory secretions, breathlessness, nausea, vomiting
• Withdraw all unnecessary medications reassess the indication and potential benefits in the terminal phase for all
medications. Only continue if required for symptom management.
o Consider route - if oral route is not appropriate, consider subcut, transdermal or rectal routes as avoids
cannulation
• Hydration and nutrition food by mouth as long as tolerated.
• Dry mouth good and regular mouth care, consider mouth wash on a swab
• Respiratory tract secretions/death rattle produced by movement of secretions in the upper airways in patients who
are too weak to expectorate effectively; can distress relatives.
o Educate family that gurgling is expected and not distressing for the patient
o Reposition patient (head up) + suction
o Anti-secretory drugs not indicated anymore (glycopyrrolate)
o Increase air movement with fans and windows
o Posturing with physio
• Dyspnoea O2 as needed; usually top out at nasal prongs (as comfort only) ± relaxation techniques
• Pain Usually hydromorphone (Dilaudid) or morphine
o “Opioids have side effects but are the most effective pain medication and will be adjusted for the patient
individually”
o “not prolonging suffering” (non-maleficence) managing pain is improving quality of life which is more
important than ↑ quantity of life, “longevity isn’t the priority anymore”
o NB: Morphine tends to accumulate in poor renal function
o Hydromorphone must be dosed carefully (5x more potent than morphine). Dosing regular + prn
• Terminal agitation/restlessness/delirium
o Non-pharmacological dimly lit room, calming environment
o Pharmacological Usually pain treatment above will help. ± Benzodiazepines (short acting midazolam or long
acting clonazepam)
• Nausea/vomiting metoclopramide (avoid ondansetron as can cause constipation/bowel obstruction)
• Reflux Common if given steroids. Ranitidine subcut and can give PPI
Indications Core medicines for the terminal phase
Dyspnoea, agitation, delirium Clonazepam 1mg injection (long half-life = subcut bolus daily or bd) or
midazolam (short acting), Haloperidol 5 mg/mL injection
Pain Hydromorphone or morphine (can consider non-opioid treatments too)
N/V Metoclopramide 10 mg/2mL injection, Cyclizine (antihistamine)
Noisy breathing/terminal secretions Glycopyrrolate*, just suction
Reflux Ranitidine and PPI
Constipation Suppositories/aperients
Dexamethasone (for nausea, hepatomegaly, pain)
* not as in fashion and only if not eating or drinking anymore as otherwise makes it very difficult

Psychosocial issues
• multidisciplinary team, aiming to address psychological, religious, cultural and spiritual care needs of patient and family
• Reassure family and patient expected changes and that timing of death is unpredictable with sensitive phrasing such
as “time is short”. Never abandon and say ‘there is nothing more we can do’
• Ask what their understanding of the situation is encourage open communication, explore fears and concerns while
being frank and avoiding medical jargon.
o "Your mother/grandmother is in her final stages of her life and it’s likely she has hours-days left."
• Explain to family signs of imminent death profound weakness, confined to bed for most of the day, drowsy for
extended periods, disorientated, severely limited attention span, loss of interest in food/drink, too weak to swallow
medication, Cheyne stokes breathing
• Explore patient + family's views of death
• Spiritual support (chaplain or religious figure)
• Grief and bereavement counsellors

Care after death (practical and legal issues)


• Verify death no palpable carotid pulse, no HS for 2 mins, no breath sounds for 2 mins pupils fixed and dilated, no
response to centralised stimulus, no motor/withdrawal response or facial grimace to painful stimulus
• Paperwork x4 death certificate, cremation certificate, coroner's checklist, donation
• Allow for grieving Ensure single room, family can have as much time as they like, then patient goes to the morgue

108
Guardianship Act
An 80 year old woman presents with haematemesis from a large gastric ulcer. Endoscopic methods of control have failed, bleeding has
recommenced, and urgent surgery is planned. Her relatives request that nothing further be done as she is too old for surgery. How would you
manage this situation?
Introductory Statement
• This is a complex case as the question of what is best for the patient’s long-term outcomes may not coincide with
immediate life-saving measures.
• Goals: My aims are 1) resuscitate the patient (and check for NFR form) 2) targeted history and examination and confirm
diagnosis of an upper GI bleed 3) explain all the possible treatment options and their potential outcomes 4) explore the
concerns of the family, the wishes of the patient and come to a mutual decision that leaves all parties feeling they’re
wishes have been respected, understood and addressed.
Resuscitation
• ABCs
• Concurrently, I would check whether the patient has completed a NFR status
o NFR has an emphasis on CPR but also discusses other treatments and therefore is useful in emergency
situations
o Doctors have the authority to deny treatment if they believe it is futile
• Endoscopy performed already as per question stem, read report
History and Examination
• Read through progress notes and investigations ± focussed history and examination
• Focus on patient’s baseline function, residential status, ADLs etc.
o Helps to gather what their prognosis will be post management

Determination of the person responsible (Adult patient enduring guardian private guardian public guardian)
• If patient has capacity obtain consent
• Assuming that the patient lacks capacity, I would determine the ‘person responsible’. This person has the role of
providing consent if the patient is incapable of consenting. Ideally, they should be aware of the patient's advanced care
directive and guardian may have written an advanced care plan. If a guardian disagrees with an advanced care directive,
doctors do not have to do what the guardian wants.
• The order of priority according to the Guardianship Act 1987 is:
1) Person responsible
i. An appointed guardian (including an enduring guardian) who has been given the right of consent to
medical and dental treatments.
ii. The most recent spouse or de facto spouse (including same sex partner) when the spouse or de facto
has a close and continuing relationship with the person
iii. The unpaid carer or the carer at the time the person entered residential care (government carer
benefit does not count as paid)
iv. A relative or friend with a close personal relationship with the patient
2) If there is no person responsible, go through the Guardianship Division of the NSW Civil and Administrative
Tribunal (NCAT)
• Definitions
o Enduring = only when the ‘principal’ has become of unsound mind e.g. lost capacity whereas appointed can use
their power at any point in time
o Guardian = make health and welfare decisions for you, which could include deciding where you live or
consenting to medical treatment for you
o Attorney = manage your assets for you and can do anything on your behalf that you can lawfully do yourself,
which could include paying bills from your bank accounts or selling your property.
o Advanced Care Directive = ‘Living Will’ which provides a clear statement that sets out your directions including
your wishes and values that need to be considered before medical treatment decisions are made on your
behalf.
can be attached to an enduring guardianship appointment or an can be written separately which gives
greater flexibility to adjust advance care directive without the need to execute a new enduring
guardianship document
no prescribed format, simply written wishes
o Next of kin = usually used to indicate blood relative but is not relevant, it is the ‘person responsible’ according to
the guardianship act that makes the decisions or if possible the appointed/enduring guardian/attorney.
• If the person responsible can’t or won’t make a treatment decision, he or she needs to decline in writing. Then the next
person down the list is responsible.
• Limitations and exceptions of the person responsible which only the NCAT can consent

109
a. There are certain treatments that the ‘person responsible’ cannot consent to. Examples of these include
termination of pregnancy, treatments to achieve infertility, and new treatments that have not yet gained
substantial support.
b. If the patient indicates or has previously indicated an objection to a treatment, the person responsible cannot
give consent. However, if the practitioner assesses the patient to have minimal or no understanding of the
treatment and the possible distress from treatment is tolerable and temporary, then this can be disregarded.
c. Additionally, if the treatment is emergent and/or necessary, treatment can be given without consent from the
person responsible. This is particularly relevant in emergency scenarios.

Discussion with persons responsible (family) and obtain informed consent


setup an appropriate environment: quiet space, interpreter, aids, drawings
• Discuss about concerns, why they don’t want surgery
• Clarify what they want, patient wants, and their understanding is correct
• Obtain informed consent from this individual by explaining the indications, procedure, risks and benefits, alternatives
and eliciting concerns (IPRAC).
o Indications
As the endoscopy has failed, this massive GI bleed is likely terminal without surgery.
Confirm also advanced care directive if present
o Procedure
Repeating endoscopic intervention
Open laparotomy to stop the bleeding and removing dead portion of GIT..
o Risks and benefits
Risks of surgery – anaesthetic risk, may not be successful, risk of bleeding, risk of post-operative
complications
Benefits – can potentially stop the bleeding.
o Alternatives
Palliation
• Explain what palliative measures could look like: fluid resus, food, medication, lines,
transfusions
• Explain there is a ceiling of care with elderly frail patients, focus is giving them the best QoL
• Likely will not transfuse as they are actively bleeding, however can consider transfusion to
relieve symptomatic anaemia.
Management
• Based on wishes of person responsible
• Documentation
o Any decisions about limitations on care (medicolegal reasons)
o Include who the plans were discussed with
o Also to help support the family and minimise guilt

Appendix
• There are 3 types of consent you may be asked to give, including:
o Implied consent - your agreement is given by cooperating with a healthcare professional's instructions for
routine procedures.
Examples of implied consent include when you extend your arm for a blood sample, take and swallow
medication that is given to you, or attend an appointment to receive information or advice for the
management of your condition.
o Verbal consent - your agreement is given verbally for a treatment or procedure that doesn't carry a significant
risk.
o Written consent - your agreement is given by signing a document for a treatment or procedure that is complex
or carries a higher risk.

110
Geriatrics
Recurrent Falls
A 76 year old woman, who is in a nursing home, has fallen over on two occasions in the past 3 days. How you would assess her?
Introductory Statement
Provisional Diagnosis: Recurrent falls
DDx: Could be due to a number of factors including drugs, age-related changes, medical conditions, environment
Goals: My approach in this case would be:
• Ensure haemodynamically stable
• Immediately identify and manage any potential injuries (fractures, head injury) esp. concerned if on anticoags
• Focussed hx/examination/Ix to determine the aetiology/contributing factors for the falls
• Initiate appropriate intervention both acute and L/T, including pharm and non-pharm modalities to reduce risk of falls
and treat potential osteoporosis
ABCDE Assessment
• Primary survey + resus as appropriate (common for elderly to be dehydrated, septic)
o If on anticoagulation need to do CT-B if so
• Secondary survey to assess for specific injuries/areas of localised pain
History
• Collateral history if possible
Symptoms ± complications
• Falls history
o Circumstances when?, mechanism (trying to distinguish mechanical vs non-mechanical)?, witnesses?
Previous episodes + how they were similar/different?
o Before sx of seizure, syncope, hypoglycaemia, hypotension, ACS
o During head strike, convulsions, tongue biting, incontinence, LOC?
o After LOC, confusion, fatigue, pain, length of lie, ability to get up
• Complications
o Consequences of fall – injury including head strike, hip pain or broken wrist (Colles fracture common), length of
lie (important for rhabdomyolysis),
Risk Factors
• DAME
o Drugs centrally acting, anti-hypertensives, polypharmacy (>5), change in medications
o Age-related ADLs, gait, vision/sensory impairment, postural stability
o Medical conditions any acute illness (e.g. UTI, chest infection, MI, hypoglycaemia), chronic neurological
conditions (e.g. stroke, dementia, Parkinson's), cardioresp status
o Environment e.g. home safety, loose rugs, steps, slippery floor, poor lighting, pets. Change in environment,
who they live with etc
• Risk factors for minimal trauma fractures – osteoporosis + secondary causes of osteoporosis (renal osteodystrophy,
Paget’s, malignancy, chronic steroids etc.)
Other
• Do they have advanced care directive, who is the next of kin
• Functional status – determines operative vs. non-operative management
• Preparation for surgery – AMPLE: allergies, medications (including anticoagulants), past medical history + previous
surgeries, last meal, events leading up to presentation
Examination
• Vitals postural BP, fevers
• Observation for injuries e.g. head (cephalohematoma) and fractures (hip, wrist, etc.)
• MSK on areas of pain/tenderness e.g. c-spine, hip, spinal
• Neuro esp. lower limb gait assessment/rhombergs + cranial nerves especially vision
• Fluid status examination
• Cardioresp HR + rhythm, BP (sitting and standing), murmurs (e.g. AS), signs of APO or resp infection
• General infections
Investigations
(broad and depends on clinical suspicion)
Bedside
• ECG, UA (infection), BSL (hypoglycaemia), VBG (electrolytes and lactate), MMSE
o Can consider holter (rule out arrhythmia)
Laboratory

111
• FBC/CRP (infection, anaemia), EUC/CMP (electrolyte disturbances can cause falls, AKI 2 o to rhabdo), coags (risk of
bleeding)
• Consider CK (if long lie), cognitive impairment screen if not done recently (TFT, B12, Folate, vit D), septic screen if
concerned re infection (incl. blood culture, urine MCS, CXR + bloods)
• G+H if expecting surgery
Imaging
• X-ray of any potential injuries e.g. NOF (AP and lateral), CT-B if confused/concerned re bleed, bladder scan for urinary
retention, AXR for faecal loading
• CXR (for anaesthetists)
• Consider DEXA scan as F/U
Management
• Issues 1) treat injuries 2) prevent recurrent falls 3) treat osteoporosis

Acute – treat injuries


• Falls risk assessment
• Treat any injuries (skin tear, soft tissue, lacerations, burns, fractures, rhabdomyolysis)
o rhabdomyolysis (urinary myoglobulin, CK, deteriorating EUC)
• Treat any precipitating factors (DAME)
o Consider suspending medication
• Manage delirium if necessary

Long term – prevent recurrence and treat osteoporosis


• Pharmacological
o Review and optimise mediations (may involve pharmacy) reduce polypharmacy
o Treat osteoporosis - If minimal trauma fracture
• Non-pharmacological
o Involvement of allied health esp. physio, OT, improve balance/mobility + living environment
o Optometrist – identify and correct vision abnormalities if present
o Podiatrist – appropriate footwear
o Complete/alter ACD
o Pendant alarms/ apple watch – reduce risk of long lie
• Safe discharge planning
o Ensure home environment suitable/receiving appropriate level of care
o Can consider rehab if appropriate

112
Fall/Fracture (Autonomic Failure)
A 72 year old woman with longstanding postural hypotension secondary to autonomic failure presents after a fall at home. Her right hip is painful
along with her left ribs. How would you assess and manage her?
Very similar to above Recurrent Falls and Recurring dizziness with autonomic failure (2020) above but consider some extra info
Introductory Statement
Provisional Diagnosis: Fall most likely due to a hypotensive episode which has potentially been complicated by a right hip and left
rib fracture
Goals:
• See Recurrent Falls
ABCDE Assessment
• See Recurrent Falls
• Rule out pneumothorax given likely left rib fracture
• Consider doing x-ray and giving pain relief early
History
• See Recurrent Falls
Symptoms
• Same falls history as Recurrent Falls
• Symptoms of autonomic failure see Recurring dizziness with autonomic failure (2020)
o Decreased sympathetic function – orthostatic hypotension, decreased sweating → heat/exercise intolerance,
ejaculatory dysfunction (point and shoot),
o Decreased parasympathetic function – constipation, nausea, urinary retention, erectile dysfunction
• Complications
o See Recurrent Falls
Risk Factors/Causes
• Same DAME See Recurrent Falls
• Medical conditions
o Causes of autonomic failure – neurodegenerative diseases (e.g. Parkinson’s, Parkinson’s plus syndromes),
neuropathies (DM, amyloidosis, Sjogren’s etc.)
o Causes of postural hypotension (drop in SBP ≥20mmHg when standing) – autonomic neuropathy, drug related
(antihypertensives, diuretics, alcohol), dehydration, cardiovascular (anaemia, arrhythmia, CCF)
Other
• Same ACD, functional status, preparation for surgery as Recurrent Falls
Examination
• Same as Recurrent Falls looking for injuries and causes
• Examine hip (remember to give analgesia first!)
o Femoral fractures typically cause severe pain, shortening of limb and are abducted and externally rotated
o Neurovascular – palpate distal pulses, assess sensorimotor function
Investigations
• See Recurrent Falls
• AP and lateral pelvic x-ray
o intra or extracapsular, displaced?, type of fracture e.g. intra-trochanteric
Management
• consult the geriatrics, orthopaedics and cardiology registrars
• Issues 1) treat injuries 2) prevent recurrent falls 3) osteoporosis
• See Recurrent Falls +

Hip Fracture
• See Surgery: Fractured Neck of Femur

Rib Fracture
• Usually no surgery necessary
• Analgesia and only surgery if significant deformity, severe flail chest or non-union

Long term
• Emphasise osteoporosis treatment

113
Delirium [Altered mental status post cellulitis] (2020)
A 50 year old woman is hospitalised for IV antibiotics for cellulitis. On the evening of the second day she becomes acutely confused. How would you
assess her?
Introductory Statement
Provisional Diagnosis:
• My primary concern in a patient with a known infection and an acutely altered mental state would be a delirium,
secondary to sepsis.
DDx:
• See appendix for all DDx
• Other causes of delirium
o Constipation/Urinary retention/Pain
o Drug overdose/withdrawal – may be the more likely in a young patient who is already on treatment for infection
o Electrolyte derangements
o Hypo/hyperglycaemia
o Neurological conditions
• Psychiatric illnesses
Goals: My priorities are
• Conduct an A to E assessment with simultaneous resuscitation to identify and treat a possible sepsis
• Complete targeted H/E/I to identify any other causes for her delirium
• Provide appropriate management to ensure safety of the patient and reverse the cause of delirium if possible
ABCDE Assessment
• Consider aggressiveness of patient and whether de-escalation is required
• Airway (as normal)
• Breathing (as normal)
o Rule out hypoxia
• Circulation
o Focus on circulation to identify sepsis – hypotension, tachycardia, warm peripheries
o Resuscitate with supplemental oxygen and IV fluids as required
o Send bloods including a VBG to assess lactate/pH and blood cultures
If the patient has an SBP < 90mmHg, lactate >4 or base excess <5.0 -> activate a rapid response/Met
Call, follow the sepsis kills pathway and consult ID regarding broadening of IV antibiotics
• IV meropenem/tazocin + vancomycin + clindamycin/lincomycin
• Disability (as normal)
o Rule out neurological causes and glucose causes
• Exposure (as normal)
o Temperature may give a hint to drug withdrawal or adverse drug reaction
History
• Likely collateral from nursing staff/family as patient is acutely confused
• Initial presentation – severity of cellulitis, investigation results, treatment received to date (current antibiotic regimen),
progress on the ward
Symptoms
• Hyperactive (uncooperative, anxious, agitated) vs hypoactive (quiet, withdrawn, drowsy)
• Baseline function - any pre-existing cognitive impairment
• Any fluctuation in level of consciousness
• Time/speed of onset/preceding events e.g. falls or injuries
• Any past episodes of delirium
Differentials
• Causes of delirium
o Infective symptoms other sources e.g. chest pain, cough, dysuria, frequency, pressure sores
o Review fluid input/output, stool chart if available - dehydration, electrolyte derangements, constipation
o Medication review - ensure adequate relief, review for any psychoactive drugs, possible toxicity/drug
interactions
o Past medical history and medication chart review
o Drug and alcohol use
• PMHX psychiatric illnesses
Other

Examination
114
• Reassess general appearance and vitals
• Assess observation chart
• Assess cognition
o 4AT (assessment test for delirium and cognitive impairment)
Alertness
AMT4 (orientation) – age, date of birth, place, current year
Attention – months of the year backwards
Acute change/fluctuating course
• Evidence of significant change or fluctuation in alertness, cognition, other mental function
arising over the last 2 weeks and still evident in last 24 hours
o Confusion assessment method
Acute onset, Inattention, Disorganised thinking, Altered level of consciousness, Disorientation, Memory
impairment, Perceptual disturbances, Psychomotor agitation, Altered sleep-wake cycle
o ?MMSE/MOCA
• Signs of infection
o Skin – areas of cellulitis (looking for signs of spread e.g. drawing a line or comparison with previous pictures),
IVC sites, pressure sores
o Cardiac exam – arrythmias, murmurs
o Respiratory exam – decreased AE, focal crackles (chest infection)
o Abdominal exam – tenderness, peritonism
o Neuro exam – focal neurological deficits which may suggest an intracranial pathology
o Signs of alcohol/drug withdrawal e.g. tachycardia, tremor, diaphoresis, pupillary changes
• Fluids assessment
Investigation
• Basic investigations + hypothesis driven investigations
Bedside
o ECG
o Finger prick BSL
o VBG – acidosis, raised lactate, electrolyte derangements
o Urine dipstick – leucocytes and nitrites
o Bladder scan – urinary retention
Laboratory
o FBC + CRP/ESR, EUC/CMP, LFT
o Blood culture, Urine culture + sputum culture (septic screen), Wound MCS if not already done (cellulitis)
o ± B12 + folate, TFT, vit D
o ± BAC and therapeutic drug levels
Imaging
o CXR
o ± CT brain or LP if indicated on history/exam
o ± further cellulitis imaging XR (osteomyelitis)
o ± EEG
Management
Cellulitis
• Commence IV antibiotics and fluids
o IV Ben Pen if strep suspected
o IV fluclox or vanc if staph suspected (purulence)
o More broad-spectrum cover if systemic symptoms/septic shock
Delirium
Definitive
• Reverse the underlying cause
o Hypoxia Oxygen
o CUPD constipation (aperients), urinary retention (check for full bladder ± catheter), pain (analgesia
optimisation), dehydration (correct electrolyte abnormalities and optimise hydration)
o Electrolyte abnormalities correct with fluids
o Drugs and medications rationalise medications, AWS if concerned about alcohol withdrawal
o Glucose correct
o Sepsis antibiotics/electrolytes
Supportive
• Frequent nursing observation due to unpredictable behaviour
• Quiet and calm environment single room if possible
• Frequent re-orientation and reassurance of the patient - have clocks/calendars is possible

115
• Encourage family presence
• Minimise sensory deprivation Encourage use of glasses and hearing aids and ideally windows in room
• Facilitate physiological sleep minimise nursing/medical assessments during sleep hours, reduce noise
• VTE prophylaxis TED stockings + subcut clexane
• ± Antipsychotics only in severe agitation where there is risk of harm to the patient of others around them
o “Frequently no medication will be needed to manage delirium if there is prompt treatment of the underlying
cause and good non-pharmacological management”
o Haloperidol 0.5mg IM (quicker onset) or PO single dose is the most studied tx for delirium (other; olanzapine,
risperidone) which acts as a sedative
250microg okay for a very small and frail person
Not useful for hypoactive delirium

Appendix: DDx of Delirium


Many aetiologies of delirium and this is what we need to consider/screen. CUPD SENDHUGS
• Sepsis/Infection (e.g., UTIs are a common cause in elderly patients; pneumonia, meningitis)
• Electrolyte abnormalities (dehydration, hypo/hypernatraemia, extreme disturbances in ↑Ca/Mg/PO4)
• Neurological (e.g., stroke, space occupying lesion, infection, trauma)
• Drugs and toxins (also referred to as toxic encephalopathy) – medication induced/substance intoxication/substance
withdrawal
o Anticholinergics, Benzodiazepines and barbiturates, Antidepressants and antipsychotics (especially those with
anticholinergic activity, e.g., quetiapine), Opioids, Diuretics (may cause electrolyte abnormalities), Recreational
drugs (intoxication/withdrawal), Alcohol use, Heavy metals (e.g., arsenic, lead, mercury
• Hypoxia (e.g., anaemia, cardiac failure, COPD, pulmonary embolism), Acute cardiovascular disease (MI, shock, vasculitis)
• Uraemia (kidney failure) or Liver failure
• Glucose (hypo or hyper glycaemia, DKA)
• Sleep deprivation
• Ongoing symptoms, including CUPD:
o Constipation
o Urinary retention
o Pain
o Dehydration
• Other
o Hyperthyroidism or hypothyroidism
o Vitamin deficiencies (e.g., vitamin B12, folic acid, or thiamine deficiency)
o Major surgery
o Trauma (e.g. hip fracture)
o Hearing or vision loss
Note: Metabolic diseases (most common cause; also referred to as metabolic encephalopathy) including electrolyte, kidney/liver
failure, glucose, hyper/hypothyroidism, vitamin deficiency

116
Altered mental status post fall (2020)
An 83 year old woman is admitted following a fall. She becomes acutely confused the following day and is very disruptive in the ward. How would you
assess and manage her?
Introductory Statement
Provisional Diagnosis: In this patient I’m concerned about delirium given her acute confusion and disruptive behaviour.
DDx:
• See Delirium [Altered mental status post cellulitis] (2020)
o Potentially joint causes for fall and the delirium e.g. polypharmacy, infection
Goals
• Determine if she requires an urgent CT-B if reduced GCS or unwitnessed fall
• Perform a thorough clinical assessment to determine the cause of delirium
• Manage appropriate with supportive care and reversing the underlying cause.
Rest of case
• Delirium See Delirium [Altered mental status post cellulitis] (2020)
• Fall pain, fractures, blood loss

117
Dementia (2020)
A 73 year old woman, who lives alone, has become increasingly forgetful and vague. Her son is worried that she is unsafe. How would you manage
this situation?
Introductory Statement
Provisional Diagnosis: In an elderly patient with signs of cognitive decline, I am concerned about dementia.
DDx:
• I would also want to consider potentially reversible causes of cognitive decline
o Normal pressure hydrocephalus (wet – urinary incontinence, wacky – dementia, wobbly – ataxic gait)
o Hypothyroidism
o Vitamin B12(?folate) deficiency
o Mimickers of dementia e.g. delirium and depression (pseudodementia)
Goals: My approach would be to:
• Take a thorough history from the patient and with collateral from her son to further explore his concerns regarding his
mother's safety and home, and perform a targeted examination including a cognitive screening test e.g. MOCA
• Perform appropriate investigations to exclude differentials
• Provide appropriate management with input from a multidisciplinary team to support and educate the patient and her
family, and optimise her living environment
History
• From the patient and collateral from son (interviewed separately from the patient if possible)
Symptoms
• Memory loss - onset, duration, progression (rate and pattern of decline), severity, pattern (anterograde vs. retrograde)
• Additional cognitive impairment exploring cognitive domains PM has CA-LM SLE
o Perceptual-motor
o Complex attention
o Learning and memory
o Social cognition
o Language
o Executive function
• Symptoms specific to different types of dementia
o Alzheimer’s (insidious over 8-10 years)
Insidious onset
Episodic impairment of memory
Characteristic order of language impairment: naming → comprehension → fluency
± visuospatial deficits and impaired executive function (impaired judgement, motivation)
o Vascular dementia (may be abrupt and stepwise deterioration with recognised/unrecognised strokes)
Characterised by prominent impairments in executive function and processing speed
May have asymmetric or focal deficits e.g. hemiparesis
o Lewy body dementia and Parkinson disease dementia (steady decline over 8-10 years)
Motor parkinsonism
REM sleep behaviour disorder, visual hallucinations, fluctuations in level of alertness and prominent
visuospatial dysfunction
Dementia of Parkinson disease – emerges 5-8 years after the onset of the movement disorder
Lewy body dementia – Parkinsonism and cognitive decline manifest at the same time
o Frontotemporal (usually 40-69)
Inappropriate behaviours and personality changes (behavioural variant FTD)
Aphasia out of proportion to memory impairment (primary progressive aphasia
Risk factors
• Age
• Genetics/family history
• Medical – vascular risk factors and hx of vascular disease e.g. previous MI, previous stroke
• Psychosocial - low education attainment, physical inactivity, social isolation
Differentials
• Delirium acute onset, fluctuating, usually reversible
• Normal pressure hydrocephalus wet (urinary incontinence), wobbly (ataxic), wacky (dementia)
• Hypothyroidism sx
• Depression ask about mood, geriatric depression scale
Other
• Safety at home
o Current participation in activities of daily living - compared to baseline

118
E.g. handing finances, community and social activities, driving and other household tasks
Impacts of memory loss e.g. forgetting to turn the stove off etc.
o Home environment e.g. steps around the house
o Supports at home - formal (i.e. services) and informal (family and friends)
• Complete the rest of the medical history, including
o Comprehensive medication review for any drugs that impair cognition (e.g. analgesics, sedatives, psychotropic
medications, anticholinergics)
o Drug and alcohol use
Examination
• General appearance + vitals
• Cognitive testing
o Screening tool e.g. MMSE, MOCA, RUDAS
o Mental state examination
• Physical examination
o Neurologic examination focal neurological deficits suggestive of prior strokes, parkinsonism and
abnormalities suggestive of alternative diagnosis
o General systems review for other medical illness (potentially causes of delirium)
Investigations
Bloods
• FBC, EUC, CMP, LFT, BSL
• B12/folate levels, thyroid function test (TFT), vit D
Imaging
• Neuroimaging e.g. CT/MRI brain to identify any intracranial pathologies
Management
• Management of dementia is largely supportive and should be conducted in a MDT setting with a focus on patient and
family education and counselling and other interventions to support function and ensure patient safety
• Goals Preserve cognition, relieve distressing symptoms, support family
• Disposition
o Are they safe to go home or do they have to be admitted for further investigations/management
o Discharge destination - home vs period of respite to organise carers and other support services
• Can use the biopsychosocial model for planning management (from psychiatry)

Psychological
• Psychoeducation and counselling
o What is dementia?
o What are the main issues patients with dementia will face?
Symptoms can include
• Forgetting all sorts of things
• Confusion
• Trouble with language
• Trouble concentrating and reasoning
• Problems with tasks such as paying bills or balancing a cheque book
• Getting lost in familiar places
As the dementia gets worse, people might
• Have episodes of anger or aggression
• See things that aren’t there or believe things that aren’t true
• Be unable to eat, bathe, dress or do other everyday tasks
• Lose bladder and bowel control
o Provide reassurance that the disease progresses differently in individual patients and may not progress rapidly
“ this is something a lot of elderly people face”, “it may not progress rapidly”
o How will it be managed and how can family help support patients?
“There is a process here that we can follow”
• Psychotherapies
o Memory training targeted stimulation, recalling past memories etc.
o Structured group cognitive stimulation program (helps with cognition and socialisation)
Biological
• Non-pharmacological
o Referral to specialists (neurologist/neuropsychiatrist) if required, social worker, occupational therapist etc. and
regular follow-up with a good GP
o Optimise SNAP

119
• Pharmacological
o Alzheimer’s and vascular dementia
Cholinesterase inhibitors (donepezil, rivastigmine, galantamine) AD patients
• Increase cholinergic transmission by inhibiting cholinesterase at the synaptic cleft and provide
modest symptomatic benefit in some patients with AD
• NB: Be wary with patients with PMHx that could have sx exacerbated e.g. bradycardia
Memantine (NMDA receptor antagonist) advanced dementia
• N-methyl-D-aspartate (NMDA) receptor antagonist
• Appears to have modest benefits in patients with moderate to severe AD
o Lewy Body
No disease modifying treatment for Lewy Body Dementia
Atypical antipsychotics - want to avoid typical antipsychotics due to existing parkinsonism symptoms
• Very low doses
• Sedating antipsychotic can be useful for sleep disturbances
Melatonin or Clonazepam REM sleep disturbances
o ± antipsychotics (delusions/hallucinations), BZD (anxiety/agitation), SSRI (comorbid depression), optimisation of
vascular health (if vascular dementia)
Social
• Mental health of the patient and carers – referral to helplines, support groups and social worker/psychologist
• Consider risk assessment (to self, to others, becoming ill, malnourishment, medication non-compliance, driving, finances)
and advise on staying safe at home
o Optimise environment
Optimise falls risk Secure loose rugs or use non-skid backing on rugs, Tuck away loose wires or
electrical cords, Wear sturdy comfortable shoes, Keep walkways well lit
o Formal services
Refer to ACAT (Aged Care Assessment Team) who will perform a comprehensive assessment to
determine whether the patient is eligible for government subsidised services (e.g. transport, shopping,
cleaning, toileting) to allow the patient to stay independent in their own home or move into aged care
homes (later down the track)
• Make plans for the future – advanced care directives, power of attorney etc.

120
Endocrinology
Dyslipidaemia (2020)
A 35 year old asymptomatic man presents for a check-up. His total serum cholesterol is 6.6 mmol/L (normal <4.0mmol/L). How would you assess and
manage him?
Introductory Statement
Provisional: My impression is that this gentleman has hyperlipidaemia.
DDx: Key differentials are between
• Congenital
o familial hypercholesterolaemia
• Acquired
o obesity, DM, physical inactivity, alcoholism, cushing’s, hypothyroidism, nephrotic syndrome, cholestatic liver
disease, drug induced (steroids, OCP, high-dose diuretic, metoprolol, etc.)
Goals: My approach in this case would be:
1. Identify the cause of hypercholesterolaemia through hx/ex/Ix incl. a repeat lipid profile
2. Ascertain CVD risk
3. Manage the hypercholesterolaemia and CVD risk with non-pharmacological measures/optimising risk factors and
consider pharmacotherapy with statins
History
Symptoms ± Complications
• History mentions asymptomatic
o Screen for symptoms of CVD SOB, chest pain, orthopnoea, leg pain etc
Risk Factors/Causes/Differentials
• Essentially cardiovascular risk factors
• Medical
o T2DM, HTN, hypothyroidism, nephrotic syndrome, cholestatic liver disease, medications (steroids, AP)
• Demographics (already know gender/age)
o ATSI, Past hx of CVD (IHD, CVA, angina, PVD), fam hx of CVD/hypercholesterolaemia (thinking familial
hypercholesterolaemia)
• Lifestyle
o Obesity, SNAP
Examination
• General Height, weight, BMI, waist/neck circumference
• Cardioresp signs of hypercholesterolaemia, CVD, peripheral vascular disease
• Consider
o Abdo exam if concerned re liver disease
o Thyroid if concerned re hypothyroidism
Investigations
Diagnostic
• Repeat fasting lipid profile and make sure first was actually fasting
Bedside
• BSL (screen for T2DM), UA (nephrotic syndrome = high protein)
Laboratory
• FBC, EUC, LFTs, TFTs (hypothyroidism)
o Good as baseline esp. if considering starting statin (can cause hepatotoxicity), also looking for derangement that
can cause hypercholesterolaemia (esp EUC, LFT)
Management
Treatment algorithm
• CALCULATE absolute CVD risk (Framingham tool risk of cardiovascular event in next 5 years) this will determine
management
• Low CVD risk <10%
o Lifestyle advice, repeat lipids every 5 years
• Moderate CVD risk 10-15%
o Intensive lifestyle advice, repeat lipids every 2 years
o Consider statin if not reach targets within 3-6mths
• High CVD risk >15% or high RF (regardless of CVD risk tool)
o Intensive lifestyle advice + cholesterol lowering therapy (statin unless CI) + anti-hypertensive therapy (if not CI),
repeat lipids every 1 year
• Target lipids total cholesterol <4.0, LDL <2, HDL >1
121
• If starting/changing tx measure lipids between 1-3 months and then continue accordingly with above

Non-pharmacological
• SNAP risk factors + counsel on CVD risk

Pharmacological (see below)


• [1] Statin
o atorvastatin, rosuvastatin and simvastatin are more potent than fluvastatin or pravastatin
• NB: non-statin medications have not been studied for primary prevention if statins are not tolerated
• [2] ± ezetimibe (↓ absorption in SI) OR PCSK-p inhibitors (↑ LDL receptors, ↓ LDL) if LDL-C target not achieved
o PCSK-9 is monoclonal Ab and should be given by specialist
• ± fibrates (induces genes that ↑ lipid metabolism) useful for predominant hypertriglyceridemia
• Others bile sequestering agents (block bile acid absorption), nicotinic acid (reduce fatty acid breakdown),
o Used if statin C/I or resistant
• The use of ezetimibe or PCSK-p as monotherapy is primary prevention has not been studied!

Long-term
• Counselling A/E e.g. muscle aches and pains not related to exercise, importance of compliance
• Follow-up investigations around 1-2 months lipids, LFTs, CK

Drug MOA SE CI Notes


[1] Statin (e.g. 1) Statins reversibly inhibit 1) muscles (myalgia, Hypersensitivity, acute
atorvastatin - HMG-CoA reductase in myopathy, liver disease,
Lipitor) hepatocytes (compete with rhabdomyolysis) pregnancy, muscle
HMG-CoA which has a similar 2) general (rash, GI) disorder
structure) → mevalonate Myalgia, myopathy
(required for endogenous 3) Hepatic: ↑ LFT Interacts with CYP e.g.
cholesterol production) is due to CYP450 warfarin
produced at reduced rates → 4) pleiotropic effect
reduced cholesterol ↓ (↓ CRP)
triglyceride
2) Results in an increase in LDL
receptors reducing
circulating levels of LDL
3) ↑ circulating HDL
Cholesterol Decrease cholesterol Sane as statin but Severe hepatic disease Often used w/ statin
lowering drug absorption in SI (apparently at not as severe Hypersensitivity
e.g. ezetimibe brush border though
mechanism is not entirely
known)
Fibrates (e.g. PPAR alpha agonist → induces Myopathy, Severe hepatic or renal - Fibrates not used
fenofibrate) genes that promote lipid hepatotoxicity, dysfunction, existing as monotherapy
metabolism by ↑ LPL gallstones gall bladder disease usually only w/
(lipoprotein lipase activity) statins.
Interacts with CYP e.g. - Less effective at
↓ LDL, ↑ HDL, ↓ ↓ ↓ warfarin reducing LDL, more
triglycerides at increasing HDL
and reducing TG.
- Increased bleeding
risk with oral
anticoagulants.
- Severe miosis and
myoglobulins with
HMG-CoA
inhibitors.
Bile acid Block bile acid absorption Rash, irritation of Complete biliary Delay or reduced
sequestrants e.g. thereby inhibiting reabsorption skin and tongue, obstruction, levels with warfarin,
Cholestyramine of biliary cholesterol into the abdominal bloating hypertriglyceridemia tetracyclines etc.
liver + ↑ bile secretion (slight and cramping
activation of TAG production)
Nicotinic Acid e.g. Reduce fatty acid release from GI disturbances, Liver disease, gout, PUD
Niacin adipose tissue inhibits hepatic flushing,
122
TG production and VLDL palpitations,
Rarely used secretion. azotaemia

In adipose tissue, binds to Hyperuricaemia


GPR109A → inhibits adenylyl hyperglycaemia
cyclase → lower cAMP →
reduces lipolysis as cAMP
activates the lipolysis pathway
through PKA
PCSK9 inhibitor Inhibits PCSK9 ↑ LDL Myalgia, Relatively new
e.g. alirocumab, receptors ↓↓↓ LDL, ↑ neurocognitive
evolocumab HDL, ↓ TAG disorders
Lipids = triglycerides (TAGs), phospholipids, cholesterol esters

123
Diabetes (2020)
60yo F found to have a fasting BSL of 14.7mmol/L. She attends the diabetes clinic for the first time. How would you assess and manage her?
Introductory Statement
Provisional Diagnosis: The issue in this case is a single elevated fasting BSL. In the absence of symptoms this is insufficient to
diagnose T2DM, she would most likely require a second abnormal result (preferably the same test on a different day) or
measurement of HbA1C.
DDx: My differentials would include
• Prediabetes
• Late onset T1DM
• Secondary diabetes (e.g. genetic disorders [cystic fibrosis, haemochromatosis], damage/removal of pancreas, endocrine
disorders [Cushing’s syndrome, acromegaly, hyperthyroidism], autoimmune disorders and medicines such as APs,
thiazides, corticosteroids).
Goals: My aims in this case would be too:
• Targeted hx (for symptoms of DM, RF and macro/microvascular complications and physical examination for general
metabolic and complications of DM
• Ix including confirmatory BSL
• Management with both non-pharm and pharm measures
History
Symptoms
• Hyperglycaemia often asymptomatic. Symptoms include polyuria, polydipsia, polyphagia, pruritis, nocturia, weight
loss, blurred vision, fatigue
• Other frequent fungal and bacterial (UTI)
Complications
• Since we don’t know how long this patient has had diabetes for, important to screen for complications
• Acute
o DKA, HHS, mortality
• Chronic
o Macrovascular - cardiovascular (ACS, angina), cerebrovascular (stroke, TIA), or peripheral vascular disease
(claudication, rest pain)
o Microvascular - diabetic retinopathy (blurred vision), nephropathy, diabetic neuropathy (typically distal
polyneuropathy, autonomic e.g. urogenital – bladder dysfunction, CVS – silent MI,)
o Other/multifactorial - Increased frequency of infections, poor wound healing, ulcers, cataracts
Differentials
• Secondary causes medication review (thiazides, AP, corticosteroids, calcineurin inhibitors)
• Signs of Cushing’s
Risk Factors/Causes
• Demographics age, fam hx,
• Medical related hx of GDM/PCOS, metabolic syndrome (HT, lipids, obesity, DM, OSA), pancreatitis, liver failure
• Lifestyle CVD risk factors
Examination
• General BP, BMI, weight, waist circumference, cushing abnormalities (e.g. moon facies, buffalo hump)
• Complications (cardioresp, neuro, peripheral vascular, ophthalmic, derm)
o Macrovascular cardioresp exam looking for signs of CCF, lower limb exam looking for signs of PVD,
neurological exam looking for focal neurological deficits (stroke)
o Microvascular fundoscopy (retinopathy, macular oedema), visual fields, peripheral neuro exam looking for
peripheral neuropathy (monofilament test)
o Foot exam – looking for ulcers or charcot foot
o Derm ulcers!, infections, acanthosis nigricans, pruritis
Other skin changes if diabetes due to other conditions e.g. cushing’s or acromegaly
Investigations
Diagnostic
• Confirm diabetes
o OGTT: 75 g of oral glucose measure BG 2 hours later >11.1
o Random: >11.1
o Fasting > 7
o HbA1c ≥ 6.5%
o NB: only need 1 test if patient is symptomatic
• Consider testing T1DM with auto-antibodies Anti-GAD, Anti IA2, ICSA
Bedside
124
• ECG, ABI, UA (infection), urine albumin:creatinine, swab if find infection
o UACR >2.5 in men and >3.5 in women is abnormal. Should be aiming to get below this with treatment
Purpose is to screen for underlying diabetic nephropathy #1 cause of renal failure in western countries
Laboratory
• EUC/eGFR (renal failure from hyperglycaemia), ketones, lipid profile (LDL, HDL, cholesterol, triglycerides), LFT (fatty liver,
pre meds)
Imaging
• Detailed retinal exam by ophthalmologist
Management
• Aim to prevent complications secondary to chronic hyperglycaemia, without risk of hypoglycaemia
• MDT management diabetes educator, GP, optometrist/opthal, podiatrist, dietician, exercise physiologist,
endocrinologist (if complicated case, tx refractory)
Non-pharm
• Diabetes educator Education about T2DM
o SNAP Physical activity, diet, weight, smoking (cessation), alcohol
o Practical issues e.g. national diabetes support scheme (NDSS), driving, hypo management)
o Other meds interaction e.g. Beta-blockers will blunt the adrenergic response to hypoglycaemia experienced,
such as increased heart rate or palpitations. Discuss this with patients and their families in order to increase
their awareness of this.
• Weight loss (SNAP above, ± bariatric surgery)
• Peripheral neuropathy
o Skin care moisturising, monitoring for lacerations/ulceration ± pharm
• Test BSL before driving to avoid hypos (BSL >5)
• Vaccination status (pneumococcal vaccine, influenza vaccine, dTpa)
• Self BGL monitoring
• Blurry vision avoid permanent solutions to refractive errors as ↓ BSL will resolve some lens issues
• Screen for mental health issues
Pharm
• For highly motivated patients with HbA1c near target (<7.5%), 3-6 month trial of lifestyle modification only can be done
first
• Target HbA1C should be individualised, ensuring hypoglycaemia is avoided. For most patients, aim for HbA1C < 7%.
• [1] Metformin
o Start at 500mg (low dose to minimise GI upset) with food and titrate slowly upwards.
o Need to renally adjust!
• [2] If glycaemic control not achieved in 3 months revisit reasons why (e.g. patient compliance etc), add second line
o SGLT2 inhibitors, DPP-4 inhibitors, GLP-1 agonists, sulphonylureas (not as common anymore)
• [3]
o Consider 3rd oral medication
o Consider insulin
Conventional insulin therapy fixed regimen of insulin injections ± oral anti-glycaemic therapy
Intensive insulin therapy
• Basal plus basal plus 1 bolus before meal that has the largest 2 hour post-prandial BGL rise
• Basal bolus regimen basal insulin 1-2 times daily + bolus insulin injection 30-45 minutes
before meals adjusted to pre-prandial blood glucose measurements
o Insulin pump continuous subcutaneous insulin infusion
• Ensure to time insulin in a consistent manner every day
• ?Mixed insulin
o Referral to specialist
• Manage co-morbidities
Follow up
• Glycaemic control: daily BSL monitoring (once daily fasting + symptomatic), GP reviews of HbA1c target <7% (3 monthly)
o self-monitoring of blood glucose is recommended for patients using insulin, or taking glucose-lowering
medicines that can cause hypoglycaemia such as sulfonylureas.
• Complications of diabetes: podiatry (6 monthly), lipids (6 monthly), ophthalmology review (12 monthly), EUC/GFR + urine
a:c (12 monthly)
• Injection advice/counselling look for lipohypertophy which might ↓ absorption
• GP chronic care plan (access to discounted services for ppl with chronic disease)

Appendix: targets
Cholesterol

125
• Cholesterol <4
• LDL <2
• HDL >1
• Triglycerides <2

Glucose
Fasting glucose (mmol/L) OGTT (mmol/L) HbA1c (%)
Normal 3.0 – 6.0 3.0 – 7.7 3.5 – 6.0
Impaired tolerance 6.1 – 6.9 7.8 – 11.0 6.1 – 6.4
Probable diabetes ≥ 7.0 ≥ 11.1 ≥ 6.5
• HbA1c ≤ 42 mmol/mol (6%) is the recommended target for new diagnosis patients treated with lifestyle modifications
and/or metformin, where the risk of hypoglycaemia from these interventions is minimal and so it is safe to aim for
tighter control.
• HbA1c ≤ 48 mmol/mol (6.5%) is the recommended target for patients treated with any glucose-lowering medicine other
than metformin or insulin. This includes sulfonylureas, DPP-4 inhibitors, GLP-1 analogues and SGLT2 inhibitors.
• HbA1c ≤ 53 mmol/mol (= 7%) is the recommended target for a patient initiated on insulin who does not experience
significant or troublesome hypoglycaemia.
• HbA1c ≤ 64 mmol/mol (8%) is the recommended target for patients with recurrent severe hypoglycaemia or
hypoglycaemia unawareness, especially in older patients. This conservative target protects the patient from adverse
consequences of hypoglycaemia, which could include falls.

126
Drug MOA SE CI Additional
Liver and peripheral tissues: ↓ hepatic gluconeogenesis + ↑ insulin sensitivity
[1] Biguanides 1) inhibit mGDP ↓ hepatic - Lactic acidosis - ↓ renal function - No weight
(Metformin) gluconeogenesis and intestinal carb (rare) - ↓hepatic function gain/weight loss
absorption - Nausea and (These both predispose - Less propensity to
Brand name: 2) ↑ insulin sensitivity peripherally vomiting to lactic acidosis) hypoglycaemia
diaformin, 3) ↑ HDL, ↓ LDL - Diarrhoea - active alcohol abuse - Reduce HbA1c by
diabex - Rash - unstable or acute 1.5-2.0%
- Vitamin B12 heart failure - Safe from a
deficiency - paused before cardiovascular
- metallic taste surgery to reduce lactic point of view
(dysgeusia) acidosis
Glitazones Agonist for nuclear receptor PPAR-y → - Weight gain - Heart failure - Rarely used, only
(pioglitazone) regulates gene expression for glucose - Peripheral - Pregnancy in patients with
and lipid metabolism → ↑ lipogenesis oedema and - Severe liver disease aversion to insulin
→ cells dependent on glucose for increases CVD - slow onset of
energy → ↓ hepatic gluconeogenesis + risk action, up to 6
↑ insulin sensitivity peripherally - Headaches, weeks
dizziness - may increase
- Osteoporotic cardiovascular
fractures disease
Pancreas: ↑ insulin secretion
[2] Sulfonylurea Bind to sulfonylurea receptor 1 (ATP- - Bone marrow - Severe renal/hepatic - Not useful in late
(glibenclamide, sensitive K+ channels on beta cells) and toxicity (rare) failure T2DM, it requires
glicazide) prevents K from exiting → - Weight gain - MAOI functioning Beta
depolarisation → ↑Ca2+ influx → (stimulates - B-blockers may mask cells
insulin release from granules → ↑ appetite) tachycardia from - Reduce HbA1c by
insulin secretion - GIT discomfort hypoglycaemia 1.0-2.0%
- Risk of avoid - Some concerns
hypoglycaemia - obesity about CVD risk
(this and insulin)
- disulfuram like
reaction
GLP 1 agonists Stimulates glucagon-like peptide-1 - pancreatitis - severe GI disease - New drug, limited
(exenatide) production in duodenum + proximal (rare) - gall bladder disease research on long
jejunum → ↑ insulin secretion + ↓ - can term SE
glucagon secretion hypoglycaemia - useful where body
Also slow glucose absorption and - N/V (due to weight is a major
decrease appetite (used in obesity) slow movement issue
of food)
[2] DPP-4 Inhibits dipeptidyl peptidase 4 → Limited research w/sulfonylurea or - New drug, limited
inhibitors prolongs action of GLP1 → ↑ insulin - AKI insulin – may cause research on long
(sitagliptin) secretion + ↓ glucagon secretion and - pancreatitis hypoglycaemia term SE
New drug slows down gastric emptying (rare) liver failure - no significant
- can effect on weight
hypoglycaemia
- GI upset and
nasopharyngitis
GIT: ↓ CHO absorption
Alpha- Inhibits alpha-glucosidase at brush - can GI – partial intestinal - Good for patients
glucosidase border of small intestine → ↓ hypoglycaemia obstruction, IBD, major who refuse insulin
inhibitors digestion of carbohydrates into - flatulence hernia, malabsorption
(Acarbose) monosaccharides - diarrhoea -Severe renal
- abdo impairment
- pain/ distension -pregnancy
Kidney: ↑ glucose excretion
[2] SGLT-2 ↓ glucose reabsorption in PCT in -hypoglycaemia - renal impairment
inhibitor kidneys → ↑ glucose excretion via - glucosuria (UTI, - withhold
Na-glucose co- kidneys vaginal thrush) perioperatively (3 days
transport 2 balanitis before surgery) can
inhibitors - volume precipitate
(dapagliflozin) depletion euglycaemic DKA and

127
restarted 3-5 days
post-op
- chronic UTI
NB: anti-hyperglycaemics usually need to be withheld pre-surgery since they will be NBM (no glucose intake). However, also
consider surgical stress response (↑ glucose)

128
Obesity (2020 x 2)
2020 release: A 35 year old previously well woman asks you for a drug to help her lose weight. She has a body mass index of 32 and is clinically well.
She has a family history of type 2 diabetes. What would you advise her?
2020 release: A 53 year old morbidly obese man asks about potential health problems. What would you advise him?
Introductory statement
• Provisional: Given this woman who is clinically obese
• Goals: I would approach this using the RACGP 5 A’s framework which would involve asking her about her symptoms and
motivations, assessing her overall health through focussed history and examination, advise her of the problems of
obesity and the benefits of weight loss, assist by providing a range of pharmacological and non-pharmacological options
to prevent/treat/reverse complications of obesity and finally arrange for follow up.
History
Symptoms
• Weight gain
o Age at onset of weight gain (important RF if <40), previous attempts to lose weight and why they failed, dietary,
history (meals, food eaten, frequency), exercise (frequency, duration, intensity)
o Motivation for losing weight / concerns– includes fertility, family history of obesity related helps to guide
motivational interviewing as it highlights their priorities
Complications
• Screen for obesity related problems according to UpToDate (case 2 has a focus on these)
o Increased morbidity and mortality
o Metabolic
DM, dyslipidaemia
o Cardiovascular
Hypertension, Coronary Artery Disease, heart failure, AF, Cerebrovascular disease, Peripheral vascular
disease, Venous thrombosis
o Increased risk of cancers (*basically lots of cancers)
↑ risk of multiple cancers and ↑ mortality from cancer.
o Respiratory
Obstructive sleep apnoea, Obesity-hypoventilation syndrome
o Gastrointestinal
Non-alcoholic steatohepatitis (NASH), biliary disease (cholelithiasis), GORD
o Renal
Chronic kidney disease, Obesity-related glomerulopathy, nephrolithiasis, urinary incontinence in
women
o Endocrine
Subfertility – due to androgen excess, insulin resistance and hyperinsulinism. Menstrual abnormalities
(women)
o Musculoskeletal
Osteoarthritis, gout
o Psychological
Depression, stigma of obesity, dementia
o Immune
↑ risk of infections
o Skin
Striae, acanthosis nigricans, hirsutism in women from ↑ production of testosterone
o Other
Increased need for medications, therefore risks of medication side effects and polypharmacy
Increased operative and post-operative risk
• Potentially screen for CV symptoms if high CV risk
• Consider mental health history (depression, anxiety, body dysmorphic disorder)
RF/Causes
• Medical medications (e.g. insulin, sulfonylureas, thiazolidinediones, corticosteroids, antidepressants, atypical
antipsychotics), endocrine (hypothyroidism, Cushing’s syndrome, PCOS, hypogonadism)
• Demographics FHx – in particular CVD and metabolic syndrome (can be genetic obesities)
• Lifestyle smoking, alcohol, drugs, diet, exercise
Other
• assess readiness to change
Examination
• Obesity screen: weight/height to calculate BMI, waist circumference.
o BMI – Underweight <18.5; Normal 18.5-24.9; Overweight = 25-29.9; Obese ≥ 30; severe obesity ≥ 40.
129
o Waist circumference (indicator of abdominal obesity) – Overweight if ≥102cm in men and 88cm in women
(≥90cm and ≥80cm for Asians)
Measured on a horizontal plane at the level of the iliac crest, measured from behind patient at the end
of normal exhalation
• Cardiorespiratory exam
o measure BP, cardio exam looking for signs of cardiovascular disease (e.g. LVH)
• Diabetes exam (acanthosis nigricans, fundoscopy, LL neurovascular, Charcot foot)
• Look for findings suggesting a secondary cause of obesity
o Hypothyroidism – goitre
o Cushing’s – proximal muscle weakness, purple striae, osteoporosis
o PCOS – acne, hirsutism
Investigations
Bedside
• ECG (looking for signs of heart disease e.g. old infarcts)
Laboratory
• screen for metabolic syndrome (fasting BSL, HbA1C, fasting lipids), TSH (hypothyroidism), LFTs (fatty liver may have
↑transaminases)
Special tests
• liver ultrasound (fatty liver), sleep study (OSA), Stress test (if high CV risk)
Management
• I would assist this lady with her weight loss goals with both non-pharmacological and pharmacological strategies and an
MDT approach with allied health
• Given that this lady is obese (BMI > 30), pharmacological management is indicated
Non-pharmacological
• Diet – encourage healthy diet consisting of mostly vegetables, some fruit and lean protein. Fatty, sugary foods only to be
eaten on occasion.
o Food diary to monitor energy intake (aim for 800-1000kcal/day)
o Consider strategies to minimise intake e.g. meal planning, brushing teeth right after dinner, eating before going
to parties, food diary, eating nutrient rich, energy sparse foods (e.g. vegetables), reduce alcohol
o dietician referral they are the experts
• Exercise – less potent than dietary restriction for weight loss. moderate exercise for 30 min/day 5-7 days to prevent
weight gain and improve cardiovascular health + incidental exercise (e.g. parking car further and walking, taking the
stairs)
• Behaviour modification e.g. Cognitive behavioural therapy – long-term changes to eating behaviour (modifying and
monitoring food intake) modifying physical activity, control stimuli in environment that trigger eating
Pharmacological
• Indications for ‘LOP it off – liraglutide, orlistat, phentermine’
o BMI ≥ 30 or BMI ≥ 27 with a comorbidity ( >25 with complications for ATSI or Asian), weight loss goals not met
(≥5% at 3-6 months); always an adjunct to comprehensive lifestyle intervention.
• Liraglutide
o MOA – glucagon-likepeptide1(GLP-1) agonist; initially used to treat T2DM. Controls hyperglycaemia without
causing hypoglycaemia or weight gain, but also decreases gastric emptying and appetite, reduces energy intake
and leads to weight loss (therefore has been approved as a weight management drug). Also improves
cardiometabolic RFs (BP, lipids, BSL etc.)
o Dosage – needs to be given as subcut injection.
o Side effects – Gastrointestinal (N/V, diarrhoea, constipation, dyspepsia);
Can be mitigated by gradual dose escalation, usually mild to moderate and will disappear. Other side
effects are symptomatic gallstones and pancreatitis (rare)
o CI – renal insufficiency, ESKD, hepatic insufficiency, pregnancy, history of pancreatitis, major depression,
psychiatric disorder.
o Strong evidence base for cardiovascular safety, but expensive
• Orlistat
o Need to adhere to low fat diet and increase dietary fibre.
o MOA – inhibits pancreatic and gastric lipases → decreases absorption of dietary fat (~30% of oral fat intake is
secreted in the stool
o A/E – fat malabsorption, leading to steatorrhea, oily spotting, flatulence with discharge, faecal incontinence, fat
soluble vitamin deficiencies, calcium oxalate kidney stones
• Phentermine
o Can be administered as combination drug with topiramate

130
o Short term use only due to risk of complications. Start at low dose and increase as needed to prevent side
effects
o MOA = sympathomimetic agent, suppresses appetite.
o CI – history of cardiovascular disease, anxiety, hyperthyroidism, drug/alcohol abuse or dependence,
concomitant treatment with monoamine oxidase inhibitors, pregnancy, breastfeeding.
o A/E– dry mouth, insomnia, agitation, constipation, tachycardia.
o Other – requires careful BP monitoring
Interventions
• Devices
o Indications May be used if pharmacological methods unsuccessful, unable to undergo bariatric surgery or as
a bridging therapy before surgery.
o Procedures Laparoscopic adjustable gastric banding, electrical stimulation, electrical stimulation systems
(block transmission of nerve signals in the vagus nerve), intragastric balloon systems, gastric emptying systems,
hydrogels (expand stomach and intestines to feel satiety)
Liposuction not recommended
• Bariatric Surgery
o Indications Indications are BMI ≥ 40 or BMI ≥ 35 with comorbidity and above
treatment does not lead to weight loss ≥ 5% at 3-6 months; adjunct to
comprehensive lifestyle intervention. Not indicated in this patient
o Options include
Sleeve gastrectomy (most common): large part of the greater curvature
is removed, so that the remaining stomach resembles a sleeve
Roux-en-Y gastric bypass (2nd most common): gastrojejunostomy or
esophagojejunostomy
Long-term
• Treat co-morbidities!
o Especially metabolic syndrome complications
o In this patient, optimise their diabetes treatment
• Follow-up
o Recidivism, the regaining of lost weight, is a common problem in treating people with obesity. The body appears
to have a "set point" of adipose tissue mass, and after weight loss, counter-regulatory hormones are secreted to
re-establish the higher body weight. In addition, the reduction in energy expenditure caused by weight loss
itself contributes to the difficulty in maintaining weight loss
o Need to manage patient expectations
Need to advise that gains are incremental, do not massive progress except with bariatric surgery
Looking at 5-10% body mass change over a few months aim for sustained change over crash dieting
and rebounding
o Arrange follow up to monitor and suggest, frequent self-monitoring

131
Hyperthyroidism (2020)
A 26 year old female presents with tiredness, palpitations and heat intolerance. She has a diffusely enlarged thyroid. How would you assess her?
Pathophysiology
• Grave’s disease (excitation of gland)
o B and T cell-mediated autoimmunity → production of stimulating immunoglobulin G (IgG) against TSH-
receptor (TRAb; type II hypersensitivity reaction) → ↑ thyroid function and growth → hyperthyroidism and
diffuse goiter
• Hashimoto’s disease (destruction of gland)
o Cellular (especially T cells) and humoral immune responses are activated. → active B lymphocytes produce
antibodies towards thyroid peroxidase (TPO) and thyroglobulin (Tg) → destruction of thyroid tissue
Early stage goitre, ± hashitoxicosis
Late stage normal sized or small thyroid, hypothyroidism
Introductory statement
Provisional Diagnosis:
• Given diffusely enlarged thyroid and signs of hyperthyroidism in a young woman, this is likely to be Grave’s disease (most
common cause given her age group)
DDx: Main DDx for hyperthyroidism include:
• Destruction of the thyroid gland NB: the below thyroiditis should have the thyroid phases of hyper hypo euthyroid
o Subacute thyroiditis
Granulomatous/de-Quervain’s (painful) usually infection viral (most likely) or bacterial/suppurative
thyroiditis
Lymphocytic/Non-granulomatous (painless) post-partum, autoimmune, drugs (e.g. lithium,
amiodarone, etc.)
o Other types of thyroiditis (shouldn’t be painful)
Hashimoto’s thyroiditis associated with thyrogastric cluster
Radiation thyroiditis
Trauma/palpation thyroiditis
Contrast-induced thyroiditis
• Hyperfunctioning thyroid gland
o Neoplastic thyroid causes
Benign toxic adenoma, toxic MNG, TSH producing pituitary adenoma
Malignant thyroid malignancy (often cold nodules) which can be papillary, follicular, medullary and
anaplastic
o Grave’s disease (painless)
o β-HCG mediated hyperthyroidism (hydatidiform mole, choriocarcinoma)
• Extrathyroidal hormone production
o struma ovarii (ovarian teratoma secreting TSH) or metastatic follicular thyroid carcinoma
• non-thyroid causes e.g. cardiac
Goals
• Rule out a thyroid storm or airway obstruction resuscitation if necessary
• Perform a H/E/I to determine the aetiology of hyperthyroidism
• Manage appropriately with pharmacological and surgical interventions.
ABCDE Assessment
• Airway Exclude airway obstruction (stridor, hoarse voice) and respiratory distress
• Exclude thyroid storm
o Symptoms
Heat intolerance high fever e.g. >40oC, volume depletion
Neuropsychiatric agitation, confusion, delirium, seizures, coma
Cardiovascular tachycardia, HTN, AF, CHF, cardiac arrythmia, death from CV collapse
Severe N/V, diarrhoea
o Management
Initial hyperthyroidism management
• B-blockers, anti-thyroid drugs to inhibit thyroid peroxidase e.g. carbimazole or
propylthiouracil (which also blocks T4-T3 conversion)
Supportive treatment (ICU, fluids, ice packs/cooling blankets)
Potassium iodide/Lugol’s iodine (paradoxical ↓ in T4 production in response to iodine load that is
transient for 5-10d according to Wolf-Chaikoff effect + ↓ peripheral conversion of T4 to T3)
Glucocorticoids inhibits peripheral conversion of T4 to T3, ↓ TSH

132
• NB: In hypothyroidism, glucocorticoids due to association with Addisonian disease
Plasmapheresis as a life-saving treatment, rarely needed
History
Symptoms ± complications
• Symptoms of hyperthyroidism see appendix for full list
o ↑ basal metabolic rate, ↑ sympathetic nervous stimulation (eyes, CV, MSK), goitre, endocrinological,
neuropsychiatric, paeds specific
• Complications
o Obstructive symptoms (high risk) – stridor, dysphonia, dyspnoea, dysphagia
Risk factors/causes
• Risk factors for Graves
o Demographics (female aged 20-40)
o Medical (pregnancy, autoimmunity incl. thyrogastric cluster of autoimmune diseases e.g. DM, coeliac disease)
o Lifestyle (triggers incl. infectious agents, stress, environmental factors such as smoke and irradiations)
Differentials
• Subacute thyroiditis
o Granulomatous painful, often viral infection can be bacterial (associated skin changes)
o Lymphocytic non-painful, drugs (lithium, amiodarone, etc.), autoimmune disease, post-partum
• Other thyroiditis radiation, trauma,
Other
• Factors affecting treatment
o Chronic airflow limitation or asthma (β blockers contraindicated in severe asthma)
o Desire for pregnancy (carbimazole CI, use PTU instead)
Examination
• Vitals ↑HR, rhythm disturbance, widened PP due to ↑ CO
• Thyroid Exam
o Signs of obstruction – dysphonia, dyspnoea, dysphagia, Pemberton’s sign (substernal goitre cause thoracic inlet
obstruction
o Examine goitre
Feel size of goitre + tenderness, swallow test (normal = symmetrical elevation), tongue protrusion
(thyroglossal cyst will rise with tongue protrusion; goitre will not)
Grave’s diffusely enlarged (symmetric), non-tender thyroid gland, bruit may be present
Subacute granulomatous thyroiditis exquisitely tender, diffusely enlarged gland (usually with a viral
illness)
Hashimoto’s thyroiditis non-tender, rubbery thyroid with moderate and symmetrical enlargement
Multinodular goitre bumpy, irregular and asymmetric
Toxic adenoma single nodule and otherwise atrophic gland
o Signs of hyperthyroidism CNS (nervous, insomnia, irritability, hyperactivity, tremor), hyperdynamic
circulation, neurological (hyperreflexia), MSK (proximal myopathy), nails/fingers (thyroid acropachy), skin
(excessive sweating, heat intolerance)
o Cervical LN: suggestive of malignancy
o Grave’s specific
Eyes exophthalmos/proptosis, ophthalmoplegia, lid lag
Legs Pretibial myxoedema
NB: Triad of grave’s disease diffuse goitre, ophthalmopathy, pretibial myxoedema
• Cardiovascular
o Look for signs of CVD
• Systems review
o Looking for signs of infection subacute thyroiditis
Investigations
Bedside
• ECG (tachycardia, AF), B-hCG
Bloods
• FBC, TFTs, thyroid antibodies, ESR (exclude subacute thyroiditis), lipid profile and BSL (complications, monitoring)
o TFTs - ↓ TSH, ↑ T4, T3 level is usually not required but will be high
o Thyroid antibodies – need to specify TRAbs as it will not be given if you order “thyroid antibodies”
Graves’ disease – ↑ TRAbs - TSH receptor antibodies (specific), ↑ anti-TPO and anti-Tg (nonspecific)
Hashimoto’s – anti-thyroid peroxidase antibodies (anti TPO; in 90%) anti-Tg (present in 50%)
Imaging
• Thyroid scintigraphy (radioactive iodine uptake test)
o Contraindicated in pregnancy
133
o ↑ if de novo synthesis of hormone
Grave’s diffuse
Toxic adenoma a hot nodule
Toxic MNG heterogenous hot/cold (patchy)
o ↓ if destruction of thyroid, extra-thyroid source or factitious hyperthyroidism
e.g. thyroiditis (damaged), Levathyroxine overdose, pituitary, ovarian cancer
• [alt.] Thyroid colour doppler US - Alternative to scintigraphy in pregnant women
Management
• Treatment would depend on the severity of the patient’s hyperthyroidism. Call for senior help and endocrine review
Emergent
• Thyroid storm management as above
Supportive
• B-Blocker e.g. atenolol symptomatic treatment e.g. palpitations, tremor
• Anti-thyroid drugs (thionamides) Until patient is euthyroid and then slowly taper and stop
o Choice of drug
Carbimazole not childbearing age as no risk of liver failure.
PTU if planning pregnancy but can cause liver failure (also tastes awful).
NB: PTU has a shorter half-life, so dosing is BD or TDS compared to carbimazole which is once daily
NB: If pregnancy planned – consider radioiodine treatment or surgery planned months ahead to avoid
the need for thionamides during pregnancy
o MOA carbimazole inhibit thyroid hormone synthesis by preventing thyroid peroxidase (TPO) from coupling
and iodinating tyrosine residues on thyroglobulin, PTU also inhibits conversion of T4 to T3.
Carbimazole is a pro-drug which is metabolised to form methimazole
o A/E minor (rash, hives, arthralgias), major (risk of agranulocytosis, vasculitis, hepatitis)
Agranulocytosis can occur at any time, but usually within 6 weeks After starting, do FBC + LFT after
6w
Definitive
• Radioiodine 131 causes destruction of thyroid follicular cells, as the radioactive iodine emits β radiation.
o NB: Thyroid cells are the only cells in the body that absorb iodine.
o Advantages Preferred over surgery as cheaper and lower risk of permanent hypothyroidism.
o A/E permanent hypothyroidism (occurs in most patients), radiation precautions for several days, CI
pregnancy and breastfeeding, increases size of goitre before reducing it (therefore risky in goitre), new
onset/exacerbation of graves ophthalmopathy
• Surgery Partial or near-total thyroidectomy + replacement thyroxine (T4)
o Indications Rarely done as radioiodine treatment preferred. Surgery preferred in large or obstructive goitres
or above treatments CI or refractory
o A/E permanent hypothyroidism, risk of iatrogenic hypoparathyroidism (monitor for hypocalcaemia after
surgery; may be permanent), thyroid storm (need to make sure patient is euthyroid first)

Long-term
• Address consequences of hyperthyroidism Skeletal health
• Monitoring clinical assessment, measurement of T3/T4 levels.
o Do not use TSH to follow up as it may remain low for weeks or even months even when the patient is
biochemically euthyroid or even hypothyroid

NB: If thyroiditis is the cause of hyperthyroidism, see Surgery: subacute thyroiditis:


• Don’t need other thyroid treatments
o NO Radioactive iodine therapy not effective, as uptake is low
o NO Antithyroid drugs (TPO inhibition) e.g. carbimazole or PTU No new thyroid hormone is produced because
follicular cell function is impaired by inflammation; therefore antithyroid drugs, which actually target the
synthesis of thyroid hormone, are ineffective

134
Appendix: DDx of hyperthyroidism
• Hyperfunctioning thyroid gland
o TSH receptor stimulation
Graves’ disease (∼ 60–80% of cases)
TSH-producing pituitary adenoma (thyrotropic adenoma)
β-hCG-mediated hyperthyroidism (hydatidiform mole, choriocarcinoma)
• structurally similar to TSH
o Unregulated thyroid hormone secretion
Toxic MNG (∼ 15–20% of cases)
• multinodular goitre with TSH-independent functioning of some nodules
Toxic thyroid adenoma (3–5% of cases)
• Follicular destruction transient hyperthyroidism
o Subacute Thyroiditis
Same as Subacute granulomatous thyroiditis (de Quervain thyroiditis) painful
hypothyr Subacute lymphocytic thyroiditis (post-partum thyroiditis, autoimmune, drug-induced) painless
oidism • Drug-induced thyroiditis (e.g., amiodarone, lithium)
o Hashimoto thyroiditis (hashitoxicosis)
o Contrast-induced thyroiditis (Jod-Basedow phenomenon)
acute o Radiation thyroiditis
o Palpation thyroiditis: due to thyroid gland manipulation during parathyroid surgery.
• Exogenous hyperthyroidism
o Iatrogenic
o Factitious
• Ectopic (extrathyroidal) hormone production
o Struma ovarii (ovarian teratoma that secretes TSH)
o Metastatic follicular thyroid carcinoma

Appendix: Symptoms of hyperthyroidism


• The function of the thyroid is to activate nuclear transcription of a large number of genes
• ↑ basal metabolic rate (due to ↑ expression of Na+/K+ ATPase in many tissues) + ↑ sympathetic
o Heat intolerance, excessive sweating (moist, warm skin)
o Weight loss despite increased appetite
o Frequent bowel movements
o Weakness, fatigue
• ↑ sympathetic nervous stimulation
o Eyes:
lid lag (spasm due to adrenergic overactivity), lid retraction ("staring look"),
Graves ophthalmopathy
o Cardiovascular (sympathetic effect + direct effect of thyroid hormones on cardiac myocytes)
Tachycardia
Palpitations, irregular pulse (due to atrial fibrillation/ectopic beats)
Hypertension with a widened pulse pressure
Cardiac failure: Elderly patients often present with features of cardiac failure (e.g., pedal oedema,
dyspnoea on exertion).
o Musculoskeletal
Fine tremor of the outstretched fingers
Myopathy with muscle weakness, particularly in patients > 40 years of age
Osteopathy: osteoporosis, fractures (in the elderly)
Hyperreflexia
• Goiter
o Diffuse, smooth, nontender goiter; often audible bruit at the superior poles
o Also seen in subacute thyroiditis, toxic adenoma, and toxic MNG
• Endocrinological (similar to hypothyroidism but different mechanism)
o ♀: Oligo/amenorrhoea and anovulatory infertility
o ♂: Gynecomastia, decreased libido, erectile dysfunction
• Neuropsychiatric system: depression, anxiety, agitation, insomnia, emotional instability
• In children,
o Bone growth: if hyperthyroid child we see excessive growth early, however the bones mature more rapidly and
epiphyses close at an earlier age so eventual height may not be that much
o Maturation of nervous system: important in foetal life + first few years of postnatal life

135
Clinical triad of Grave’s Disease
• Diffuse goitre - above
• Ophthalmopathy – fibroblasts in tissue around eyes stimulated by thyroid-stimulating immunoglobulin, make
glycosaminoglycans that buildup in tissue
o Exophthalmos +/- corneal ulcers (exophthalmos dries eyes)
o Weakened muscles lid lag, lid retraction
• Dermopathy –
o Pretibial myxoedema – non-pitting oedema and firm plaques on the anterior/lateral aspects of both

Appendix: Aetiologies of hypothyroidism


• Congenital
o Sporadic (85%)
o Hereditary (15%)
• Acquired
o Primary hypothyroidism
Subacute thyroiditis (late stage) as above
• Subacute lymphocytic thyroiditis
• Subacute granulomatous thyroiditis
Other thyroiditis
• Hashimoto thyroiditis
o The most common cause of hypothyroidism in iodine-sufficient regions
o Associated with thyrogastric cluster (e.g., vitiligo, pernicious anaemia, type 1
diabetes mellitus, and systemic lupus erythematosus)
• Others above: Radiation thyroiditis, Trauma/palpation thyroiditis, Contrast-induced thyroiditis
Nutritional (insufficient intake of iodine): the most common cause of hypothyroidism worldwide,
particularly in iodine-deficient regions
Riedel thyroiditis (autoinflammatory fibrous change)
Wolff-Chaikoff effect (transient ↓ in thyroid hormone after iodine ingestion)
o Secondary hypothyroidism:
pituitary disorders (e.g., pituitary adenoma) → TSH deficiency
o Tertiary hypothyroidism:
hypothalamic disorders → thyrotropin-releasing hormone (TRH) deficiency (very rare)

136
Dermatology
Drug reaction (2020)
A 70 year old woman is admitted with hypertension and cardiac failure. She is commenced on indapamide. Her admission is complicated by a
urinary tract infection. She is treated with amoxicillin as she had a previous drug reaction to cotrimoxazole (sulphamethoxazole, trimethoprim). On
the third day of amoxicillin treatment she becomes febrile and develops a generalised maculopapular rash. How would you assess and manage her?
Introductory Statement
Provisional Diagnosis: Given this fever and rash in a patient recently commenced on a new medication, my provisional diagnosis is
a delayed hypersensitivity reaction to the amoxicillin (commonly type IV hypersensitivity)
DDx:
• Sepsis
• Systemic drug reaction – anaphylaxis (severe delayed anaphylactic reaction)
• Cutaneous drug reaction
o Serious adverse skin reactions
acute generalised exanthematic pustulosis (AGEP) causes widespread erythema with pustules.
Stevens-Johnson Syndrome (SJS)/toxic epidermal necrolysis (TEN) defined when >30% skin involved
drug rash with eosinophilia and systemic symptoms (DRESS)
o Less serious drug reaction –exanthema most commonly due to viral infections (e.g. CMV, EBV etc.)
Goals
• Assess and resuscitate the patient via primary survey
• Determine cause of rash, exclude DDx
• Cease the amoxicillin and treat appropriately
ABCDE Assessment
• ensure that the patient is not in anaphylactic shock and haemodynamically stable.
History
Symptoms ± Complications
• History of rash
o skin pain (itchy is less sinister) + morphology (blistering is more sinister), onset (temporal relationship with
medication)
• Symptoms of severe reaction: high fever, facial oedema, mucositis, blistering, skin tenderness
• Presence of systemic symptoms – fever, sore throat, arthralgia
Differentials
• Infective symptoms
• Anaphylaxis symptoms angioedema, respiratory distress etc.
• Sepsis high fever, organ involvement etc.
Risk Factors/Causes
• Sick contacts
• Family history – family history of allergies, especially if Han Chinese (due to association with severe drug reactions)
• Drug history - need to know all medications, including when they started each one, recent changes in medications or
dose, and whether they have taken the drug previously (if sensitised, typical time to presentation decreases)
o Determine indications for these drugs and if they can be ceased
o Drugs commonly implicated in severe cutaneous drug reactions –
mnemonic = BERNSA: antibiotics (penicillins, cephalosporins, sulphonamide antibiotics), anti-epileptics
(phenytoin, carbamazepine, oxycarbazepine, phenobarbitone, lamotrigrine), anti-retrovirals (abacavir,
nevirapine), NSAIDs, sulfa drugs, allopurinol
o Testing mandatory for certain patient groups before starting carbamazepine and oxcarbazepine (HLA-B*1502),
and allopurinol (HLA- B*5801); mandatory for all patients before starting abacavir (HLA- B*5701).
o NB: ACEI + allopurinol can cause SJS
Examination
• Dermatological exam
o General inspection – do they look well
o Appearance of rash
Size, distribution (% of body surface using rule of 9’s or palm of patient is 1%), shape, border, colour,
morphology, texture, temperature, mobility, consistency, tenderness, blanching, progression
Specifics
• Delayed hypersensitivity maculopapular rash ± itch
• Amoxicillin rashes are usually morbilliform (like measles which is a macular rash)
• SJS mucositis ± TEN (>30% involvement)
• Erythema multiforme (NSAIDs, sulphonamides, antiepileptics, etc.) characteristic target
lesions
137
o Red flags
Signs of SJS – Nikolsky sign (desquamation when lateral pressure is applied)
Mucosal involvement – eyes (gritty eyes, photophobia), mouth, genitals
o Other – arthritis, arthralgia, lymphadenopathy, crepitations
Investigations
• Not usually required but if severe-suspicious
Laboratory
• FBC (eosinophils ↑ DRESS, leukopenia SJS, leucocytosis AGEP), EUC (systemic involvement), LFT (can be deranged in
severe reactions)
• Blood cultures exclude sepsis
• ± viral screen
• ± later patch testing, intradermal testing, in vitro testing
Invasive
• Punch biopsy (often non-specific, but can exclude the dangerous causes)
Management
• If a severe drug reaction was suspected I would consult a senior clinician - immunologist/allergy specialist or
dermatologist (very good at identifying what the rash is and the likely cause)
Supportive
• Cease the suspected drug if possible stopping drugs early reduces mortality in severe reactions
o If a likely drug is not identified, stop non-essential drugs (e.g. NSAIDs, OTC medications, alternative therapies)
• Document ADR on medication chart, patient notes and discharge summary
• Patient education – educate regarding allergy and inform the patient that the rash may worsen and will take 7-14 days to
resolve. Give patient name of medication (both brand/ generic names) and advise them to avoid the medication
• Emollient use
Definitive
• If mild
o Topical or oral corticosteroids and antihistamines [Diphenydramine (benadril), hydroxyzine or cetirizine (Zyrtec)]
• If severe or internal involvement
o Systemic steroids + MDT management
o Replace fluid and electrolytes
o Analgesia
o Maintain body temperature
o Monitor and treat organ dysfunction
o Specifics
AGEP - often responds to short course of corticosteroids
SJS - good wound care e.g. nursing care in a burns centre (Concord/RNS), IVIG.
• Consult ophthalmology, urology, pain team, dietician (lips affected) as multiple organs affected
• Treat UTI appropriately
Follow-up
• Counsel on future use of penicillins Depends on severity of drug reaction
o ± cross-reactivity with other drugs in future

Appendix

138
Infectious Diseases
HIV
A 30 year old man is recently diagnosed with HIV following a routine test as part of a sexual health assessment. He is asymptomatic. How would you
assess and manage him?
Pathophysiology
• HIV infects CD4+ lymphocytes
Introductory Statement
Provisional Diagnosis: This patient has HIV infection.
Goals: My approach for this patient would be
• Reassure patient that consultation is entirely confidential and that developments in HIV research have made it a very
manageable disease
• Assess patient via a targeted hx, ex, Ix to determine patients current health, stage of his infection and screen for any co-
existing illnesses
• Manage via non-pharmacological and pharmacological means e.g. counselling and pharmacotherapies to prevent
progression to AIDS and prevent spread to other individuals.
History
Symptoms ± Complications
• Sx – asymptomatic acute seroconversion illness AIDS (AIDS defining illness or CD4 <200)
o Acute seroconversion illness – viral sx (fever, lymphadenopathy, sore throat, rash, myalgia/arthralgia, GI
symptoms, oropharyngeal sx such as sore throat/ulcerations/dysphagia)
o Clinical latency – may still be asymptomatic
Non-AIDS defining conditions
• Chronic subfebrile temperatures, lymphadenopathy, opportunistic infections (oral candidiasis,
vaginal infections), oral hairy leukoplakia (lateral borders of tongue), chronic diarrhoea, skin
manifestations (molluscum contagiosum, warts, exacerbations of psoriasis, shingles)
o AIDS-related
AIDS defining conditions
• https://next.amboss.com/us/article/1o02aS#Z9ee3f30cf6167ef5113bed457533d70e
• E.g. candidiasis of oesophagus, encephalopathy, kaposi’s sarcoma, non-hodgkin lymphoma,
mycobacterium TUB, pneumocystis jirovecii pneumonia, etc.
Risk Factors/Causes
• Risk factors (for transmission)
o sexual hx (esp. protection, higher risk behaviours e.g. sex workers/under influence of drugs)
o needle (IVDU sharing/needlestick injury)
o blood transfusions
o vertical (high maternal load)
Other
• Medical hx – comorbidities, immunisations, regular meds (esp. CYP450 metabolised can interact with HAART), Screen
for related conditions – STIs, other BBVs (e.g. hep B, hep C)
• Psych – occupation, social supports, drug/EtOH
Examination
• Most likely normal unless advanced HIV/AIDS
• General – cachexia, wasting syndrome
• Infections – look in mouth (leukoplakia caused by EBV, HSV-related vesicular rash), listen to chest (Pneumocystis and TB
pneumonia)
• Cancers
o AIDS-defining – Kaposi sarcoma (skin), NHL (lymphadenopathy, hepatosplenomegaly)
o Non-AIDS-defining – SCC (anus)
Investigations
• Mostly bloods to stage HIV and assist selection of antiretrovirals.
Diagnostic
• Screening test
o Combination 4th gen HIV-1/2 antigen and antibody test (ELISA)
o OR Rapid point of care test
• Confirmatory
o HIV-1/2 antibody-ONLY differentiation immunoassay
o OR Western blot
o OR HIV RNA PCR useful during window period

139
Laboratory
• Staging serum viral load (high in recently infected/end-stage disease), CD4+ T-cell count (flow cytometry), CD4:CD8+
ratio, HIV resistance testing
• Assoc. diseases chlamydia/gonorrhoea (urine PCR), hep A/B/C/syphilis/toxoplasmosis/CMV/EBV (serology), TB
(Quantiferon Gold)
• HAART monitoring EUC/LFT (metabolism/ baseline for S/E), FBC/BSL/cholesterol (baseline for S/E)
Imaging
• Consider CXR (baseline)
Management
• MDT – immunology, psychology/counsellors, support groups
Supportive
• PENIS SNAP
o Patient Education – understanding of disease (“virus attacking immune system esp. CD4+ T cell, important in
fighting infection), prognosis (incurable but with modern treatments still lead a long + fulfilling life!)
o Notifiable disease – advise NSWH, they must be made aware of diagnosis (de-identified) under Public Health Act
2010
o Intimate partner contact tracing – encourage to have conversation re. HIV+ status with recent sexual partners
(also children conceived after pt infected need to be tested)
Consider PReP for sexual partner if they are in a couple with a sero- discordant partner (reduces risk of
HIV acquisition)
o Sexual education + minimise spread – barrier contraception + safe sex measures + avoid sharing needles
Disclosure is not necessary in NSW if reasonable precautions are taken (e.g. barrier contraception)
If unprotected sex disclosure is mandatory
o SNAP
• Vaccinations – (pneumococcal, HBV, MMR) give early as possible
o use live vaccines cautiously if CD4 count < 200 (e.g. MMR, polio)
o Pneumococcal polysaccharide vaccine (Pneumovax) every 5-6 years, Influenza vaccine yearly
o Hepatitis B vaccine if not already immune

Definitive
• treat with HAART has multiple benefits including
o 1) reduction in HIV associated morbidity and mortality + improved QOL; 2) prevention of HIV transmission to
sexual partners and 3) prevents HIV transmission from an infected mother to baby
• Debate about when to start HAART (e.g. early vs based on CD4 count) according to UpToDate, current
recommendations are to start early regardless of CD4 count
• [1] 2 NRTIs (e.g. abacavir + lamivudine) + INSTI (e.g. dolutegravir) OR NNRTI OR protease inhibitor
o Continued indefinitely
o follow up 2-4wks to check compliance, screen S/E incl. bloods and depression
• ± prophylactic treatment of opportunistic infections
o Pneumocystic pneumonia (PCP) – Pneumocystis jiroveci, Toxoplasma gondii, Mycobacterium tuberculosis,
Atypical mycobacteria – Mycobacterium avium complex (MAC)
Long-term
• Monitoring
o Clinical status and treatment adherence
o HIV viral load
o CD4 cell count
o Toxicity – monitor FBC, EUC, LFT
o Risk factors for CVD – smoking, diet, exercise, BP, weight, BSL, lipids
• Treatment failure
o Causes of deterioration of a patient with HIV/AIDS
opportunistic infections, neoplasms, medication related toxicities, co-infections (e.g. HBV, HCV, STIs),
non-AIDS related comorbidities (e.g. cardiovascular, renal, hepatic, neurocognitive, bone disease
o Defined clinically (HIV progression), immunologically (failure to increase CD4 count by 25-50 over 1st year of
treatment or CD4 decrease >100 over 1 year) or virologically (failure to achieve viral load <40 copies/mL after 6
months)
o Assess adherence, assess drug interactions, perform resistance testing, rule out opportunistic infections, rule
out marrow suppression, construct new 3 drug regimen

Appendix: AIDS defining Illness


• Infections – oesophageal Candidiasis, extrapulmonary Coccidioidomycosis, extrapulmonary Cryptococcosis,
Cryptosporidiosis >1 month duration, CMV disease outside lymphoreticular system, herpes simplex infection >1
140
month/visceral herpes simplex, recurrent Salmonella bacteraemia, extrapulmonary histoplasmosis, mycobacterial
disease (disseminated or extrapulmonary), Tuberculosis infection, PCP, recurrent pneumonia (≥2 episodes/year),
progressive multifocal leukoencephalopathy, cerebral toxoplasmosis
• Malignancies – Kaposi sarcoma (HHV-8), lymphoma (primary CNS, immunoblastic or Burkitt’s lymphoma; EBV), invasive
cervical or anal cancer (HPV 16, 18)
• Miscellaneous – HIV-related encephalopathy, wasting syndrome due to HIV

Appendix: Pharmacology

141
Travel Preparation (2020)
An 18 year old man is leaving for a year’s holiday trekking through South East Asia. What advice would you give him with relation to staying healthy?
Introductory Statement
Provisional Diagnosis: In this young man who may be traveling through several developing countries endemic to malaria/other
infectious diseases
Goals my aims are to
• 1) assess the patient’s risk by taking a hx of his general health + travel plans (any endemic conditions) + considering
investigations e.g. serology
• 2) manage the pt with prophylactic vaccinations + counselling on safe travel practices to minimise health risks and
financial risks
History
• Travel plans – location, rural area? Animal contacts?
• PMHx
o Current comorbidities Regular meds
o Allergies esp. to previous vaccinations
o Immunisation record
o Immunosuppression
o DVT risk
Investigations
Laboratory
• Serology – check if boosters required e.g. Hep B
Management
Prophylaxis
• Communicable risks – vaccine preventable
o Hepatitis A and B, Typhoid, Yellow fever, Cholera, Diptheria/tetanus/pertussis, Measles/mumps/rubella,
Influenza, Japanese encephalitis, Rabies, Meningitis, Polio, Tick borne encephalitis, Tuberculosis
• Communicable risks – no vaccines
o Dengue fever, HIV/other STIs, Schistosomiasis, Amoebiasis/giardiasis, Avian influenza/SARS
• Risk in visiting country e.g.
o India – Hep A, typhoid
NB: both are faecal-oral
o Nepal/SEA – above + Hep B, malaria
• Ensure up-to-date – MMR, DPT, varicella, flu
• Common vaccines available
o Hepatitis A
o Combined Hep A / Typhoid (developing countries especially southeast Asia)
Recommended for travellers with exposure to potentially contaminated foods and beverages and
persons with intimate exposure to a chronic carrier of Salmonella typhi
o Japanese encephalitis (south east Asia)
o Rabies (rabies is endemic but several countries are considered rabies-free)
Recommended for travellers visiting countries with a high occurrence of animal rabies and low
availability of antirabies biologics (e.g., countries in Africa, South America, and Southeast Asia)
Depends also on planned activities (e.g., caving, wildlife work in rabies-epizootic areas)
o Yellow fever (Africa/Americas)
Usually transmitted through bites of infected mosquitoes
o Cholera
Only recommended if travelling to area with a known outbreak. May be recommended as a vaccine for
traveller’s diarrhoea
o BCG
TB but only 50% effective
• Malaria
o Should be initiated before traveling to regions with a high risk of malaria: e.g., tropical Africa, Asia, and Latin
America
o Drug of choice is based on the area
Areas with P. falciparum
• If chloroquine-sensitive P. falciparum: chloroquine
o Need to take 1 week before and continue 4 weeks after
• If chloroquine-resistant P. falciparum (most malaria endemic regions): malarone (atovaquone-
proguanil), doxycycline, mefloquine
142
o Malarone 1 or 2 days before and continue 1 week after
o Doxycycline 1 or 2 days before and continue 4 weeks after
o Mefloquine 2 weeks before and continue 4 weeks after
Areas without P. falciparum (some areas of Central/South America, Mexico, China, South
Korea): primaquine
o Agents that are safe during pregnancy: chloroquine, mefloquine
o Prophylactic medication cannot prevent infection but suppresses the clinical course and symptoms by killing the
parasite before it can cause a severe infection. There is no prophylactic medication that provides protection
against all potential parasites.

Counselling
Safe travel practices
• Look up smart traveller + CDC website for latest health info give printouts to pt
• Health insurance + know where to get healthcare!
• Food + drink – drink bottled water, avoid raw vegetables/meat or unpasteurised dairy
• Prevent insect bites – cover exposed skin + insect repellent (with DEET)
• Dog bite management
• Medications and their use – bring regular meds, paracetamol, repellent, ± anti-diarrhoea (loperamide, hydralyte), ± anti-
nausea (ondansetron), ± antibiotics (stat dose of azithromycin for diarrhoea)
• Accessing local medical services
• ± DVT prevention – hydration + mobilise on plane
• ± Sexual activity – use barrier contraception
• ± altitude illness importance of descent for symptomatic individuals ± acetazolamide prophylaxis (↑ bicarbonate
excretion in urine)
• ± motion sickness

143
Miscellaneous
Needle Stick Injury (2020)
2020 release: A 25-year-old intern is attempting to insert an IV cannula but fails and accidentally sticks himself after withdrawing the cannula. How
would you manage this situation?

You are the night intern who has been asked to replace an IV cannula. During the process, you accidentally prick yourself and draw blood. Describe
the process that you should undergo.
Introductory Statement
Provisional Diagnosis:
• A needle stick injury is a protocol managed situation that requires immediate action and reporting. There are three
infections that can be transferred via this route, HIV, HBV and HCV. The risk of transmission is quite low.
Goals: My aims are to
• Provide immediate emergency first aid measures for safety
• Complete appropriate documentation
• Assess risk of transmission of blood borne viruses
• Appropriate follow up investigations and behavioural and medical management.
Resuscitation
• Ensure safety
o Dispose of sharps in sharps bin
• First aid – immediately wash exposure site with soap and water for 5-10 minutes, if eye injury wash gently with water or
normal saline
o Let wound bleed freely (don’t milk)
• Inform seniors and complete documentation
History and Investigations
Symptoms ± Complications
• Note event details (important for medicolegal reasons)
o type of exposure, volume of blood, depth of injury, any other bodily fluids (CSF, vaginal secretions, semen),
significant mucus membrane exposures, whether skin is intact or not, protective factors in place (wearing
gloves)
• Assess potential for transmission of BBV
o Review patient’s BBV status after asking permission
Review patient’s BBV status and medical record (NB: HIV is not on eMR, need to call infectious
diseases)
Document and obtain consent from patient to look back at their serology and potentially obtain
serology (HBV, HCV, HIV)
Order bloods – HBV (sAb, sAg, cAb), HCV antibody (HCV RNA if suspected acute HCV infection), HIV
antigen and antibody (ELISA)
o Review intern’s BBV status
Review immune status ± serology (considered immune to HBV if anti-HBs level ≥10 mIU/mL)
Order bloods – HBV (sAb, sAg, cAb), HCV antibody, HIV antigen and antibody (ELISA), B-HCG if female
Repeat bloods post-exposure at 6 weeks, 12 weeks ± 24 weeks (if HBV/HCV)
Management
• For urgent management refer to ED
• At POWH, contact the CHESS (Centre for hospital epidemiology and staff services) occupation exposure management RN
Supportive
• Full documentation + incident form
• Tell NUM that It happened may have a procedure manual and who you should see potentially e.g. ID reg, ED, chest
clinic etc.
• Intern education
o Safe needle practices
o Serology may take 1-2 days to come back so in that time no unprotected sex. No blood/sperm donations. Seek
medical advice if acute illness, pregnancy or breastfeeding
o If there is any suspicion that patient could still be seroconverting, consider alternative diagnostic method and
also continue safe practices for intern as outlined above
o Modify work practices according to local protocol
Definitive
• If source negative
o + no recent risky behaviour that places them at risk of BBV No further action required
• If source is unknown or positive
144
o HBV – depends upon recipient’s HBV status. Considered immune to HBV if anti-HBs level ≥10 mIU/mL
HBV vaccine + HB immunoglobulin (IM 400 IU stat dose)
o HCV – no PEP course available, but treatment of acute HCV infection is highly effective
o HIV – recommended PEP course of 28 days. Any medical officer can prescribe 3-7 days of PEP start pack.
Specialist must prescribe the remaining regimen.
Undetectable viral load: 2 drug regimen – Tenofovir disoproxil fumarate/emtricitabine 300mg/200mg
daily (Truvada) or Tenofovir 300mg + lamivudine 300mg daily
Unknown viral load: 3 drug regimen – Truvada + dolutegravir 50mg daily/raltegravir 400mg
BD/rilpivirine 25mg daily

145
Appendix
MET Call
• MET Call options
o Call the nurse!
o Push the button
o Patient bell multiple times
o Call the number “222”

146
Managing SOB in lung cancer patients
• Pharmacological
o Opioids seem to have a class effect
Kapanol is the only PBS opioid for SOB but shouldn’t start on this because pretty big dose
• Long acting opioid, daily dose
Can start on ordine (fast acting version of morphine)
• Non-pharmacological
o Oxygen therapy
Only indicated if hypoxic
If not hypoxic --> can provide symptomatic relief (due to flow of air) but not indicated as can be
dangerous (can cause hypercapnia, trip over wires)
• Can use a handheld fan in front of the face which is useful
o Dyspnoea action plan
Sit upright, try to focus on your breathing
Use a handheld fan
Put on O2 if available
Try 1-2mg PO ordine and wait 15 min
Try 0.5-1mg SL lorazepam and wait 15 mins
If still distressed, take ordine and lorazaepam together and wait 30 mins
If remains distressed, consider 2.5mg SC morphine

147

You might also like